You are on page 1of 183

Grammar & Usages

iQue ideas Page 1


Table of Contents
1. Useful Grammar Terms------------------------------------------------------------3

2. Parts of Speech----------------------------------------------------------------------8

3. Nouns--------------------------------------------------------------------------------16

4. Determiners-------------------------------------------------------------------------27

5. Pronouns----------------------------------------------------------------------------34

6. Adjectives & Adverbs------------------------------------------------------------46

7. Verbs--------------------------------------------------------------------------------62

8. Prepositions------------------------------------------------------------------------82

9. Conjunctions-----------------------------------------------------------------------96

10. Subject Verb Agreement--------------------------------------------------------105

11. Modifiers--------------------------------------------------------------------------112

12. Parallelism------------------------------------------------------------------------116

13. Redundancy and Wordiness----------------------------------------------------119

14. Punctuation-----------------------------------------------------------------------124

15. Practice Exercise I---------------------------------------------------------------134

16. Practice Exercise II--------------------------------------------------------------147

17. Explanatory Answers-----------------------------------------------------------169

iQue ideas Page 2


1
Useful Grammar Terms
Before you undertake a study of Grammar and Usage. it is a good idea to familiarize yourself with these
terms in Grammar. These terms may occur some places in this book and in other grammar books that you
may refer to. This list and the explanations and examples will then become very handy. Or, you may decide
to just read through this so that it may serve as the foundation of your further study:

Abstract nouns: Abstract nouns are names given to ideas that are intangible. As abstract nouns are qualities
and characteristics, they are generally non-count nouns; they may not have a plural form. However you must
have already noticed that quality and characteristic are already used in the plural in the earlier sentence. But
abstract nouns like courage. pride, goodness etc., are non-count and singular.

Action verbs: These words show action. There are also verbs that do not show action. But Action Verbs are
verbs like, sing, run, stand that show physical action. Some verbs like love, admire, approve etc. are also
action verbs, as they show mental action.

Adjective phrase/clause: This is a group of words beginning with these words: who, whose, whom, which,
that, when and where (no other word) and will describe/rename the word immediately before it. (... music
that everybody likes, the man whom you met yesterday, my friend whose parents are in the US etc.

Adjectives: These are words that describe/modify or affect the meaning of nouns and pronouns. These
words help us understand nouns or pronouns better. In a 'small car', small helps us understand the car better
— which type. There are seven (7) words in the English language that are always adjectives. They are: a, an,
the, my, our. your, and their.

Adverb phrase/clause: This is a group of words that modifies a verb, adjective, or another adverb in that
sentence. The most frequent are the ones that modify verbs. When you have sentence beginning with the
following words you can be sure that it is an adverb clause: after, although, as, as if before, because, if, since,
so that, than, though, unless. until, when, where, and while. These are just some of the more common adverb
clauses.

Adverbs: These are words that modify verbs, adjectives, and other adverbs. The help us understand verbs,
adjectives or other adverbs in a better way in terms of how (manner), when (time), where (place), how much
(degree), and why (cause). Why is a common one-word adverb. She drives slowly. She drives very slowly.
She is very slow. Slowly and very are adverbs.

Antecedent: Antecedent is word that is replaced by a noun/pronoun. Study these sentences — The girl was
reading a novel. She was reading it for the second time. In the second sentence, She replaces the girl, and it
replaces the novel. She is thus the antecedent of the girl, and it is the antecedent of the novel. In the sentence,
He likes his new bike, his is the antecedent of he. So the antecedent will always be placed before the
noun/pronoun it replaces.

Appositive: An Appositive is a word or a group of words that identifies or renames the noun or pronoun.
E.g., Anjali, our counselor, is very helpful. our counselor renames Anjali. Appositives (our counselor) is
always paced right next to the noun/pronoun it renames. Generally commas can set off the appositive.
However, when the appositive is used to identify the noun before it rather than to rename it the commas
cannot be used. Study these: 1. My boss, Manish, is bald. 2. My boss Manish is bald. Both are correct

iQue ideas Page 3


sentences. The first sentence (with commas) renames my boss. I have only one boss. In the second sentence -
I have several bosses, but I am talking about a particular boss - my boss Manish.

Articles: Adjectives a, an, and the are called Articles. A and An are indefinite articles. The is definite article.

Case: The different form in which a pronoun is used in different parts of the sentence is called its Case.
There are three cases: nominative, objective, and possessive. Nominative case pronouns are I, she, he, we,
they, and who. They are used as subjects. Objective case pronouns are me, her, him, us, them, and whom.
They are used as direct objects. indirect objects, or objects of the preposition. You and it are both nominative
and objective case. Possessive case. pronouns are my, mine, your, yours, his, her, hers, its, our, ours, your,
yours, their, and theirs. They are used to show ownership.

Clause: A clause is a group of words having a subject and a verb. An independent clause can stand alone as
a sentence. E.g., Jaimin loves teaching; he really enjoys pursuing students.' This sentence has two
independent clauses.

A dependent clause is always used as some part of speech. It can function as an adjective, adverb, or noun. It
cannot stand alone as a sentence. E.g., Because I forgot my homework, I got sent home. Because I forgot my
homework, is a dependent clause and, I got sent home is an independent clause. Another example: Raashi,
who reads well, is nice to talk to. Raashi is nice to talk to is an independent clause and who reads well is a
dependent clause.

Co-ordinate conjunction: They join words, phrases, clauses, or sentences of equal rank. The co-ordinate
conjunctions are the following: and, but, or, nor, for, and yet. (For and yet can only join clauses/sentences)

Collective nouns: Collective nouns name groups, such as team, class, and crew.

Complex sentence: A sentence made up of an independent clause and a dependent clause. Example: Manish
was teaching (independent clause which can stand alone and snake sense) as I left the room (dependent
clause which must he attached to the independent clause to make sense).

Compound nouns: They are made up of more than one word, such as Dining Hall, Girls' common room,
The Right to Information Act, etc.

Compound sentence: A combination of two or more independent clauses (often joined by a co-ordinate
conjunction. E.g., [The lightning flashed (independent clause) and (conjunction) the rain fell (independent
clause).]Only clauses closely related in thought should be joined to make a compound sentence.

Concrete nouns: They name things that exist physically as sidewalk, bird, toy, hair, and rain.

Conjunction: A word that joins other words, phrases (groups of words), or clauses (groups of words with a
subject and verb). There are three types: Co-ordinate conjunctions, Subordinate conjunctions, and
Correlative conjunctions.

Correlative conjunctions: Some authorities treat them as a type of co-ordinate conjunctions. However,
correlative conjunctions come in two parts: either-or, neither-nor, both-and, not only-but also, and whether-
or.

Count nouns: They are nouns that can be counted. You can use a, an, many, or a number before count
nouns.

Demonstrative pronouns: They are pronouns that point out. They include: this, that, these, and those. For
example: That is my friend. 1 like this not those.

iQue ideas Page 4


Dependent clause: A clause that is always used as some part of speech. It can be an adjective, adverb, or
noun and cannot stand alone as a sentence. Refer to the term clause above.

Determiners: Determiners are used to identify things in further detail. They are used in front of nouns to
indicate whether the reference is to something specific or something of a particular type. The definite and
indefinite articles a/an/the are all determiners. A determiner let people know exactly which thing(s) or
person/people is being talked about. Examples: a few, any, all, enough, many, more, much, several, some
etc. are determiners.

Direct object: The direct object receives the action performed by the subject. Hence. it always come after an
action verb (only action verbs). Example: The car hit the tree. To find the direct object, ask whom or what to
the verb. The car hit whom or what? The answer is 'tree'. so tree is the direct object. The direct object must
be a noun or pronoun.

Elliptical clause: An adverb clause that uses than and as to introduce the clause.(Ellipsis refers to the
omission from speech or writing of a word or words that are superfluous or able to be understood from
contextual clues. )That means an Elliptical clause has some of its parts understood but not stated. Example:
You are smarter than I. (am smart.) The term is useful for understanding sentences like. He is taller I etc.

First person pronouns: These are pronouns that refer to the speaker or speakers. First person pronouns
include: I. my. mine, me, myself, we, our, ours, us, ourselves. They are also considered personal pronouns.

Gerund: Gerund is a form of verb that always ends in ing and is used as a noun. Example: Eating is fun The
gerund can do almost everything that a noun does.

Helping verbs: They are words (verbs) used in forming the tenses, moods, and voices of other verbs. They
are also called auxiliary verbs: Group 1: is, am. are. was, were, be, being, been; Group 2: has, have, had;
Group 3: do. does. did; Group 4: shall, will, should, would; Group 5: may, might, must, can, could.

Indefinite pronouns: Indefinite pronouns point out generally, instead of pointing out specifically. Indefinite
pronouns include such words as another, any. anybody. anyone, anything, both, each, either, everybody,
everyone. everything, many, neither, nobody. none, no one, one, other, others, some, somebody, and
someone.

Independent clause: A clause that can stand alone as a sentence. See the entry for clause above.

Indirect object: An object that is really part of a prepositional phrase in which the preposition to or for is
not stated but understood. It tells to whom or for whom something is done. The indirect object always comes
between the verb and the direct object. Example: He gave her a gift. It is used with verbs such as give, tell,
send, get. buy. show. build, do, make, save, and read.

Infinitive: The form of the verb with to. Examples: to be, to see. to be seen, to be eaten.

Intensive (Emphatic) pronouns: They are the 'self words like: myself. yourself, yourselves, himself herself,
itself, ourselves, and themselves. It is used to emphasize its antecedent. An example would be: I did it
myself. The same words have another function; then they are called Reflexive pronouns. See the entry
below.

Interjection: A word or word group that shows feeling. A comma follows a mild interjection; a strong
intersection is followed by an exclamation mark. Examples: Well, we will soon be home. Oh! I didn't know
he had died.

Interrogative pronouns: They begin questions. Who, whom, whose, which, and what.

iQue ideas Page 5


Intransitive verbs: When a verb does not need an object it is called Intransitive. Intransitive sentences will
have no receiver of the action of the verb. They are further classified as intransitive complete or intransitive
linking. In short, complete verbs, or linking verbs: the examples are respectively, He runs fast. He is fast.

Linking verbs: These are verbs showing (state of being verbs) that something exists; they do not show
action. Some common linking verbs include: is, am, are, was, were, he, being, been, seem. look, feel, and
become.

Mass nouns: Nouns that are not countable - words like, water, milk, happiness. coffee etc.

Nominative case pronouns: Nominative case is the subject form of the Pronoun: I, she, he, we, they. and
who. They are used as subjects.

Noun: A word that names a person, place, or thing.

Noun adjuncts: Nouns used as adjective or nouns used to describe another noun.. They tell us whose or
what kind For example: vegetable soup.

Noun clause: A dependent clause that can be used in the same way as a noun or pronoun. Some of the words
that introduce noun clauses are that, whether, who. why, whom, what, how, when. whoever, where. and
whomever. (Notice that some of these words also introduce adjective and adverb clauses.) To check a noun
clause substitute the pronoun it or the proper form of the pronouns he or she for the noun clause.) Examples:
I know who said that. (I know it.) Whoever said it is wrong. (He is wrong.) Sometimes a noun clause is used
without the introductory word. Example: I know that he is here, is commonly written as. I know he is here.

Object of the preposition: The noun or pronoun that comes after a preposition examples: on the table, after
her etc.

Objective case pronouns: These are the object pronouns —me, her, him, us, them, and whom. They are
used as direct objects. indirect objects. and objects of the preposition. ( You and it are both nominative and
objective case.)

Participial adjectives: These are verb forms used as adjectives. Examples: the lost world, the crying child.

Participle: A verb form that is an adjective and ends various ways. A present participle always ends with ing
as does the gerund. but remember that it is an adjective.

Personal pronouns: They are. I. we: you: he, she, it, they, and their other forms- objective case, possessive
case etc.

Phrase: A group of words used as a sentence part. Phrases not have a subject and a verb as clauses have.
They can functions as nouns. adjectives. or adverbs. Examples: Alter having eaten, in answer to your
question, to hike me home etc.

Possessive case pronouns: my. mine, your, yours, his, her, hers, its. our, ours, your, yours, their. and theirs.
The) are used to show ownership. The difference between my and mine is: my is possessive determiner also
called possessive adjective, and mine is the possessive pronoun. Examples: my book, the book is mine.

Preposition: A word that shows the relationship between its object and another word in the sentence. Words
are prepositions if they have an object to complete them. To decide if the word in question is a preposition.
say the preposition followed by whom or what. If a noun or a pronoun answers the question. the word is a
preposition. If there is no noun or pronoun to complete the sentence. the word is not a preposition. Example:
The boy stood up. Up what? There is no answer: therefore up is not a preposition. The boy ran up the stairs.
Up what? The stairs answers the question: therefore. up is a preposition.

iQue ideas Page 6


Prepositional phrase: A phrase that starts with a preposition and ends with a noun. e.g., before coming to
the class...

Pronoun: A word that replaces a noun, or a group of words used as nouns.

Relative pronouns: They are used to join dependent clauses to independent clauses. Relative pronouns
include. who, whose, whom, which, and that. Example: He found the book that he had lost. That joins the No
clause, together into one sentence.

Reflexive pronouns: The personal pronouns myself, yourself, yourselves, himself. herself. melt. ourselves,
and themselves are compound personal pronouns. combining the personal pronoun with self or selves. when
the action in the verb reflects back to the subject. the reflexive pronoun is used. For example: lie hurt
himself.

State of being verbs (linking verbs): They show that something exists; they do not shins action Some
common linking verbs include: is. am. are. was. were. he, being. been. seem. look. feel, and become.
Example : She looks pretty.

Subject: A word that tells who or what about the verb. When finding the subject and the verb in a sentence,
always find the verb first and then say who or what followed by the verb. Example: The bell rang. Find the
verb - rang. Now say who or what rang? The bell rang. Bell is the subject.

Subordinate conjunctions: They join dependent clauses to independent clauses. Some common subordinate
conjunctions are after, although, as, as if, because, before, if, since, so that, than, unless, until, when, where,
and while.

Third person pronouns: The third person pronouns are: he. his, him, himself she, her, hers, herself it, its,
itself, they, their. theirs, them, themselves.

Transitive active verbs: These are the verbs in sentences with a direct object. Example: The child broke the
toy. The subject is the doer and the direct object is the receiver of the action.

Transitive passive verbs: These have the subject receiving the action with the doer in a prepositional phrase
(or omitted in the sentence). Example: The toy was broken by the child. (The toy was broken.)The toy is the
subject of this sentence. The verb in the transitive passive voice always has is, am, are, was, were, be, being,
or been as an auxiliary or helping verb.

Transitive verbs: These are verbs that have subjects or objects that receive an action. They are either active
voice or passive voice.

Verbs: Words that show action (mental or physical) or state of being. Most verbs are action words, but a few
verbs indicate state of being or existence. She looked at herself in the mirror. Here, looked is an action verb.
She looks pretty. Here, looks is a stative or linking verb.

iQue ideas Page 7


2
Parts of Speech
In traditional grammar, English words are classifiedaseight parts of speech: the noun, the pronoun, the verb,
the adjective, the adverb, the preposition, the conjunction, and the interjection.

The part of speech does not explain what the word is. but how the word functions in the sentence. So. the
same word can be a noun in one sentence and a verb or adjective in the next.

 abook of selected poems (in this context. 'book' is a noun)


 book early to avoid disappointment. (in this context 'book is a verb)
 book smart but not street smart or book profits (in these contextsbook, as well as street, is an adjective)

So, when we say 'book' is a noun, or 'before' is a preposition etc., we are in fact referring to the most
common function of that word and nothing more.

Noun

A noun is a word used to name a person, animal, place. thing, and abstract idea. Nouns are usually the first
things we learn in any language. The italicized words in the following sentences are all nouns:

 Last week my dad bought a new car.


 My wife is an AssociateProfessor
 The trainconductordemanded to see the passengers 'passes.
 According to Plutarch, the library at Alexandria was destroyed in 48 B.C.
 Philosophy is of little comfort to the starving.

A Noun can function in several ways in a sentence. They can be: a subject, a direct object, an indirect object.
a subject complement, an object complement. an appositive, an adjective or an adverb.

Noun — Number (singular and plural)

Most nouns change their form to indicate number (plural) by adding "-s" or "-es", as illustrated in the
following, pairs of sentences:

 He rarely speaks the truth.


 She does not believe that truths are self-evident.
 a fashion that is an echo of an earlier style.
 found echoes of past civilizations in the artifacts.

There are other nouns, which form the plural by changing the last letter before adding (' )s". Some words
ending, in "f' form the plural by deleting "f' and adding "yes," and words ending in "y" form the plural b)
deleting the "y" and adding "ies," as in the following pairs:

 wharf – wharves
 city — cities

Other nouns form the plural irregularly. Native speakers know these forms instinctively. When in doubt,
consult a good dictionary.

iQue ideas Page 8


 child—children
 mouse – mice

Noun - Possessive Nouns

Usually, nouns become possessive by adding a combination of an apostrophe and the letter "s." (`s)

If the singular noun does not end in "s", add an apostrophe and "s,"(`s)to get its possessive.

 Parag's laptop
 prime minister's prerogative
 student's worried face

However, when a singular noun ends in "s", you may use either the (`s) or merely an apostrophe (').

 the bus's seats


 the bus' seats
 Vikas's books
 Vikas' books

Plural nouns that do not end in "s" become possessive by adding an (`s).

 children's movie
 sheep'spen
 jury's verdict
 women's team

The possessive case of a plural noun that end in "s"are formed by adding an apostrophe (`). All the examples
below are plural nouns ending in "s"

 dogs' barking
 babies ' squalling
 janitors' room
 girls' common room

When you look at the above examples, you will notice that a noun in the possessive case frequently functions
as an adjective modifying another noun. In "student's face", student's (adj) modifies face(noun)

Nouns — Types

There are many different types of nouns. We capitalize some nouns, such as "Mumbai" or "Delhi," and do
not capitalize others, such as "dog" or "tree" (unless they appear at the beginning of a sentence). In fact,
grammarians have developed a whole series of noun types, including the proper noun, the common noun, the
concrete noun. the abstract noun, the countable noun (also called the count noun), the non-countable noun
(also called the mass noun). and the collective noun. You should note that the classification is not rigid. A
noun can at times be proper or common, abstract or concrete, and countable or non-countable or collective.

Proper Nouns

You always write a proper noun with a capital letter, since the noun represents the name of a specific person.
place. or thing. The names of days of the week, months, historical documents, institutions, organizations.
religions, their holy texts and their adherents are proper nouns. A proper noun is the opposite of a common

iQue ideas Page 9


noun. Examples: Manish, Mumbai, Moon, Monday, March, Rig Veda, Mithibai College. Greenpeace,
Christianity, the Gita, Hindus etc...

Proper nouns are not normally preceded by an article or other limiting modifier as any or some. Nor are the)
usual!). pluralized. But the language allows for exceptions. Proper nouns may occasionally have a definite
article as part 01 the name, as in the case of The Titanic, The Humane Society, and The Plaza.

Common Nouns

A common noun is a noun referring to a person, place, or thing in a general sense -- usually, you it is not
written with a capital letter unless it begins a sentence. A common noun is the opposite of a proper noun.

 According to the notice, the test will be in a three days.


 The book on the table is mine.

Concrete Nouns

A concrete noun names anything (or anyone) that you can perceive through your physical senses: touch,
sight. taste. hearing, or smell. A concrete noun is the opposite of an abstract noun.

 The teacher handed the books to the student.


 Whenever they take the child to the park. she spends hours chasing the butterflies.
 The salesman urged the woman to buy the new refrigerator.

Abstract Nouns

An abstract noun names anything that you cannot perceive through your five physical senses. and is the
opposite of a concrete noun.

 Buying the printer was an afterthought.


 Her nostalgia about childhood
 Justiceis often denied.

Countable Nouns

A countable noun (or count noun) has singular and plural forms, and it names anything (or anyone) that you
can count. Countable nouns are the opposite of non-countable nouns and collective nouns.

 We printed the books pink and the exercisesblue.


 He spends every weekend in the public library.
 She found some twenty-five paise coins in her old wallet.

Non-Countable Nouns

A non-countable noun (or mass noun) does not have a plural form, and which refers to something that you
could (or would) not usually count. A non-countable noun always takes a singular verb in a sentence.

 Joseph Priestly discovered oxygen.


 Oxygen is essential to human life. (Notice the singular verb is.)
 The furniture is on sale.
 She cooked rice for dinner

Collective Nouns

iQue ideas Page 10


A collective noun names a group of things, animals, or persons. You could count the individual members of
the group, but you usually think of the group as a whole or as one unit. You need to be able to recognize
collective nouns in order to maintain subject-verb agreement.

 Apack of wolves is on the prowl in the night.


 The team is playing its first match today.
 The class was startled by the bursting light bulb. (Notice the singular verb)

Pronoun

A pronoun can replace a noun or another pronoun. You are use pronouns like he, which, none, and you to
make your sentences less cumbersome and less repetitive.

Grammarians classify pronouns into several types, including the personal pronoun, the demonstrative
pronoun, the interrogative pronoun, the indefinite pronoun, the relative pronoun, the reflexive pronoun and
the intensive pronoun.

Personal Pronouns

A personal pronoun refers to a specific person or thing and changes its form to indicate person, number,
gender, and case.

Subjective Personal Pronouns

A subjective personal pronoun indicates that the pronoun is acting as the subject of the sentence. The
subjective personal pronouns are I, you, she, he, it, we, you, they.

 You are undoubtedly the best student in the class.

Objective Personal Pronouns

An objective personal pronoun indicates that the pronoun is acting as an object of a verb, compound verb,
preposition, or infinitive phrase. The objective personal pronouns are: me, you, her, him, it, us, you, and
them.

 She forced him to speak the truth.(him is the direct object of the verb forced)
 The leader will address you in a short while. (you is the direct object of the verb address)
 Give the list to me. (me is the object of the preposition to)
 She was glad to see him studying. (him is the object of the infinitive phrase to see.)

Possessive Personal Pronouns

A possessive pronoun indicates that the pronoun is acting as a marker of possession and defines who owns a
particular object or person. The possessive personal pronouns are mine, yours, hers, his, its, ours, and theirs.
Note that possessive personal pronouns are very similar to possessive adjectives like my, her, and their.

 The best gift is mine.


 This is yours.

Demonstrative Pronouns

A demonstrative pronoun points to and identifies a noun or a pronoun. This and these refer to things that are
nearby either in space or in time, while that and those refer to things that are farther away in space or time.

iQue ideas Page 11


The demonstrative pronouns are this, that, these, and those. This and that are used to refer to singular nouns
or noun phrases and these and those are used to refer to plural nouns and noun phrases.

 This must not continue.


 Three students wanted these.

When the same words are attached to (modify) nouns, they are called demonstrative adjectives.

Interrogative Pronouns

An interrogative pronoun is used to ask questions. The interrogative pronouns are who, whom, which, who!
and the compounds formed with the suffix ever (whoever, whomever, whichever, and whatever).Note that
either which or what can also be used as an interrogative adjective, and thatwho, whom, or which can also be
used as a relative pronoun.

You will find who, whom used to refer to people, and which and what used to refer to things and to animals.

Who acts as the subject of a verb, while whom acts as the object of a verb or a preposition

 Who wrote the novel the Fountainhead? (subject of wrote)


 Whom do you think we should invite? (object of invite)
 To whom do you wish to speak? (object of preposition to)

Relative Pronouns

You can use a relative pronoun to link one phrase or clause to another phrase or clause. The relative
pronouns are who, whom, that, and which. The compounds whoever, whomever, and whichever are also
relative pronouns.

 The candidate who clears the CAT is not always selected.

In this sentence, the relative pronoun is the subject of the verb clears and introduces the subordinate clause
whoclears the CAT. This subordinate clause acts as an adjective modifying candidate.

Indefinite Pronouns

An indefinite pronoun refers to an identifiable but not specified person or thing. An indefinite pronoun
conveys the idea of all, any, none, or some.

The most common indefinite pronouns are all, another, any, anybody, anyone, anything, each, everybody.
everyone, everything, few, many, nobody, none, one, several, some, somebody, and someone.

 Many were invited.All came.


 Everything was well arranged.Someone is responsible.
 Everybody who's anybody is here.

Reflexive Pronouns

You can use a reflexive pronoun to refer back to the subject of the clause or sentence. The reflexive
pronouns are myself, yourself herself, himself itself ourselves, yourselves, and themselves. Note each of
these can also act as an intensive (emphatic) pronoun.

 I asked myself why I failed. (Reflexive)


 The Prime Minister himself said that corruption would be abolished.(Intensive)

iQue ideas Page 12


Verb

The verb is perhaps the most important part of the sentence. A verbor compound verb asserts something
about the subject of the sentence and express actions, events, or states of being. The verb or compound verb
is the critical element of the predicate of a sentence.

 She does her homework regularly. (does describes the actions of the subject)

In late October you will he taking the CAT. (the compound verb will he taking describes an action in the
future)

 Our math teacher was Manish. and I remember his famous one-liners.
In this sentence, the verb was (the simple past tense of is) identifies a particular person and the verb
remember, describes a mental action.)
 The database was destroyed in the cyber attack.
In this sentence, the compound verb "was destroyed" describes an action which took place in the past.

Adjective

An adjective modifies a noun or a pronoun by describing. identifying. or quantifying words. An adjective


usually comes before the noun or the pronoun which it modifies. Some nouns. many pronouns, and many
participle phrases can also act as adjectives. In the sentence

 The monkey-shaped balloon fascinated the child.


 She painted her kitchen walls in her favorite pink.
 The muffled sounds of the gun.
 The money hidden under the pillow.

Grammarians also consider articles the, a, an to be adjectives.

Possessive Adjectives

A possessive adjective - my, your, his. her, its, our, their - is similar or identical to a possessive pronoun:
however, it is used as an adjective and modifies a noun or a noun phrase. as in the following sentences:

 I can't complete my assignment. (possessive adjective my modifies assignment)


 What is your phone number.(possessive adjective your modifies assignment the noun phrase phone
number)
 We have lost our way in this city. (the possessive adjective our modifies way)

Demonstrative Adjectives

The demonstrative adjectives this. these, that. those. and what are identical to the demonstrative pronouns,
but are used as adjectives to modify nouns or noun phrases, as in the following sentences:

 I have got that book. (that modifies the noun hook)


 This book needs to be read carefully. (this modifies hook)

Interrogative Adjectives

An interrogative adjective (which or what) is like an interrogative pronoun, except that it modifies a noun or
noun phrase rather than standing on its own (see also demonstrative adjectives and possessive adjectives):

 What book are you reading? (what modifies hook and is attached to the noun hook)

iQue ideas Page 13


Indefinite Adjectives

An indefinite adjective is similar to an indefinite pronoun, except that it modifies a noun, pronoun, or noun
phrase.

 Many people believe that big businesses are under-taxed. (many modifies the noun people)

Adverb

An adverb can modify a verb, an adjective, another adverb, a phrase, or a clause. An adverb indicates
manner, time. place, cause, or degree and answers questions such as how, when, where, howmuch. While
some adverbs can be identified by their "/y - suffix (e.g., 'quickly), others must be identified by analyzing
their grammatical relationships within the sentence or clause. Unlike an adjective, an adverb can be found in
various places within the sentence.

 She quickly apologized. (quickly modifies the verb apologized)


 The boldly spoken words would return to haunt her.( boldly modifies the adjective spoken.)
 She should have done it more quickly. (the adverb more modifies the adverb quickly.)
 Unfortunately, she could not pacify him. (the adverb unfortunately modifies the entire sentence.)

Conjunctive Adverbs

You can use a conjunctive adverb to join two clauses together. Some of the most common conjunctive
adverbs are also, consequently, finally, furthermore, hence, however, incidentally, indeed, instead, likewise,
meanwhile, nevertheless, next, nonetheless, otherwise, still, then, therefore, and thus. A conjunctive adverb
is not strong enough to join two independent clauses without the aid of a semicolon. So, remember to use the
semicolon when a conjunctive adverb comes in between two independent clauses.

 The government has cut education budgets; consequently, class sizes have been increased.
 Everything looked fine at first; then, there is always something unpredictable about them.

Preposition

A preposition links nouns, pronouns and phrases to other words in a sentence. The word or phrase that the
preposition introduces is called the object of the preposition.

A preposition usually indicates the temporal (time), spatial (space) or logical relationship of its object to the
rest of the sentence as in the following examples:

 The book is on the table.


 The book is beneath the table.
 She held the book over the table.
 She read the book during class.

In each of the preceding sentences, a preposition locates the noun book in space or in time. The most
common prepositions are about, above, across, after, against, along, among, around, at, before, behind,
below, beneath, beside, between, beyond, but, by, despite, down, during, except, for, from, in, inside, into,
like, near, of, off, on, onto, out, outside, over, past, since, through, throughout, till, to, toward, under,
underneath, until, up, upon, with, within, and without.

Conjunction

A conjunction links words, phrases, and clause.:

iQue ideas Page 14


 I brought the tablet and the book.
 Call me when you are done.

Coordinating Conjunctions are:

but, or. nor, for, so. let - they join individual words, phrases, and independent clauses. Note that but
and.forare also prepositions.

 Pooja and Anjali are friends. (joins nouns)


 They are friends, for they work together. (joins two sentences)
 Pooja says that she spends most her time listening to Anjali and learning from her. (joins two phrases)

Subordinating Conjunctions

A subordinating conjunction introduces a dependent clause and indicates the nature of the relationship
between the independent clause (main clause) and the dependent clause (sub clause).

The most common subordinating conjunctions are after, although, as, because, before, how, if, once, since,
than, that, though, till, until, when, where, whether, and while.

 Though they are very good friends, they do have arguments occasionally.

Correlative Conjunctions

Correlative conjunctions are in two parts and they link equivalent sentence elements. The most common
correlative conjunctions are both... and, either... or, neither... nor„ not only... but also, so... as, and whether...
or.

 CPLC is popular both in the western suburbs and in the central suburbs.

Interjection

An interjection is a word used to convey emotion. It is not grammatically related to any other part of the
sentence. We usually follow an interjection with an exclamation mark, but it is not compulsory.

 Ouch, that hurt!


 Oh no, I forgot that the test was today.
 Hey! Put that down!
 He has a new car, eh?

iQue ideas Page 15


3
Nouns
A noun is a word used to name a person, animal, place, thing, and abstract idea. Nouns are usually the first
things we learn in any language.

She is an author OR She is an authoress?

Gender

Many common nouns, like "professor" or "doctor" can refer to men or women. Some of them change
depending on their gender. For example. an "author" is a man while an "authoress" is a woman. However,
this use of gender-specific nouns is rather rare today. Hence an 'actor' a 'waiter' an 'author' an 'engineer' etc.
(most nouns referring to occupation) can be used for men and women.

Correct: She is an excellent actor.


Correct: She is an excellent actress.

James's Book OR James' book?

Possessive

Usually, nouns become possessive by adding a combination of an apostrophe (') and the letter "s." However,
one needs to know certain conventions:

Singular nouns take apostrophe and "s" to form the possessive.

The red is tie is Raj 's.


The red tie is the House Leader's.

Plural nouns take apostrophe and "s" to form the possessive.

The children's shoes were scattered on the floor.


The men's hockey match is tomorrow.
The jury 'sverdict is final.

Singular Nouns ending in "s" take an apostrophe (only), or an apostrophe and "s" to form the possessive.

Correct: Professor Srinivas' sense of humour is well-known.


Correct: Professor Srinivas's sense of humour is well known.

Correct: The bus's frequency is low.


Correct: The bus' frequency is low.

Plural Nouns ending in "s" take an apostrophe (only) to form the possessive.

He was kept awake by the dogs' barking. (several dogs)


I couldn't locate the squirrels' nest. (several squirrels in one nest)

Correct: The buses' roofs are low.

iQue ideas Page 16


Correct: The boys' common room is full at this hour.
Incorrect: The boy's common room is full at this hour.
Incorrect: The bovs's common room is full at this hour.

Incorrect: The new trains 's seats are comfortable.


Correct: The new trains' seats are comfortable.

The media is responsible OR The media are responsible?

Countable Nouns

A countable noun names anything that can be counted and has a singular and a plural form. Singular
countable nouns become plural in several ways. Study these:

Singular Plural
Snake Snakes
Ski Skis
Watch Watches
James Jameses
Child Children
Deer Deer
Nucleus Nuclei
Thesis Theses
Crisis Crises
Index Indices
Criterion Criteria
Phenomenon Phenomena
Knife Knives
Dwarf Dwarfs
Alumna (feminine) Alumnae
Alumnus (masculine) Alumni

Media is the plural of medium. The agencies of mass communication are often referred individually as 'the
print media'. Or, at times collectively to refer to all agencies as a mass noun: The media is responsible for
making the event a success, is a correct sentence. Similarly though data is the plural of datum, the word data
is now widely used as a singular or mass noun, and this use is accepted as correct.

She made fewer mistakesORShe made lesser mistakes?

Non-Countable Nouns — also called Mass Nouns.

Non-countable nouns have these characteristics: they cannot be counted; some of them do not have the plural
form: they take singular verbs; they are similar to collective nouns.

We did not move the furniture.


The furniture is on sale.

Usage Notes:

Count nouns are used with: a, an, the: many, few/fewer, number: this, that, every, each, either, neither: these.
those. some, any, enough, a number of

iQue ideas Page 17


Non-count nouns are used with: much, less/lesser, this, that, some, any, enough, amount of

Correct: She made fewer mistakes in her paper today


Incorrect: She made less mistakes in her paper today.
Correct: They have less money than we have.

Incorrect: They have fewer money than we have.

Mistakes is a countable noun., money is not countable though currency notes and coins are countable. ‘some
experience’ OR ‘many experiences?’

We should note that some words can be either a count noun or a non-count noun depending on how they're
being used in a sentence. Hence when a noun, which is generally non count, is used as a count noun it gets a
plural form as well. E.g., foodstuffand foodstuffs.

He got into trouble. (The noun trouble is used as an uncountable noun)


He had many troubles. (The noun troubles is here used as a countable noun)
Experience (non-count) is the best teacher.
We had many exciting experiences (countable) in college.

Whether these words are count or non-count will determine whether they can be used with articles (a, an,
the) and determiners (a few, some, etc.) or not. We would not write "He got into the troubles," but we could
write about The troubles that the poor have ....".

Since 'some' as a determiner can precede both the countable and the uncountable nouns, both 'some troubles'
and 'many troubles' are correct depending on the context.

Water AND Waters.


Here is a list of Mass Nouns (non-count) for you to consider. We do not use the plural form of most of these
words in common speech and writing.

Materials:
wood, cloth, ice, plastic, wool, steel...this category can get a plural form.

Foodstuffs:
water, milk, wine, beer, sugar, rice, meat, cheese, flour ... this category can get a plural form.

Activities (including sports, languages, fields of study, natural events): reading, boating, smoking, dancing,
soccer, hockey, weather, heat, sunshine, electricity, biology, history. mathematics, economics, poetry,
Chinese, Spanish, English, luggage, equipment, furniture, experience, applause, photography, traffic, harm,
publicity, homework, advice this category cannot get a plural form

There are new beers being introduced every day. (New brands of beer)
Still waters run deep. (As distinguished form moving ones)
The Indians are famous for their curries.
The rains came early this year.
These foodstuffs are exceedingly rich and can harm your heart.

Sunlight is good for your skin OR The sunlight is good for your skin?
Use of the article a, the with non count nouns.

iQue ideas Page 18


It is appropriate to precede the above nouns with a definite or indefinite article.
the weather
an applause (that deafened us)
a harm (which cannot be undone)
the sunlight ( is good for your skin)

But they frequently appear with zero article:


Smoking is bad for you.
Sugar is sweet.
Experience is the best teacher.
Sunlight is good for your skin.

Speed AND Speeds


Plural Abstract Nouns.
Abstract Nouns refer to ideas or concepts. Some of them can never be pluralized and some can be:
These cannot be pluralized: peace. warmth, hospitality, information. anger, education, conduct, courage.
leisure, knowledge, safety ....
These can be pluralized: speed, experience, time, friendship, trouble, work. culture, virtue, taste, Evil.
liberty. democracy, death, grief piety ...

The actual speeds specific to a particular aircraft ...


All my friendships have been successful.
These are difficult times.

'hair' AND 'hairs'


When a non-count noun is used to classify something. it can be treated as a count noun. Almost all mass
nouns (there are exceptions) can become count nouns this way.

They import some nice beers.


There were some real beauties in that class.
We had some serious difficulties in English.

But we can never have ‘furrnitures'. 'informations', knowledge', Isolinesses', or 'chaoses'.

Correct: He advised me several times on this project.


Correct: He gave me his advice on this project.
Incorrect: He gave me his advices on this project.
Incorrect: Please get me two waters. (two glasses of water)
Correct: Your hair looks great. Correct: Your hair looks great except for the several white hairs.

Collective Nouns

A collective noun names a group (of things, animals, or persons). A collective noun is similar to a non-
countable noun except that we can clearly count the constituents or the members in that group. In the
following sentences the collective nouns are italicized.

A flock of sheep is grazing in the meadow.


The jury is delivering the verdict today.
The committee meets every Monday.

iQue ideas Page 19


Collective nouns generally take a singular verb:

Correct: A group of boys is waiting outside.


Correct: A group of boys and girls is waiting outside.

Incorrect: A group of boys are coming to meet you.


Incorrect: The group of boys and girls are large.

A group of boys and girls is just as collective as a group of boys or a herd of cattle and other collective
nouns.

The team has won the match OR The team have won the match?

There are collective nouns (usually the single word ones like family. committee etc with living members)
which are singular when we think of them as groups and plural when we think of the members within those
groups.

Audience, hand, class, committee, crowd, dozen, family, jun. public, stall, team etc.

All the above collective nouns can take either the singular or the plural verb depending on the way they are
used in the context. Changing (singular or plural) the verb will alter the meaning at times, and create errors at
times.

Correct: The team has won the match.


Incorrect: The team have won the match.

It is the collective effort of the team that led it to victory. Hence the plural verb will be unsustainable in the
context it will mean the members have individually, or each member separately, has won the match, which is
not possible.

When the unit is implied the singular verb is correct and when the members are considered the plural verb is
correct.

Correct: The jury is giving the verdict today.


Correct: The jury are taking their seats.

Incorrect: The jury are giving the verdict today.


Incorrect: The jury is taking their seats.

(A few other collective nouns: army, audience, board, cabinet, class, committee, company, corporation,
council, department, faculty, family, firm, group, jury, majority, minority, navy, public, school, society,
team, troupe etc.)

M/S Tata Motors is in Pune OR M/S Tata Motors are in Pune?


Companies and organizations are generally considered singular and you will never be wrong to use the
singular.

Correct: Tata Motors has increased its profits.


Incorrect: Tata Motors have increased their profits.

iQue ideas Page 20


Tata Motors is the name of a company and takes a singular verb.

However, though Singh, Agarwal and Associates is the name of a firm of CAs, and calls for a singular verb.
some people use the plural in informal contexts. The two sentences below are on the firm's website. And
both are correct.

Singh Agarwal and Associates is a premier chartered accountancy firm in India that offers a comprehensive
range of services by chartered accountants....

M/s Singh Agarwal and Associates are distinguished providers of financial advisory services.

Competitive examinations rarely fling such debatable questions at the candidates; hence, it is wise to stick to
the singular verb for companies.

Correct: M/s Singh Agarwal and Associates is a distinguished provider of financial advisory services.

India have won the match OR India has won the match?

The names of sports teams have to be treated as plurals, regardless of the form of that name.

For example: Bangalore (Royal Challengers ), Delhi(Dare Devils), Chennai (Super Kings)Mohali (Kings XI
Punjab), Kolkatta (Knight Riders), Hyderabad(Deccan Chargers), Jaipur (Rajasthan Royals), Mumbai
(Mumbai Indians) are the teams of the IPL.

If we refer to these teams either by their names — Royal Challengers — or by their cities — Bangalore —
each will take a plural verb:

Correct: The Royal challengers have recruited a new batsman.


Correct: Bangalore have recruited a new batsman.

Incorrect: The Royal challengers has recruited a new batsman.


Incorrect: Bangalore has recruited a new batsman.

Correct: India have won the match. India are a great team.
Incorrect: India has won the match. India is a great team.

The trousers that she is wearing is expensive OR The trousers that she is wearing are expensive?

A few nouns in English appear to be plural in form but take a singular verb. These nouns have to be
recognized and given the correct verb. One cannot arrive at the verb by reasoning. We can say that the
number (singular or plural) is attributed to these nouns.

For example, I (first person singular) is considered plural in the present tense and singular in the past tense.

Correct: I have a problem with these nouns.


Correct: I was puzzled with these nouns.

Some of the nouns which are thus attributed a number are given below. Pay attention to the noun and the
corresponding verb used.

iQue ideas Page 21


Correct: The news is shocking. (news is singular)
Correct: Gymnastics is popular in China. (Gymnastics will always take a singular verb)

Correct: Economics is a subject at B.Com. (Economics, Mathematics, Statistics, Physics as subjects of study
are singular.)
Correct: My data is lost. (Data the plural of datum is now widely used as a singular noun. — even in
computer text books)
Correct: The media is responsible for this problem. (medium as a single means of communication is rarely
used)
Correct: My pair of scissors is lost. (A pair of scissors is singular)
Correct: This pair of trousers is expensive. (A pair of trousers is singular)

Correct: My spectacles are lost. (spectacles is one item but takes a plural verb)
Correct: My pants are new.
Correct: Her glasses are branded. Correct: Her scissors were lost.
Correct: The economics of the situation are worrying.
Correct: These statistics are compiled from credible sources.
Correct: The trousers that she is wearing are expensive

Cupsful OR Cupfuls?

Compound nouns

In English compound nouns are common. They are formed by the combination of two or more words. They
are of the following types:

(i) One word compound nouns like doorknob, bookmark, greenhouse, firefly, softball, keyboard. makeup.
notebook etc.
(ii) Hyphenated compound nouns like age-group, runner-up, great-grandmother, mother-in-law. paper-clip,
eight-year-old, lady-in-waiting etc.
(iii) Compound Nouns of two or more words like: dining room, motion picture, fish monger. vegetable
vendor etc.

These nouns sometimes pose a problem when pluralized. There are broad guidelines about how to pluralize
compound nouns, but we can always find exceptions to these 'rules'. Hence when ever you are in doubt,
consult a good dictionary.
(i) The one word compound nouns can be easily pluralized by adding an "s" or "es" at the end of such nouns:
doorknobs, necklaces, fireflies etc.
(ii) The hyphenated compound nouns can be pluralized by making the most important part of the word
plural: age-groups, runners-up, great-grandmothers, mothers-in-law, paper-clips, eight-year-olds, ladies-in-
waiting passers-by, lookers-on, sons-in-law etc.
(iii) Compound Nouns of two or more words can be pluralized by making the most important part of the
word* plural: dining rooms, motion pictures, .fish mongers, vegetable vendors etc.

*Words like spoonful, plateful and cupful do not follow the same principle. They form their plurals by
adding an 's' at the end, even though the principal words are spoon. plate and cup.

Correct: Add a spoonful of sugar to a cupful of milk, changes to


Correct: Add two spoonfuls of sugar two cupfuls of milk.

iQue ideas Page 22


Incorrect: I have two daughter-in-laws. (see notes above)
Incorrect: I have two son-in-laws. (see notes above)
Correct: I have two daughters-in-law. (see notes above)
Correct: I have two sons-in-law. (see notes above)

Most important part of the word can be generally (not always) identified by looking at the word that is
modified by the others. E.g., in-waiting modifies lady, in the noun lady-in-waiting. When in doubt consult a
good dictionary like Merriam Webster's Collegiate Dictionary.

Parag's and Bhavin's plans AND Parag and Bhavin's plans.

Compound Possessives

Parag and Bhavin's plans — The plans have been drawn up by Parag and Bhavin together, or they have the
same plans.

Parag's and Bhavin's plans — The plans have been drawn up individually and they have different plans.

When the one of the nouns is a pronoun use the possessive of both.
Correct: Parag's and my plans.
Incorrect: Parag and my plans.

Correct: Ami and Amit's mother. (they are siblings)


Correct: Ami's and Amit's mothers. (they are not siblings)

My friends objects to me smoking OR My friends object to my smoking?

Possessives and Gerunds

A Gerund is the ing form of any verb that functions like a noun. Examples: Swimming is a sport, Walking is
good for your health, Smoking is injurious to health etc. Swimming, Walking, and Smoking are gerunds).
The -ing form can also be a participle functioning like an adjective.

Gerunds need to be preceded by a possessive pronoun.

Incorrect: My friends object to me smoking.


Correct: My friends object to my smoking.

But also notice the difference of other nouns in combination with gerunds.

Correct: I am worried about my dad coming home late.


Correct: I am worried about my dad's coming home late.

The first sentence means I am worried about my dad, and the fact the he comes home late. The second
sentence emphasises 'dad's coming home late' — I am not otherwise so worried about dad. However a
pronoun cannot be used before a gerund, but only its possessive or object form.
Incorrect: I am worried about he coming home late.
Correct: I am worried about his coming home late.
Correct: My friends saw me smoking.
For more on gerunds see under Verbs.

iQue ideas Page 23


Review Exercise

Underline all the compound nouns in each sentence. Then rewrite the sentences, changing the singular
compounds to plurals. (Ignore the awkwardness of some of the sentences.)

Example:My grandparent had an eight-year-old as a looker-on.

My grandparents had eight-year-olds as lookers-on.

1. The bridesmaid joked with the girlfriend.


2. The eighteen-year-old became the newlywed
3. The brother-in-law gave the place card to the groomsman.
4. Lily-of-the-valley made the centrepiece on the tabletop.
5. They ate ladyfinger, pancake, and gooseberry.

In the following sentences certain words are in boldface. These words could be either Count or Non count
nouns, but you should determine how each is being used in the context of the sentence in which it appears.
Write Count Noun or Non count against each.

6. My relative sent me a pack of Omani dates.


7. This joint is famous for Thai food.
8. The traffic during rush-hour is maddening.
9. Women love jewelry.
10. The army moved the equipment.
11. We need money to buy furniture.
12. Fill out the blank space in the document.
13. We hold these truths to be self evident.
14. Truth will never be known in this case.
15. The Patel sisters dance with grace.

Each boldface word in the sentences below carries an "opportunity for error" in the formation of plurals or
possessives. Underline the errors and rectify them.

16. My friend's, Patel's, had a party last year.


17. It is so far the biggest party in the 2000's.
18. Some of their cousins and the new baby's in the family were present.
19. Mihir and Amisha's family had really grown.
20. Some his childrens have PhD's
21. Mihir and Amisha's Ph. Ds are in psychology.
22. The guest's arrived in busses and taxis.
23. The childrens' menu was interesting.
24. The waitresses could not keep up with the order's.
25. They had a lot of laughes about washing all those dish's.

In the following sentences, fill in the gaps with one of the following: much, many

26. Overwork can cause __________ depression.


27. ________ books are available.
28. How _____________ material are we supposed to read for Verbal?
29. We generally get __________ assignments in quant.
30. I've had _________ headaches already because of stress.

iQue ideas Page 24


Write the plural of the following nouns in the space provided. If the noun cannot be pluralized put 'X' in the
space.

No. Singular Plural


1. Man Men
2. Furniture
3. Video
4. Biology
5. URL
6. Curriculum
7. Goose
8. Knowledge
9. Difficulty
10. Roof
11. Aluminium
12. 1990
13. Dancing
14. Applause
15. Work
16. Memo
17. Criterion
18. Cactus
19. Peace
20. Experience
21. Ski

Answers and Explanations


1. The bridesmaids joked with the girlfriends.
2. The eighteen-year-olds became the newlyweds.
3. The brothers-in-law gave the place cards to the groomsmen
4. Lilies-of-the-valley made the centrepieces on the tabletops.
5. They ate ladyfingers, pancakes. and gooseberries. (ladyfinger means : a small finger-shaped sponge
cake)
6. My relative (Count Noun) sent me a pack of Omani dates. (Non count)
7. This joint (Count Noun) is famous for Thai food (Non count).
8. The traffic (Non count) during rush-hour (Count Noun) is maddening.
9. Women(Count Noun) love jewellery. (Non count)
10. The army (Count Noun) moved the equipment (Non count).
11. We need money(Non count) to buy furniture (Non count)
12. Fill out the blank space (Count Noun) in the document. (Count Noun).
13. We hold these truths (Count Noun) to be self evident.
14. Truth (Non count) will never be known in this case. (Count Noun)
15. The Patel sisters(Count Noun) dance with grace. (Non count)
16. My friends, Patels, had a party last year. (friend's, Patel's are incorrect- no apostrophes required for
these plurals)
17. It is so far the biggest party in the 2000s. (2000's is incorrect. Plural of numbers should be without
apostrophe. However, single digit plural is expressed with an apostrophe. E.g. 3's and 4's. and not 3s and
4s)
18. Some of their cousins and the new babies in the family, were present.[( baby's is incorrect baby - babies
( pl )]

iQue ideas Page 25


19. Mihir and Amisha's family had really grown. (Correct — Mihir and Amisha are husband and %% ire
hence the family is common to them)
20. Some his children have PhDs. (childrens is incorrect children is already plural. PhD's is incorrect
abbreviations do not take apostrophe to show the plural unless written as Ph.D. with periods)
21. Mihir's and Amisha's Ph. Ds are in psychology. (Mihir and Amisha cannot ha% e a common Ph.D.
hcnce both the names should have the apostrophe. Ph.D plural is Ph.Ds. without the apostrophe)
22. The guests arrived in buses and taxis. [guest's is incorrect (guest - guests (pi) busses is incorrect bus
(p1), taxi — taxis or taxies (p1)]
23. The children's menu was interesting. (childrens' is incorrect)
24. The waitresses could not keep up with the orders. (order's is incorrect)
25. They had a lot of laughs about washing all those dish's. [(laughes and dish's are incorrectlaugh —
laughs (pli dish — dishes (p1)]
26. Overwork can cause much depression.
27. Many books are available.
28. How much material are we supposed to read for Verbal?
29. We generally get many assignments in quant.
30. I've had many headaches already because of stress.

No. Singular Plural


1. Man Men
2. Furniture X
3. Video Videos
4. Biology X
5. URL URLs
6. Curriculum Curriclua
7. Goose Geese
8. Knowledge X
9. Difficulty Difficulties
10. Roof Roofs
11. Aluminium X
12. 1990 1990s
13. Dancing X
14. Applause X
15. Work Works
16. Memo Memos
17. Criterion Criteria
18. Cactus Cacti
19. Peace X
20. Experience Experiences
21. Ski Skis

iQue ideas Page 26


4
Determiners
Determiners are the broad classification of words such as articles, quantifiers and certain other similar words
(possessive, demonstrative). They invariably come before a noun or noun phrase.

Study this list :

Articles : A, AN, THE

Quantifiers : ALL, BOTH, MANY, EACH, EVERY, SEVERAL, FEW, A FEW, QUITE A
FEW, ENOUGH, NO, LITTLE, A LITTLE, SOME, A BIT OF, A COUPLE OF, NONI OF,
A GOOD DEAL OF, A LOT OF, LOTS OF PLENTY OF, A LACK OF etc.

Demonstrative : ANY, THAT, THOSE, THIS, SOME, WHATEVER, WHICHEVER etc.

Possessive : MY, YOUR, OUR, THEIR, HER, HIS etc.

Numerals : Numerals are determiners when they appear before a noun: ONE, TWO, THREE
etc. E.g., ONE book Ordinal numerals express sequence: E.g., FIRST impressions,
SECOND chance. THIRD prize. General ordinals express sequence: LAST, LATTER,
NEXT, PREVIOUS, and SUBSEQUENT. E.g.„NEXT month, LAST year, PREVIOUS
class, SUBSEQUENT developments.

After studying the above list try to put a noun after any or all of these words. You will see that their place in
a sentence is essentially before a noun/noun phrase.

[NOTE: Some of demonstrative determiners listed above are also pronouns (MY, YOUR, OUR etc.). Their
function in a sentence makes them either a determiner or a pronoun. When the word can be easily replaced
by a noun it is a determiner, otherwise it is a pronoun.

For example: This film is interesting. In this sentence 'this' is a determiner because it cannot be replaced by
another noun. However, in This is an interesting film, 'this' can be replaced by another noun as, Shofar is an
interesting film. This is a pronoun in the second sentence.]

It is very important to be able to distinguish between a countable and uncountable noun (see the chapter 3)
when you learn the usage of determiners.

Indian mobile phone market OR The Indian mobile phone market?

Articles (THE, A, AN)

Always remember: If the noun is COUNTABLE and SINGULAR it must almost always be preceded by an
ankle (or some other 'determiner').

The noun phrase 'Indian mobile market ' is eminently countable. Hence a determiner is a must before it.

Correct: The Indian mobile phone market has been flourishing since 1995.
Incorrect: Indian mobile phone market has been flourishing since 1995.

The use of articles, however, is not as straight-forward and simple as stated above.

iQue ideas Page 27


 'THE' is not normally used if the noun is plural or uncountable. when things or people are mentioned in
general way.

Correct: People are against the government policy.


Incorrect: The people are against the government policy.

Correct: Traffic is a problem in most cities.


Incorrect: The traffic is a problem in most cities.

The traffic in Mumbai OR Traffic in Mumbai?

Use the article with uncountable nouns when referring to a particular incidence of that noun.

Correct: The traffic in Mumbai is a problem.


Correct: The traffic this morning was heavy.
Incorrect: Traffic this morning was heavy.
Incorrect: Traffic in Mumbai is a problem.

The last sentence may appear correct, because there are nouns that can take an article or no article (zero
article explained later) and make equal sense. We should not accept all nouns as such.

 A job always takes an indefinite article (a/an), as do classification of people or objects into types:

Correct: Parag is an academic.


Correct: My dad is a doctor
Correct: A phone is also a camera.

 Uncountable nouns cannot be used with a/an:

Incorrect: I need an advice.


Correct: I need advice/I need some advice.
Incorrect: Is there a progress without violence?
Correct: Is there progress without violence?

 The is used with things already identified and recognized.

Correct: I bought a pen yesterday. I lost the pen on my way back.


(a pen — first reference — the pen subsequent reference — already identified.)

Earth OR The Earth?


the is used to refer to unique things, - in the world, or in our environment.

Incorrect: Earth revolves around sun


Correct: The Earth revolves around the sun.
Incorrect: Legislature was closed due to bomb scare.
Correct: The Legislature was closed due to bomb scare.

An Orange AND One orange.

iQue ideas Page 28


A/an are usually used with singular, countable nouns. The article is not interchangeable with 'one'. They
have different meanings.

Correct: Have a drink. (Have something to drink — not necessarily one drink)
Correct: Have one drink (not two, or three)

An MBAORA MBA?
Use of the indefinite article an
The use of an depends on pronunciation, not spelling..

Count nouns (singular) beginning with a vowel (/a/, /e/, /i/, /o/, /u/) will take the indefinite article before
them. E.g.. an orange, an apple, an open door, an impossible question etc.

A word (count noun singular) beginning with 'h' often require the indefinite article because the 'h' sound is
not pronounced but the beginning sound is that of a vowel

For example: an hour, an honour, an honest girl, (`h' is not pronounced)

If the 'h' is pronounced, use the indefinite article a.

For example: a history book, a horse, a historian, (`h' is pronounced)

Even if the word begins with a vowel but has a different pronunciation use the indefinite article a.

For example :a useful device, a union matter, a university, a unicorn, a one-film wonder etc. [(because the u
of those words actually sounds like yoo) (as opposed, say, to the u of an umbrella, 'won' film)]

Correct: An NGO (because 'en' is a vowel sound)


Correct: A UFO (because 'yu' is a consonant y sound)
Incorrect: A MBA ( because 'ern' is a vowel sound)
Correct: A URL (because 'yu' is a consonant y sound)
Correct: An URL (If you read the word as "earl")

 There are certain phrases in English which will not allow the article before the noun:

For example: go to school, in hospital, in bed, sent to prison, on foot, by car/bus, from home, at night etc. An
article before the noun will have different meaning.

More on the Use of Articles:

Do NOT use articles with:


 tables, figures, pages, appendices, chapters : e.g., As in figure 5 ...
 a noun with an '-s' possessive : e.g.

Correct: India's Prime Minister,


Incorrect : The India's Prime Minister.

 with days and months, unless you are talking about a particular day or month.
e.g., The test is in November; it will take place on Sunday.

iQue ideas Page 29


Also, The test will take place on a Sunday (on one of the four/five Sundays)
 with television/radio when you are referring to the medium.
E.g. Television is a powerful medium.- - if you use an article, 'television' will mean a television set.
 with titles when name is also used along with the title.

Incorrect: The Prime Minister Modi.


Correct: Prime Minister Modi.

USE the definite article with the following kinds of Proper Nouns:

 Newspapers: The Times of India, The Economic Times, The Time Magazine etc
 Public Institutions: The legislature, The Taj Hotel, The Siddivinayak Temple, The Mahim Church, The
Jurria Masjid, The Golden Temple etc.
 Pluralised Names :The Himalayas, The Vindhyas, The Lakkadives etc. The Birlas, The Ambanis, The
Tatas, The Nehrus etc.
 Geographical places: The Gulf of Mannar, The Sea of Kutch, The Arabian Sea

I climbed the Mount Everest OR I climbed Mount Everest?

Zero Article:

Several kinds of nouns never use articles.

 plural and uncountable nouns referring to things in general:

Correct: I'm afraid of heights.


Incorrect: I'm afraid of the heights.
 before countries, towns, streets, languages and single mountains:

Correct: I've climbed Mount Everest.


Incorrect: I've climbed the Mount Everest.
 In exclamations with what + uncountable noun:

Correct: What sound advice! (advice is uncountable)


Incorrect: What a sound advice!
 with time of day

Correct: We study mostly by night.


Incorrect: We study mostly by the night.

Quantifiers

Quantifiers determine how many or how much of the noun.

See the table below for which quantifiers will work with what kind of nouns.

iQue ideas Page 30


Count Nouns Non count nouns. Count and Non count
Eg. boys Eg. Swimming Nouns.
Eg. boys/swimming
many march all of the
a few a little some
Quantifiers few little most of the
several a bit of enough
a couple of a good deal of a lot of
none of a great deal of lots of
no plenty of
a lack of

I have little money AND I have a little money.

Little means not enough for the purpose.


A little means a small quantity but probably enough for the purpose.
Quite a little means a large quantity.

The above are used with non count nouns and the same way few, a few, quite a few are used with countable
nouns.

Can you please lend me some money? — To this question the different responses will mean the following:

Correct: I have little money (I have no money/ not enough to lend any)
Correct: I have a little money (I have just enough money to lend you the amount you require)
Correct: I have quite a little money (I have a large amount with me and can lend quite a big sum)

(Money is non count noun, though coins and currency notes are countable)

Most students in this college OR Most of the students in this college?

Most of the is used when it is followed by a specific noun (count or non count). Most is used when the
following noun is used with a general plural noun.

Correct: Most of the students in this college are clever.


Incorrect: Most students in this college are clever.
Correct: Most students apply to several colleges.
Incorrect: Most of the students apply to several colleges.
Correct: Most of the students in this school apply to several colleges.

Many a student AND Many students.

Correct: Many a student has asked this question.


Correct: Many students have asked this question.

Both the above are correct constructions (though the first is considered somewhat 'literary') and mean the
same. Note that many a + singular takes a singular verb, and many + plural takes a plural verb.

iQue ideas Page 31


Review Exercise

Correct the following sentences by deleting or supplying the articles (a, an, the).

Example:

Question: I don't really like the sweets but I love an ice-cream. I have my lunch in cafeteria.

Answer: I don't really like the sweets but I love an ice-cream. I have my lunch in the cafeteria.

1. She likes the fashion. She plays tennis and likes travelling. She likes watching the TV and going to
cinema. She hates the noise and driving the cars.
2. He loves snow-capped mountains; they fill him with a happiness. He dislikes the arguments and the
fights and sight of the blood. He loves the peace and the humanity.
3. I love swimming. I love getting wet in rain. I hate snobbish people. 1 hate pollution. I like the tests. I
love taking the grammar tests.
4. My family are most important people for me. I love my country. I hate the racism because skin colour is
not important. I hate the war and the violence. I think that the dialogue is better than the violence.
5. They usually go to Maldives for holiday, but last year they went to see Alps. They skied on the Mont
Blanc. He's from Andaman Islands and she's from the Sri Lanka. She bought pair of jeans and a most
expensive watch I've ever seen.

Fill in the blank with the appropriate article (a, an, the). Leave it blank for zero article.
6. I met my wife at ______ intercollegiate competition.
7. She was from _________ Stephen's College.
8. She really likes _____ classical music. but I prefer ______ pop.
9. I like ________ mountaineering, but she is terrified of _________ heights.
10. Despite our differences, we have _______ romantic tale to tell our children.

Choose the correct determiner in the following sentences:


11. I saw a/an/the meteorite yesterday.
12. Don't look directly at a/an/the sun
13. Is there any milk left in a/an/the refrigerator?
14. I have a/an/the glass of orange juice for breakfast.
15. The children were each/every given a/an/the apple.
16. The mother spoke to each/any/every teacher.
17. She wore a ring on each/any/every finger.
18. She met a/an/the Principal without some/any/every problem.
19. She always kept any/every/some money with her for emergencies.
20. He was having a great deal of/ much of/a lot of/ plenty of trouble passing his exams.
21. A lot of / much of / plenty / a little/ a great deal of the papers were difficult.
22. A lot of / much of / plenty / a little/ a majority of information was false.
23. The exam is close, yet there is some/plenty/several/many/little time to study.
24. Although there are a great deal of/ a majority of/ a lot of / a little/ several of/ brilliant students, a few/
many/ a majority of/ a lot of / a little will qualify.
25. They are able to clear several of/ some/plenty/ a great deal of/ much of the papers, but many/ a little/
a few/ a lot of/ several of remain difficult.
26. Many/several/a few/a majority of/ several of/ a little a student has been discouraged.

iQue ideas Page 32


27. Although a great deal/ a few/ a little/ some/ enough/ of the garden is open to the sun, there are a great
deal/ a few/ a lot of/ some of / enough/ trees to make it shady and cool.
28. She ate much of/ a few/ plenty/ a lot of food last night.
29. Many of/ Several of/ Much of /A majority of/ plenty the evidence was lost.
30. Much/A lot/ Many/ A little/ A great deal students are happy.

Answer & Explanations

1. She likes the fashion. She plays tennis and likes travelling. She likes watching the TV and going to the
cinema. She hates the noise and driving the cars.
2. He loves snow-capped mountains; they fill him with a happiness. He dislikes the arguments and the
fights and the sight of the blood. He loves the peace and the humanity.
3. I love swimming. I love getting wet in the rain. I hate snobbish people. I hate pollution. I like the tests. I
love taking the grammar tests.
4. My family are the most important people for me. I love my country. I hate the racism because the skin
colour is not important. I hate the war and the violence. I think that the dialogue is better than the
violence.
5. They usually go to the Maldives for a holiday, but last year they went to see the Alps. They skied on the
Mont Blanc. He's from the Andaman Islands and she's from Sri Lanka. She bought a pair of jeans and
the most expensive watch I've ever seen.
6. I met my wife at an intercollegiate competition.
7. She was from the Stephen's College. (Public Institutions will take the)
8. She really likes (zero article) classical music, but I prefer (zero article) pop.
9. I like (zero article) mountaineering, but she is terrified of the heights.
10. Despite our differences, we have a romantic tale to tell our children.
11. I saw a meteorite yesterday.
12. Don't look directly at the sun.
13. Is there any milk left in the refrigerator?
14. I have a glass of orange juice for breakfast.
15. The children were each given an apple.
16. The mother spoke separately to each teacher. (every is incorrect because it is distributive- 'separately'
prevents the use of a distributive determiner)
17. She wore a ring on every finger.
18. She met the Principal without any problem.
19. She always kept some money with her for emergencies.
20. He was having a lot of trouble passing his exams.
21. A lot of the papers were difficult.
22. A lot of information was false.
23. The exam is close, yet there is some time to study.
24. Although there are a lot of brilliant students, a few will qualify.
25. They are able to clear several of the sections, but a few remain difficult.
26. Many a student has been discouraged.
27. Although some of the garden is open to the sun, there are a lot of trees to make it shady and cool.
28. She ate a lot of food last night.
29. Much of the evidence was lost.
30. Many students are happy.

iQue ideas Page 33


5
Pronouns
Pronouns generally replace nouns. The noun that a pronoun replaces is called its antecedent. A pronoun must
have a clear antecedent to avoid ambiguity. However, some pronouns can start a sentence and may have no
clear antecedent.

For example:
Sheetal is a good girl; she does not hurt anyone. The antecedent of she is Sheetal.
Everyone in this class is appearing for the CAT. The pronoun everyone has no antecedent.

There are different kinds of pronouns. They are:

Personal Pronouns : I, WE, YOU, HE, SHE, IT, THEY.


Demonstrative Pronouns: THIS, THAT, THESE, THOSE, SUCH.
Indefinite Pronouns : EVERYBODY, ANYBODY, SOMEBODY, ALL, EACH, EVERY,
SOME, NONE, ONE.
Relative Pronouns : WHO, WHOEVER, WHICH, THAT.
Reflexive Pronouns : MYSELF, YOURSELF, HERSELF, OURSELVES, THEMSELVES.
Intensive Pronouns : MYSELF, YOURSELF, HERSELF, OURSELVES, THEMSELVES.
Interrogative Pronouns : WHO, WHICH, WHAT.
Reciprocal Pronouns : EACH OTHER, ONE ANOTHER.

Notice that the Intensive and the Reflexive pronouns have the same form, though their functions differ.

You do not have to learn the list by heart. It will enable you to recognize a pronoun (whatever kind) when
you see them in the questions.

Though pronouns are also nouns, unlike nouns (most) pronouns change form creating problems in their
usage. Nouns only add an "s" to become plural (Cat — Cats) or an apostrophe + sto show the possessive (Cat
's tail).

But I is used as the subject of a sentence (/ am glad.), me is used as an object in various ways (Help me, She
gave me a smile, Solve this problem for me), and my is used as the possessive form (That's my problem).

The table below gives the complete list of personal pronouns:

Pronouns
Person and Subject Object Possessive Possessive Possessive
number Pronouns Pronouns Adjectives Pronouns Pronouns
First Person
I ME MY MINE MYSELF
Singular
First Person
WE US OUR OURS OURSELVES
Plural
Second Person
YOU YOU YOUR YOURS YOURSELF
Singular
Second Person
YOU YOU YOUR YOURS YOURSELVES
Plural

iQue ideas Page 34


HE HIM HIS HIS HIMSELF
Third Person
SHE HER HER HERS HERSELF
Singular
IT IT ITS ITS ITSELF
Third Person
THEY THEM THEIR THEIRS THEMSELVES
Plural

He is taller than I OR He is taller than me.

Incorrect: He is taller than me.


Correct: He is taller than I.

Object pronoun (me. us etc.) is used in two contexts:


o As the object of a verb.
o As the object of a preposition.

As the object of a transitive verb (a verb that requires something else to complete its meaning):
He loves her.

(He loves what? -- We can say: He loves money (noun). The verb loves needs an object to complete its
meaning. Money then is the object of the verb loves. If a pronoun is the object, use the objective case –her/
them etc.)

As the object of a preposition (prepositions Generally require something else to complete their meaning):
He is crazy after her.
He is crazy after what? — the preposition 'after' requires an object to complete the meaning of the sentence.
We can say: He is crazy after money (noun). Money is then the object of the preposition afier. If you use a
pronoun instead of a noun, the object form is called for.

Correct: Aisharya is taller than Amir.


Incorrect: She is taller than him.
Correct: she is taller than he (is).

In the first sentence you have two nouns as the object and the subject. In the second sentence you have two
pronouns. The first one lie is the subject, the second is neither the object of any verb, nor is there a
preposition in the sentence. The use of the object case cannot be justified. In fact, the second pronoun he is
the subject of the abbreviated clause he is that follows the conjunction than.

I love her more than then OR I love her more than them?

Try to analyze the meaning of these sentences. / love her more than they suggests that / love her more than
them love her. And the second sentence / love her more than them suggests that Hove her more than I love
them. Hence both are correct.

When you have two nouns (pronouns) is a sentence in comparison we can safely follow the words than and
as with either the subject or the object pronouns.

Correct: I love her as much as they.


Correct: I love her as much as them.

iQue ideas Page 35


As you can see. the meaning will change depending on the pronoun you choose (subject or object). Both the
above sentences are correct: only they mean different things.

‘It is I’ OR ‘It is me’?


The words that follow the different form of the verb be are called its complement.

It is a pleasant morning. (a pleasant morning is the complement of the verb is.)

In other words, verbs — he, being, am, is, was, are, were( forms of 'be') are followed by a complement and
not an object. Hence, an object pronoun is a misuse after such verbs.

Correct: It is I
Incorrect: It is me.

You will get better clarity on this if you try to continue this sentence, like, 'It is I who came first' ... or It is
me who came first... ' etc. You will probably see that it is ridiculous to say 'me who...'

Subject and Object Pronouns — other issues.

When a personal pronoun is connected by a conjunction to another noun or pronoun. its case does not
change. We would write
I am taking a course in C++
Priya and I are taking a course in C++( not Priya and me)
She and I are taking a course in C++.

The same is true when the object pronoun is used. :


Professor Srinivas gave all his books to Priya
Professor Srinivas gave all his books to Priya and me.

When a pronoun and a noun are combined (which will happen with the plural first- and second-person
pronouns), choose the case of the pronoun that would be appropriate if the noun were not there.

We students are against the fee hike. .


The administration has put us students in a bad situation.

With the second person, we don't really have a problem because the subject form is the same as the object
form, ‘you’:

You students do not realize that education is costly.


We expect you students to appreciate the situation

'My' AND 'Mine'


mine, yours, ours, theirs are called nominative possessive pronouns.
This house is yours.
Theirs is really dirty.
Ours is beautiful
This new house is mine.

iQue ideas Page 36


The difference between the two types of possessives (refer to the table) e.g. my and mine is that one my will
always come bore the noun (as in my pen) and mine will always follow the noun (The pen is mine).
This is my party.
This bill is mine.
Words like, my, our ..., etc. also qualify nouns. Therefore, besides being pronouns they are also termed as
possessive adjectives or pronominal adjectives.
The difference between Possessive adjective and a Possessive pronoun can be easily understood in the above
two sentences.

'This' AND 'That'

this/ that/ these/ those/ such are called demonstrative pronouns. They can also function as determiners.
As pronouns, they identify or point to nouns.
That is beautiful.
I will never forget this.
Such is my belief.

When used as subjects, the demonstratives, they can refer to objects as well as persons.
This is my mom.
Those are my cousins.

‘Who', 'Whoever', 'Which', and 'That'

These are relative pronouns. They relate descriptions to a noun or pronoun.


The girl who lives next door is an IIM alumna.

The word who 'connects or relates' the subject, the girl, to the verb (lives). That is why they are called
relative pronouns.
Choosing correctly between which and that and between who and whomis one of the most frequently ask
questions in grammar.

Generally, which introduces a description that can be removed from the sentence without causing too much
harm to the meaning of the sentence. In other words the description is parenthetical in nature and is generally
set off with a comma or surrounded by commas.
That introduces a description essential for the identification of the preceding noun and cannot be set off or
surrounded by commas.
The pronoun whichis used to describe everything except people. (If used to describe a person it is incorrect)

Incorrect: The man which I was with is my uncle.


Correct: Than man whom I was with is my uncle.

Who (and its forms) refers to people;that usually refers to things, but it can also refer to people.

Correct: The man that I was with is my uncle.

For more on this see the section below.

'who', 'which', OR 'that'?

iQue ideas Page 37


Who refers to people. Which refers to everything other than people (including animals). That generally refers
to things other than people, but can be used to refer to people as well — in other words that can be used in
place of which and who.

This is the professor who teaches us mathematics.


She is also a consultant with TCS, which is famous for its corporate ethics.

That introduces essential clauses while which introduces nonessential clauses.

I do not like Hindi movies that are copies of English movies.

Without the that clause one may misunderstand that I do not like any Hindi movies. It would be gross
injustice to say that the speaker does not like Hindi movies. He dislikes only the copies. 'That' clauses are
hence essential clauses. (restrictive clauses)

To understand the difference between which and that, read the above sentence with which and work out the
meaning.

I do not like Hindi movies, which are copies of English movies.

That the speaker does not like any Hindi movies at all is what the sentence suggests. It is as if the reason for
his dislike is that they are copies of English movies. Besides, 'which ' clause can also be separated from the
other clause by a comma. The comma is not possible with the 'that' clause.

If that has already been used in the sentence, use which to introduce the essential clause that follows.

Incorrect: That is a decision that you must live with for the rest of your life.
Correct: That is a decision which you must live with for the rest of your life.

If the essential clause starts with this, that, these, or those, use which to connect.

Incorrect: Those decisions that you made in your childhood are the decisions of a child anyway.
Correct: Those decisions which you made in your childhood are the decisions of a child anyway.
Better: The decisions you made in your childhood are the decisions of a child anyway.

Choose the 'Better' sentence for your answer if forced to choose between the two.

'Who' AND 'Whom'


'Whoever' AND 'Whomever'
Who and Whoever are the subject (relative) pronouns and Whom and Whomever are the object pronouns.
The difference between who and whoever (by extension whom and whomever) is that who is (usually)
definite and whoever is indefinite in sense and means whatever person – no matter who.

Correct: My friend, who is a banker, is honest.


Incorrect: My friend, whoever is a banker, is honest.

Correct: Sell it to who has the money.


Better: Sell it to whoever has the money

iQue ideas Page 38


whoever, whomever, whichever, whatever — are known as indefinite relative pronouns. Read these
examples to understand why they are called indefinite.

Correct: The IIMs select whomever they like.


Correct: Take whatever you want.
Correct: Whoever solves this riddle gets a prize.

To choose between who and whom (whoever and whomever), re-phrase the sentence so you choose between
he and him. If the sentence makes sense with him, write whom; if it makes sense with he, write who.

Whom did you meet at the Seminar? (Did you meet him at the Seminar?)
Give the money to whomever you please. (Give the prize to him.)

But a peculiar problem arises when the rephrasing can be done in both ways.

The IIMs issue calls to whomever/whoever has cleared the CAT.

Important: Use the ever suffix when who or whom can fit into two clauses in the sentence.
Rephrasing it with he/him, you get

The IIMs issue calls to him. He has cleared the CAT.

Because we can substitute 'him' and 'he' in both the clauses, we must use the 'ever' suffix. Now, to determine
whether to use whoever or whomever, follow this rule: him + he = whoever; him + him = whomever

Therefore,
Correct: The IIMs issue calls to whoever has cleared the CAT.
Incorrect: The IIMs issue calls to whomever has cleared the CAT.

IIMs will select whoever/whomever the panel recommends.

Rephrasing you get,


IIMs will hire him. The panel recommends him.
Correct: IIMs will select whomever the panel recommends.
Incorrect: IIMs will select whoever the panel recommends.

`Everyone' OR 'Everyone'?
The indefinite pronouns everybody, anybody, somebody, all, each, every, some, none, oneare indefinite and
can have no antecedents.
Except for all, some and none all these pronouns will take a singular verb.

Correct: Everyone is eagerly awaiting the CAT.

Though 'everyone' looks plural, a plural verb creates an error with ‘every’. (Everybody is present.) Think of
it as ‘every single one’.
When you use ‘everyone’ it means ‘everybody’. When you use ‘every one…’ it is followed b of ‘Every one
of the boys…’

'None has qualified' OR 'None have qualified'?

iQue ideas Page 39


The indefinite pronoun none can be either singular or plural, depending on its context. Generally follow up
'none' with a plural verb unless something else in the sentence prevents the use of the plural verb, as in 'None
of the food is fresh.'

Correct: None of you claim responsibility for this incident?


Incorrect: None of the students has done their homework.
Correct: None of the students have done their homework.
Incorrect: None of the luggage have reached us.
Correct None of the luggage has reached us. (` luggage' determines the number.)
Correct: None of the food is fresh ( food determines the number)

Some
Some can be singular or plural depending on whether it refers to something countable or non-countable.

Correct: Some students were not present


Correct: Some money has been lost.

With some what follows the of will decide whether the verb is singular or plural.
Correct: Some of the students were absent. (students is countable)
Correct: Some of the sheen is lost. (sheen is not countable)

‘Anyone' or 'Any one'?


Compare 'everyone' and 'every one'.

The usage issue related to 'Anyone' and 'any one' is different from that of 'everyone' and 'every one of..'
though they have a lot in common. First, any can take either a singular or plural verb depending on how it is
construed:

Correct: Any of these boys is good enough for the leader's post.
Correct: Are any of these boys good enough for the leader's post? (here the meaning is 'are some of these
boys')

Anyone like 'everyone' is always singular. Anyone and anybody are singular terms and always take a
singular verb.

The one word anyone means any individual. Or anybody.


Anyone may enter the class (means any person can enter the class)
Any one may enter the class (means any one person only may enter the class)

When followed by of as it should be, after separating any and one see the difference in meaning. Now, study
these sentences for better clarity.

Correct: Any one of the girls (not anyone) could have reported the matter.
Correct: Have any of the girls (some) reported the matter?
Correct: Any one of the Test Series is enough.
Correct: Are any seats available?

‘Myself’

iQue ideas Page 40


The self-pronouns myself yourself herself, ourselves, themselves have two names: Emphatic (intensive)
Pronouns and Reflexive Pronouns. Though the form is the same it gets different names because the functions
are different. The emphatic or intensive pronoun is used to emphasize the noun.

Did you copy paste this project? "No. I did it myself'


Reflexive Pronouns are used to indicate that the subject also receives the action of the verb. In other words
the action in the verb reflects to the subject.

Correct: Students who copy-paste their projects are fooling themselves.


Correct: You can congratulate yourself
Correct: She promised herself not to copy-paste the project.

What this means is that whenever there is a reflexive pronoun in a sentence there must be a person to whom
that pronoun can 'reflect'. In other words, if the reflexive pronoun has nothing to reflect to that sentence will
be incorrect. In the above sentences the italicized words show this relationship. The action in the verb
fooling is received by or reflected to the subject students.

Incorrect: Please hand that book to myself (there is no "I" in that sentence for the "myself' to reflect to).
Incorrect: Manish and myself are responsible for this decision.
Correct: Manish and I are responsible for this decision.
Incorrect: These decisions will be made by myself
Correct: These decisions will be made by me.

When pronouns are combined, the reflexive will take the first person:

Incorrect: Parag, you, and I are deceiving themselves.


Correct: Parag, you, and I are deceiving ourselves.

When there is no first person, the reflexive will take the second person:

Correct: You and Manish have deceived themselves.


Correct: You and Manish have deceived yourselves.

‘Which' AND 'What'


The interrogative pronouns who, which, what ask questions.

What is that?
Who will help me?
Which do you prefer?

Which is more specific than what in implication. In a test, "Which questions are easy?" means specific
questions on that test (questions 10, 15 and 24 etc.) "What questions are easy for you?"means what kind of
questions (algebra, geometry, numbers etc.) is easy for you.

‘Each other' AND 'One another'


The reciprocal pronouns are each other and one another.

My friend and I gift each other frequently. .

iQue ideas Page 41


If more than two people are involved we would use one another.

Six of us gift one another frequently.

However. Six of us gift each other frequently is also correct because the exchange happens between any two
members at one time and among six members. In other words, just like between and among, each other can
accommodate more than two, but one another cannot be used for only two people. We all borrow each
other's ideas..

One
One is an impersonal, generic, representative pronoun. Hence attributing gender or specificity to one later on
in a sentence will be an error.

Incorrect: If one fails several times he is likely to get discouraged.


Correct: If one fails several times one is likely to get discouraged.
Incorrect: One must be clear about his or her career goal.
Correct: One must be clear about one's career goal.
Incorrect: One must not lose control of himself or herself in a GD.
Correct: One must not lose control of oneself in a GD.

Also note:
One in the subject will always require a singular verb:

More than one student is confused


One of the girls is crying.
One in four boys is in love with her.

Its AND It's.


Unlike nouns, pronouns do not need an apostrophe+s' to show the possessive. Mine, Yours, His, Hers, Its,
Ours, Theirs are possessives. The confusion is mainly with it's and its. It's is a contraction of 'it is' or 'it has'.

Correct: It's a holiday tomorrow. (It is a holiday .....)


Correct: A snake can change its skin.

Everyone loves their mother OR Everyone loves his/her mother.


The inconsistency in the number (singular/plural) of pronoun use arises mostly when students use the
possessive form.

Incorrect: Not one of the passengers offered their help to the injured woman
Correct: None of the passengers offered their help to the injured woman OR
Correct: Not one of the passengers offered help to the injured woman.

Incorrect :In the beginning everyone has problems setting up their personal computer.
Correct: In the beginning everyone has problems setting up his/her personal computer.

The use of 'their' as a singular, gender neutral pronoun is a debated area in English grammar. In the grammar
questions in competitive examinations, do not use their to refer to a singular pronoun. ["It's enough to drive
anyone out of their senses" — G.B. Shaw.]

iQue ideas Page 42


Let us go AND Let you and me go.
Let us go is an imperative in which the subject is an object pronoun. When we separate the `us' into its
constituents, we get you and me. But since these are subjects in this sentence we may want to transform it
into let you and I go. In a long sentence, this looks much more plausible. Let you and I decide on this issue
of compensation. But, you have to resist the temptation to use the subject form. Let us go expands to Let you
and me go. And Let you and me decide on this issue of compensation is correct.

To recapitulate:

Correct: Let us work together.


Correct: Let us decide on this issue.
Incorrect: Let you and I go together.
Incorrect: Let we work together.
Correct: Let you and me decide on this issue.
Correct: Let them decide on this issue.
Correct: Let him and her decide on this issue.

Review Exercise
Strike out the inappropriate word/s.
Example: She is not hesitant about giving her /herself/ himself/ oneself/ himself or herself credit for it.
Answer: She is not hesitant about giving her/ herself/ himself/ oneself/ himself or herself credit for it.
1. The invitations are a matter for her and him / she and he / she and him to decide.
2. Everyone in the family is eagerly awaiting their/ his or her invitation.
3. Neither the wife nor the husband is/ are ready with his /her /their / her or his list of invitees.
4. No one in this class seems to know his / his or her /their way around Mumbai well.
5. Amitabh is definitely taller than we / us.
6. I thought of sharing the room between my friend and I / me/ myself
7. If it were for my sister and I/ me, we would have bought a new car.
8. The principal was apprehensive that we/us students would oppose the fee hike.
9. Parag did not want anyone but she / her/ herself writing the explanations.
10. 'Hamlet' that / which was written in the sixteenth century is still being studied by students of literature.
11. He has watched all the movies which / that/ that or which were made by Bergman.
12. She wanted to buy a top that / which would complement her new jeans.
13. The answer key, that / which / who accompanies the test are often incorrect.
14. Every week Parag and we / us get together to review our preparation.
15. Most of the students were wearing his or her / theirblazers.
16. The person whom / whose/ who house they visited was an old friend of theirs.
17. The students whom / whose/ who designed the machine received an award.
18. Everyone prepared himself or herself! themselves well for the CAT.
19. The college is running out of money, but that's between you and me / I.
20. I did not know who was writing, but it might be she/ her/ herself.
21. Parag and they / them/ themselves can never see eye to eye.
22. We knew that Bhavin and he / him/ them would be great teachers.
23. The crowd shouted it's / their/ his or her/its approval when MSD hit a six.
24. The drivers of the car, my sister and he / him paid for the damage.
25. I really don't like him / his/ he walking late into my lecture.
26. I can handle stress better than she / her/ herself.
27. She likes TV shows which / that make her laugh.
28. Parag, Bhavin and she / her conduct lectures there.

iQue ideas Page 43


29. We knew also that Manish is as smart as she / her/ herself.
30. Parag isn't very happy about us / our/ we arriving late for the lectures.
31. The team attended the party along with its / their / it's relatives.
32. The professors seem not to care much about us / we students.
33. Who else could be writing other than she / her/ herself?
34. She was not sure about who / whom to invite for her wedding anniversary.
35. We / Us students must learn to study regularly.
36. No one has come for the class except for you and I/ me/ ourselves.
37. Among the students were several prospects who / that/ which were regarded for next year's CAT.
38. Students who / that/ which scored the highest had also spent the greatest efforts.
39. Neither of those engineering students knew what their /his or her scores would be.
40. The Teacher would not tell us whom / whose/ who answer was correct.
41. Its/It's likely that the person whom / whose/ who you consulted was a quack.
42. Dad made my sister and I/ me pay for the damage to the car.
43. Either all the rooms or the hall must have it's / Its / their walls repainted.
44. She writes as well as me / I/ myself.
45. The only girls present were Rachana and her / she/ herself
46. Every article in this house has been utilized to it's/ its /their fullest.
47. What / Which novel of Grisham have you read recently?
48. Rahul said that we / us students need to register for CAT before 5th September.
49. The teacher found no evidence of malpractice in the exam, that / which / who surprised no one.
50. People who / that/ which / who or that live in glass houses shouldn't throw stones.
51. No one but him / he shall win her heart.

Answers & Explanations

1. The invitations are a matter for her and him to decide.


2. Everyone in the family is eagerly awaiting his or her invitation.
3. Neither the wife nor the husband is ready with her or his list of invitees.
4. No one in this class seems to know his or her way around Mumbai well.
5. Amitabh is definitely taller than we.
6. I thought of sharing the room between my friend and me. (between is a preposition. The first object of
between is ' my friend' and the second object is me. Myself is not required.
7. If it were for my sister and me, we would have bought a new car.
8. The principal was apprehensive that we students would oppose the fee hike.
9. Parag did not want anyone but her writing the explanations.(but is preposition in this sentence).
10. 'Hamlet', which was written in the sixteenth century, is still being studied by students of literature. (Non
essential clause, hence which is correct. That may suggest there are Hamlets written in other centuries)
11. He has watched all the movies that or which were made by Bergman. (that is appropriate because it is
an essential clause. However, since Bergman is mentioned which would not be incorrect.)
12. She wanted to buy a top that would complement her new jeans. (which would not be incorrect)
13. The answer key that accompanies the test is often incorrect.(essential information, hence that is
appropriate. Which would not be incorrect).
14. Every week Parag and we get together to review our preparation.
15. Most of the students were wearing their blazers.
16. The person whose house they visited was an old friend of theirs.
17. The students who designed the machine received an award.
18. Everyone prepared himself or herself well for the CAT.
19. The college is running out of money, but that's between you and me.

iQue ideas Page 44


20. I did not know who was writing, but it might be she.
21. Parag and they can never see eye to eye.
22. We knew that Bhavin and he would be great teachers.
23. The crowd shouted its approval when MSD hit a six.
24. The drivers of the car, my sister and he paid for the damage.
25. I really don't like his walking late into my lecture.
26. I can handle stress better than she.
27. She likes TV shows that make her laugh.
28. Parag, Bhavin and she conduct lectures there.
29. We knew also that Manish is as smart as she.
30. Parag isn't very happy about our arriving late for the lectures.
31. The team attended the party along with their relatives.
32. The professors seem not to care much about us students.
33. Who else could be writing other than she?
34. She was not sure about whom to invite for her wedding anniversary.
35. We students must learn to study regularly.
36. No one has come for the class except for you and me.
37. Among the students were several prospects who were regarded for next year's CAT.
38. Students who scored the highest had also spent the greatest efforts.
39. Neither of those engineering students knew what his or her scores would be.
40. The Teacher would not tell us whose answer was correct.
41. It's likely that the person whom you consulted was a quack.
42. Dad made my sister and me pay for the damage to the car.
43. Either all the rooms or the hall must have its walls repainted.
44. She writes as well as I.
45. The only girls present were Rachana and she.
46. Every article in this house has been utilized to its fullest.
47. Which novel of Grisham have you read recently?
48. Rahul said that we students need to register for CAT before 5th September.
49. The teacher found no evidence of malpractice in the exam, which surprised no one.
50. People who or that live in glass houses shouldn't throw stones.
51. No one but he shall win her the princess's heart. (but is conjunction here — No one shall win but he
shall.)

iQue ideas Page 45


6
Adjectives and Adverbs
Adjectives AND Adverbs are words that describe or modify something else in the sentence. Hence they are
called modifiers.

An adjective modifies a noun — cannot do any other work, and occurs almost always before the noun. it can
occur after the noun as a modifier with certain pronouns:

For Example: the beautiful car (adjective before the noun).


somethinghorribie (adjective after the pronoun).

An Adverb can modify a verb, an adjective or another adverb. And can occur almost anywhere in the
sentence (discussed later).

For example: She talks softly (softly modifies the verb talks)
She is very loud. (very modifies the adjective loud.)
She drives very slowly (very modifies the adverb slowly)

Adverbs often describe when, where, why, or thecircumstances of something.

Though adverbs generally have an `ly' ending (softly, slowly)many words and phrases not ending in -ly
serve an adverbial function (fast, well) .Also, there are `ly' words that are not adverbs. The words lovely,
lonely, motherly, friendly, neighbourly, are adjectives:

For example: the lonely man (lonely is an adjective modifying the noun man)

ADJECTIVES

Stupider OR more stupid?


Commonest OR the most common?

Comparison of adjectives

Adjectives and adverbs can show degrees of modification.

E.g., Amit is rich. Parag is richer, Sanjaya is the richest.

The degrees of comparison are known as the positive (the adjective itself), the comparative, and the
superlative.

[In comparative degree, comparison is made between two things only. And we require at least three things to
correctly use the superlative degree of comparison. Notice also that the word than frequently accompanies
the comparative (taller than) and the word the precedes the superlative (the tallest)]

When you form the comparative and the superlative remember this:

iQue ideas Page 46


Adjective + ether or est (quicker/quickest) is used if the adjective has only one syllable.

More/Most + adjective, as well as Adjective + ether or est (stupider/stupidest as well as more stupid/most
stupid). is used for adjectives with two syllables.

More/Most + adjective, is used for adjectives (more/most beautiful) with more than two syllables.

If you are in doubt about the number of syllables in a word, look it up in a dictionary and notice how it is
broken down: you can easily see the number of syllables.
E.g. com-pli-men-ta-ry — means the word complimentary has five syllables.

Correct: He is stupider than I thought. (alsomore stupid)


Correct: He is the handsomest man on earth. (alsothe most handsome)
Correct: This habit is commoner in India than in Pakistan. (alsomore common)

The true Southern watermelon is a boon apart, and not to be mentioned with commoner things. Pudd'n'head
Wilson by Mark Twain.

Positive Comparative Superlative(the +)


Cute Cuter Cutest One syllable
Lucky Luckier/more lucky Most Lucky/Luckiest Two syllable*
Beautiful More beautiful Most beautiful More than two syllables

* Adjectives that have two syllables and end in y (happy), ow (shallow, narrow), and le (gentle, able), can
take -er and -est. There are, however, two syllable words that cannot take —er and —est (e.g., complex)

More worse OR worse?


Less OR lesser

Irregular Comparative forms.

Certain adjectives have irregular forms in the comparative and superlative degrees:

Adjective (Positive Degree) Comparative Degree Superlative degree


Good Better Best
Bad Worse Worst
Little Less/Lesser/Littler Least/Littlest
Much More Most
Many More Most
Far Further or Farther Furthest or Farthest

The table shows that 'worse' and 'better' are the comparative degrees of bad and good respectively. Double
comparisons like 'more worse' or 'more better' are hence redundant and incorrect.

Incorrect: Things can't become more worse than this.


Correct: Things can't become worse than this.

Incorrect: Whose policies are more worse?


Incorrect: It just gets more worse.
Incorrect: This car is no more worse than the other one.

iQue ideas Page 47


Correct: Whose policies are worse?
Correct: It just gets worse.
Correct: This car is no worse than the other one.

The same way, doublesuperlatives like, the most happiest, or the most heaviest is also incorrect.

Less AND Lesser

Less and lesser are often synonymous. However, Less suggests amount; lesser suggests degree

Lessis also used in the non-comparative sense (that is, the positive degree) in sentences like, "Not that I
loved Caesar the less."
This use is different form the common expressions like, the lesser of two evils, or I have less money than he
has.

Correct: She has less money.


Incorrect: This is less of the two evils.
Correct: This is the lesser of the two evils.
Correct: We have less money than they have.
Correct: We have lesser (amount of) money than they have.

Correct: We are less studious than they are.


Incorrect: We are lesser studious than they are.

Notes:
 less can be used in the comparative form with an adjective, while lesseras a comparative form is used
with a noun.
 lesser is as an adjective is preceded by an article (the lesser evil, a lesser person)
 less can be an adjective (less money, less time)
 less can be an adverb ( less pretty, it costs less)

She eats too less OR She eats too little?


Incorrect: She eats too less.
Informal: She eats too little.
Correct: She eats little or She eats very little.

The problem here is with the word 'too' which cannot take a comparative after that. It is not correct to say 'he
acts too smarter'. He acts too smart or she is too beautiful may be acceptable in informal use, though He acts
smart/very smart or She is beautiful/very beautiful or even 'too smart to be selected' or too beautiful to lose
are better expressions.

The same way, though she eats too little may be acceptable in informal use 'she eats little/very, little' or even
too little to survive are better expressions.

An ideal choice OR A more ideal choice?

Although most adjectives and adverbs can have comparative and superlative forms, there is a class of
adjectives and adverbs in English that express an extreme of comparison or an extreme state. Trying to
intensify these words or trying to form a comparative of these words will create errors in usage.

iQue ideas Page 48


Incorrect: The Kayans of Borneo developed a more unique culture.
Incorrect: The Kayans of Borneo developed a very unique culture.
Correct: The Kayans of Borneo developed a unique culture.

Incorrect: Cherai lake is a more ideal choice for budget holidays.


Correct: Cherai lake is the ideal choice for budget holidays.

Similarly, there is a class of words which, strictly speaking, are not open to comparison or intensification,
but lend themselves to the context for comparison and intensification to produce a ‘more profound’ effect.

Correct: He believes that meditation has helped him lead a .fuller life.
Correct: His project is morenearly complete.

Correct: His explanation seemed less adequate to his audience.


Correct: Her explanation was more precise.

In all the above sentences the italicised comparatives are made correct only by the context.

Given below is a list of more such words that are not generally open to comparison:

absolute, impossible, principal, adequate, inevitable, stationary, chief, complete, irrevocable sufficient, main,
unanimous, devoid, manifest, unavoidable, entire, minor unbroken, fatal, paramount, unique, final, perpetual,
universal, ideal, preferable, whole etc.

Notes:
Both adverbs and adjectives in their comparative and superlative forms can be accompanied by words that
intensify the degree. (much, very much, etc.)

Correct: The students were a lot more careful in this test.


Correct: And they did so much better.
Correct: Their percentiles have been somewhat better.
Incorrect: The old man is much more smarter than the young man. (much smarter)

If the intensifier very accompanies the superlative, a determiner is also required:

Incorrect: Rolex is very finest in watches.


Correct: Rolex is the very finest in watches (the — determiner)
Correct: They're trying their very best. (their — determiner)

More than OR Over?

The use of over and more than in numerical expressions of age, time etc. is correct.

All the following are correct sentences.

The wall is more than ten feet high.


The wall is over ten feet high.
She is over forty.
She is more than forty.

iQue ideas Page 49


The presentation lasted for more than two hours.
The presentation lasted for over two hours.

Commas with adjectives:


When there are more than two adjectives modifying a noun use a comma to separate them all with commas.

Incorrect: He is a tall, dark and handsome man.


Correct: He is tall, dark, and handsome man.

When there two adjectives check if a conjunction (and or but) can be inserted between the two adjectives. If
you can, put a comma between the two adjectives.

Incorrect: He is a learned wise man


Correct He is learned, wise man.

We can say learned and wise man; hence, place a comma between the two adjectives.

If it is not possible to insert a conjunction, do not use the comma.


Incorrect: This is a pretty, old trick.
Correct: This is a pretty old trick.

We cannot say pretty and old trick; hence, do not use comma.

The rich has to help the poor OR The rich have to help the poor?

Collective Adjectives and Collective Nouns.

In combination with a definite article an adjective often becomes a noun referring to a group of people.

the poor, the rich, the oppressed, the homeless, the dear departed , the bold, the beautiful, the honest are all
nounsreferring to a class of people.

Unlike a Collective Noun (which is generally singular, but plural in certain contexts) a collective adjective is
always plural and requires a plural verb:

Correct: The poor are exploited.


Correct: The rich are insensitive.
Correct: The old have a tough life.
Correct: The young are a joy to be with.

Incorrect: The rich has to help the poor.


Correct: The rich have to help the poor.

Floating AND Afloat.

There is a long list of adjectives that have the prefix a — afloat, adrift, aground, awake asleep, alike, ablaze,
ashore, awash, ashamed and so on. These adjectives generally indicate a state that is achieved by the word to
which the prefix a is attached. (afloat— in a floating condition).

iQue ideas Page 50


Another class of a adjectives is: averse, aghast, aware, afraid, aloof and so on.

Be careful their position in the sentence. Some of these adjectives cannot be used before a noun. They are
used after a stative (not active) verb like is, feel, look or seem.

Incorrect: The asleep boy.....


Incorrect: The alone girl ....

Correct: The boy is asleep.


Correct: The girl is alone.

However, an alert person, an active person, a half-asleep boy, a wide awake child are all correct.

`Flying planes can be dangerous'.


Participles and ambiguity. (compare with 'gerunds' above)

A verb in its -ing for or —ed/en (the past perfect) form is called a participle. For example swimming and
loving are participle. The -in form is called a present participle. They can function as nouns (gerunds) or
adjectives.

I love travelling. (noun — gerund)


She is a travelling salesman. (adjective — participle)

The past perfect form of the verb is called a past participle. They can also function as a noun or an adjective.

He is a learned man. (adjective)


The learned are not always easy to get along with. (noun)

Since we know that participles (verb+ing, verb+ed, etc.) are also adjectives we have to be clear about their
use and learn to spot and avoid ambiguities arising out of their careless use.

Ambiguous: Flying planes can be dangerous. (What can be dangerous? Planes that are flying or the action of
flying panes?)

Improvement: Planes that are flying can be dangerous OR Flying a plane can be a dangerous activity.

A frightening student and afrightened student are entirely different things. Whether this book is a confusing
one or a confused one will speak a lot about either of us.

ADVERBS

An Adverb can modify a verb, an adjective or another adverb. Since adverbs modify several things in several
ways they are categorized into different types — of manner, place, frequency, time, purpose etc. In this
book, our emphasis is not on the technical aspects of grammar, but on usage and function. We will consider
adverbs as words that modify something else in the sentence.

Adverbs tell us when, where, why, or under what conditions something happens or happened.

iQue ideas Page 51


Adverbs of Adverbs of Adverbs of Adverbs of Adverbial Negative
Frequency Manner Location Time Conjunctions Adverbs
always carefully ahead again also barely
ever correctly back early consequently hardly
frequently eagerly forward late furthermore little
generally easily here now hence never
never fast high sometime however not
often loudly low then moreover nowhere
rarely patiently near today nevertheless rarely
seldom quickly outside tomorrow otherwise scarcely
sometimes quietly somewhere tonight therefore seldom
usually well there yesterday thus

"real sick" OR "really sick"?

Incorrect: She was real sick.


Incorrect: He worked real fast.

Correct: She was really sick.


Correct: He worked really fast.

Adverbs can modify adjectives, but an adjective cannot modify an adverb. In the first sentence we have real
trying to modify another adjective sick, and in the second sentence real tries to modify the adverb fast
(worked fast). Both are hence incorrect. Adverbs can do both these functions.

(Constructions like 'She left real quick', 'He drives slow on the highway' etc. are informal use and are
incorrect)

Pretty good AND Pretty well

Correct: Her scores were pretty good.


Correct: She scored pretty well.

In the first sentence good is an adjective; pretty which is commonly an adjective can also function as an
adverb. Though certain grammarians frown upon the first sentence, pretty is classified as an adverb as well,
in most dictionaries. We can safely substitute another adverb in the place of pretty, e.g., fairly. (Her scores
were fairly good. She scored fairly we/L)In the second sentence the adverb pretty modifies the adverb well.
Both the sentences are grammatically correct.

Like adjectives, adverbs can have comparative and superlative forms to show degree.

Examples:
Walk faster if you want to catch the train.
The fastest reader will get a prize.

Like with adjectives, more and most, less and least are used to show degree with adverbs:

Examples:
They walked more quickly.
She answered less confidently.

iQue ideas Page 52


This is the most beautifully coloured picture.
He played the least skilfully.

I only invited my friends OR I invited only my friends?

Placement of Adverbs

Always place the adverb (modifiers) immediately next to the word that it modifies. Misplacement can occur
with very simple modifiers, such as only and barely:

Incorrect: I only invited my friends.


Correct: I invited only my friends.

The first sentence has a different meaning from the second, which may indicate that I only invited, but didn't
want them to come — which is ridiculous.

The error of misplaced modifiers is rather widespread. These errors are sometimes related to the
misplacement of a modifying phrase or clause. We will study these in a different chapter.

Incorrect: The panel was unable to discuss the matter completely owing to lack of time. (completely
modifies discuss, but is placed next to words that it cannot modify.)
Correct: The panel was unable to completely discuss the matter owing to lack of time. Incorrect: Students
who seek the advice of their mentors often can improve their performance. (often can modify either seek or
improve. Its wrong placement has created an ambiguous sentence. This error is called a squint modifier)

Correct: Students who often seek the advice of their mentors can improve their performance.
Correct: Students who seek the advice of their mentors can often improve their performance.

We have to remember, however that adverbs (especially adverbs of manner) have the flexibility to occur
anywhere , in the sentence. The placement will not constitute an error in construction.

Correct: Carefully the mother picked up the child.


Correct: The mother carefully picked up the child.
Correct: The mother picked up the child carefully.

Likewise, the adverbs of frequency appear at various points in a sentence:

Correct: I never watch the weather forecast. (before the verb watch)
Correct: I have often told you earlier (in between has told)
Correct: I often used to go to the beach (before used to)

The same is true about adverbs of time too.

Correct: She finally made it to JBIMS. (before the verb made)


Correct: She has recently called me up with doubts. (in between has called)

Questions to test your awareness of the correct placement of the adverbs do appear in competitive
examinations.

iQue ideas Page 53


‘late' AND 'lately'

A few adverbs have two forms, one that ends in -ly and one that doesn't. In certain cases, the two forms have
different meanings:

Correct: He arrived late.


Correct: Lately, he could not attend any of the lectures. (lately means recently)

Compare this with hard and hardly (below).

However, nevertheless, therefore... etc.

A few adverbs can function in ways similar to a conjunction — connecting sentences. Words like, however,
nevertheless, therefore fall in this category. Sentences in the test are sometimes based on their use.

Examples:
If he has nothing new to say day after day, then I am not attending his lectures.
I have told him times without number that I will not tolerate his pranks, and yet again he has gone and done
it.

At the extreme edge of this category, there are the entirely conjunctive devices known as the conjunctive
adverb (often called the adverbial conjunction):

Examples:
He is a good speaker; nevertheless, he is the most nervous person in the GD.
I love these B-Schools; however, I don't think I can afford their tuition.

Notice the punctuation in the above two sentences. When these words are used as conjunctive adverbs
(however, nevertheless, therefore), this is the only correct way you can punctuate the sentences. In their use
as a normal adverb punctuation will be different.

Examples normal adverbial use of however, never the less etc:

It still seems possible, however, that conditions will improve


However did you manage to do it?

'hungry enough' OR 'enough hungry'?

The adverbs enough and not enough usually are placed after the word that they modify.

To be clever enough to get all the money, one must be stupid enough to want it - Gilbert K. Chesterton.

Incorrect: Am I enough tall to play basket ball?


Correct: Am I tall enough to play basket ball?

Incorrect: I am enough hungry to eat a horse.


Correct: I am hungry enough to eat a horse.

When enough comes before a noun it is an adjective.

iQue ideas Page 54


Correct: We had enough time.

The adverb enough is often followed by an infinitive:


She didn't work hard enough to win.

Too.

The adverb too comes before adjectives and other adverbs:


She works too slow.
She works too quickly.

If too comes after the adverb it is usually set off with a comma:

Sanjaya works hard. She works quickly, too.

The adverb too is often followed by an infinitive (to+verb):

She works too slowly to finish this test.

Another common construction with the adverb too is too followed by a prepositional phrase:

This topic is too difficult for us to discuss.

A viewpoint adverb generally comes after a noun and is related to an adjective that precedes that noun:

Correct: A person successful in CAT is often an achiever academically.


Correct: Investing all our money in IT was probably not a sound idea financially.
Incorrect: Financially speaking, investing all our money in IT was a bad idea.

Not and Never

Negative words 'not' and 'never' are technically not part of the verb; they are, in fact, adverbs. These negative
words should not be used along with the so-called negative adverbs like, hardly, seldom, rarely, scarcely etc.

A negative adverb, however, creates a negative meaning in a sentence without the use of the usual
no/not/neither/nor/never constructions

Incorrect: I am not saying nothing.


Incorrect: I am not hardly speaking.

Correct: I am not saying anything .


Correct: I am hardly speaking.

'No sooner did I reach the station ....' OR 'No sooner I reached the station....'?

if a sentence begins with a negative adverb, inverted word order must usually be used.

Incorrect: No sooner I reached the station, than the train left.


Correct: No sooner did I reach the station, than the train left.

iQue ideas Page 55


This construction baffles many students. Study the following examples for clarity.

Examples:

Never before was I in so much pain.


Little did we realise that it was our last meeting. .
Seldom had I felt so uncertain of myself.
Scarcely had we reached the station, when the train left. *
Not for many years were we likely to meet.
No sooner did I reach the station than the train left. *
Rarely had there been more students in the class than there were that day.

*Remember to follow up the paired conjunctions No sooner ... and Rarely... with than, and Scarcely... and
Hardly... with when.

Incorrect: Scarcely had we reached the station, than the train left.*
Incorrect: No sooner did I reach the station when the train left.*
Incorrect: Hardly had I reached the station than the train left.*

The adverb so also requires inverted word order if the sentence begins with it.

So intense was the heat, that we felt dizzy.

So

The intensive use of so is sometimes condemned in handbooks of grammar, but it is standard.


Correct: But the idea is so obvious.
Correct: Why are Bollywood filmsso shallow?

'I feel good' AND 'I feel well'


Adjective or adverb.
(Good - adjective; Well — Adverb)
We are sometimes confused whether in the context we should use an adverb or an adjective. A typical
situation is when we are asked, "How are you?"

Use well after linking verbs (feel, look etc — for more on linking verbs see the section on verbs) relating to
health. In fact, to say that 'you are good or that you feel good' usually implies not only that you're OK
physically but also that your morale is high.

"How are you?" "I am well, thank you."


— refers to health.
"How are you?" "I am good, thank you."
— refers to physical as well as mental well-being.

'He drives slow' OR 'He drives slowly'?


Adjective or adverb.
(Slow - adjective; slowly - adverb)

iQue ideas Page 56


The choice of the adverb or adjective in a sentence depends on the relationship between the subject of the
sentence and the verb used. Compare the following sentences and try to understand the relationship between
the subjects and the verbs used.

He drives slowly.
She looks pretty.

In the first sentence He (the subject) performs the action in the verb drives. In contrast She in the second
sentence does not perform the action of looking at all. (in fact somebody else is looking at her). Once this
relationship is understood, remember to use the adverb when the subject is actively performing the action in
the verb, and use the adjective when the subject is not actively performing the action in the verb.

Correct: He drives slowly.


Incorrect: He drives slow.

For clarity you can study the following sentences in which the use of the adjective and adverb are
grammatically correct.

He became happy.
He grewtall
The food tastesgood.
She appearssad
She looksgreat.
She went wrong in her answers

All the above sentences have verbs followed by an adjective. Compare the last two sentences with

She lookedquickly at the mirror.


She went wrongly into the next classroom.

These are also correct sentences, but with a different meaning.

'bad'AND 'badly'?

Correct: I felt bad looking at her suffering.


Incorrect: I felt badly looking at her suffering.

Use the adjective form after verbs that have to do with human feelings. (Compare with good and well)
However, with other verbs badly will be the correct choice.

Correct: She performed badly in the test today.


Incorrect: She performed bad in the test today.

The use of the adjective in the second sentence is wrong. (Compare with 'He drives slow' OR 'He drives
slowly')

'hard' AND 'hardly'?


Most adjectives add anly and become adverbs. But not all. Hard and hardly are two different words. The
similarity is only superficial. (Hardly is an adverb which means certainly not ... that news is hardly surprising

iQue ideas Page 57


... hardly is normally used with a positive you can hardly find a red one — refer to negative adjectives
above)

Incorrect: You cannot hardly identify what is good for you.


Correct: You can hardly identify what is good for you.

Correct: The children played hard. (hard is an adverb modifying the verb played)
Correct: They are going through a hard time. (hard is an adjective modifying time)
Incorrect: The children played hardly.
Correct: The children hardly played (The children did not play at all)

Remember hardly and hard are two distinct words with different meanings and functions.

‘sure'OR 'surely'?

Sure is an adjective, and surely is an adverb.

The use of sure and surely can at times be a little confusing. But, if you have mastered the adjective you
would surely be able to understand the correct usage.

Correct: He is sure to be here at 9 a.m. (adjective modifying he)


Correct: He will surely be ready at 9 a.m. (adverb modifying ready)
Correct: Surely, man has harmed the environment in no small measure. (adverb modifying harmed)

'near'AND 'nearly'?

The problem with near is that it can function as a verb, adverb, adjective, or preposition. Nearly is used as an
adverb to mean 'in a close manner' or 'almost but not quite'.

Correct: The moment of truth neared. (verb meaning approached)


Correct: We will meet in the near future. (adjective modifying future)
Correct: He moved near. (adverb modifying moved)
Correct: He stays near my house. (preposition) Correct: He nearly missed the train. (adverb)

Review Exercise

Underline the adjectives (if any) in the following sentences. If there is no adjective in the sentence write 'no
adjective' against the sentence. .

1. Sudoku is hardly a fun puzzle.


2. My computer has a virus.
3. Computer games are my favourite pastime.
4. I have upgraded the RAM in my computer.
5. The computer often hangs because of less memory.
6. My service agent is very helpful.

Write Adjective or Adverb against the last word of each sentence by studying its function.

7. The fire burnt the shrubs quickly.

iQue ideas Page 58


8. It burnt the hillside black.
9. He drove the point hard.
10. She drives me mad
11. I find this option unlikely.
12. We found her friendly
13. The traffic jam made most of us late.
14. The doctor's medicine made her well.
15. Why do you say this problem is so hard?
16. We shall go home early.
17. We shall leave the glasses empty.
18. You can paint the wall white.
19. He can paint this wall fast.
20. Pull the rope hard.
21. Pull the rope tight.

Strike out the incorrect word

22. The policeman looked cautious/cautiously around the corner.


23. Everything fitted in place beautiful/beautifully.
24. Please work quiet/quietly during the test.
25. She spoke firm/firmly to the salesman.
26. The suspect appeared calm/calmly
27. The policemen searched careful/carefully for clues.
28. During the show the audience grew quiet/quietly.
29. That man looks suspicious/suspiciously to me.
30. She looked sad/sadly at the little puppy.
31. The surface felt rough/roughly to the touch

Do as directed:

32. Dry the table with a soft absorbent towel. (Insert 'carefully' at the most appropriate place in the sentence)
33. She has never been a fan of Big B. (Rewrite beginning with 'never')
34. The medicine takes effect after a few hours. (Insert 'usually' at the most appropriate place in the
sentence)
35. We have seldom had a student with such self discipline. (Rewrite the sentence beginning with 'Seldom')
36. I thought I would never complete this book. (Rewrite the sentence beginning with 'Never')
37. She had scarcely finished speaking when the audience began to leave. (Rewrite the sentence beginning
with "Scarcely')
38. I have trouble accessing my email account. (Insert 'sometimes' at the most appropriate place in the
sentence)
39. Leroy has wanted to get into an JIM . (Insert 'always' at the most appropriate place in the sentence)
40. She buys her provisions at the mall. (Insert 'usually' at the most appropriate place in the sentence)
41. Weeds grow throughout the rainy season in front of my house in the village. (Insert 'rapidly' at the most
appropriate place in the sentence)

iQue ideas Page 59


Answers & Explanations

1. Sudoku is hardly a fun puzzle.


2. My computer has a virus.- No adjective
3. Computer games are my favourite pastime
4. I have upgraded the RAM in my computer. No adjective.
5. The computer often hangs because of less memory.
6. My service agent is very helpful.
7. The fire burnt the shrubs quickly. (Adverb)
8. It burnt the hillside black (Adjective)
9. He drove the point hard. (Adverb)
10. She drives me mad (Adjective)
11. I find this option unlikely. (Adjective)
12. We found her friendly.(Adjective)
13. The traffic jam made most of us late. (Adjective)
14. The doctor's medicine made her well. (Adjective)
15. Why do you say this problem is so hard? (Adjective)
16. We shall go home early. (Adverb)
17. We shall leave the glasses empty. (Adjective)
18. You can paint the wall white. (Adjective)
19. He can paint this wall fast. (Adverb)
20. Pull the rope hard. (Adverb)
21. Pull the rope tight. (Adjective)
22. The policeman looked cautiously around the corner.
23. Everything fitted in place beautifully.
24. Please work quietly during the test.
25. She spoke firmly to the salesman.
26. The suspect appeared calm.
27. The policemen searched carefully for clues.
28. During the show the audience grew quiet.
29. That man looks suspicious to me.
30. She looked sadly at the little puppy.
31. The surface felt rough to the touch.
32. Dry the table with a soft absorbent towel. (Insert 'carefully' at the most appropriate place in the sentence)
Carefully dry the table with a soft absorbent towel OR Dry the table carefully with a soft absorbent
towel.
33. She has never been a fan of Big B. (Rewrite beginning with 'never') Never has she been a fan of Big B.
34. The medicine takes effect after a few hours. (Insert 'usually' at the most appropriate place in the
sentence) The medicine usually takes effect after a few hours.
35. We have seldom had a student with such self discipline. (Rewrite the sentence beginning with 'Seldom')
Seldom have we had a student with such self discipline.
36. I thought I would never complete this book. (Rewrite the sentence beginning with 'Never') Never did I
think I would complete this book.
37. She had scarcely finished speaking when the audience began to leave. ( Rewrite the sentence beginning
with "Scarcely') Scarcely had she finished speaking when the audience began to leave.
38. I have trouble accessing my email account. (Insert 'sometimes' at the most appropriate place in the
sentence) I sometimes have trouble accessing my email account.
39. Leroy has wanted to get into an IIM. (Insert 'always' at the most appropriate place in the sentence) Leroy
has always wanted to get into an IIM

iQue ideas Page 60


40. She buys her provisions at the mall. (Insert 'usually' at the most appropriate place in the sentence) She
usually buys her provisions at the mall. OR Usually she buys her provisions at the mall.
41. Weeds grow throughout the rainy season in front of my house in the village. (Insert 'rapidly' at the most
appropriate place in the sentence) Weeds grow rapidly throughout the rainy season in front of my house
in the village.

iQue ideas Page 61


7
Verbs
Verb shows mental or physical action (of the subject):

He leads a quiet life.


She runs fast.

Verb shows being (of the subject): She is fast

Verb shows (the subject) being acted upon: The book was written in the 19th century.

With transitive verbs, the action of the verb passes over from the subject to the object.

She writes poems.


(The action of the subject writes passes over to the object 'poems'.)

With intransitive verbs the action is complete in itself..

She weeps.
When the action of the subject passes over to itself the verb becomes reflexive.

We enjoyed ourselves.
(In other words the subject and the object are the same.)

Some verbs may not require an object, but remain incomplete in meaning without something else to
complete their meaning. What completes their meaning is called a complement. Such verbs are called

Linking (or Copulative) Verbs.


She appears honest.

There are certain words like be, have, shall, will, etc. which occur with other verbs. These are called
auxiliary verbs.

The functions of the verb are to indicate the Tense, the Mood, and the Voice (active and passive).

There are only four basic verb forms. They are ( for the verbs walk, take, and sing)

i. I walk/ She walks, I am walking, I walked, I have walked.


ii. I take/ She takes, I am taking, I took, I have taken.
iii. I sing/ She sings, I am singing, I sang, I have sung.

These four forms combine with other words to perform the various functions of the verb. i.e., I will be
singing indicates the future continuous action by combining with 'will be'.

iQue ideas Page 62


Linking Verbs.
Many important verbs do not express action. They help make a statement and they link a noun or adjective to
the subject. These are called linking verbs. The following are linking verbs: become, feel, grow, taste,
remain, he, am. is, was, been, are, were, being, look, seem, smell, sound, appear, stay, prove, turn etc.

Verbs related to the senses (sound, smell etc) and verbs that describe a state (appear, become etc.) are linking
verbs. They are also called stative verbs.

She is all right.


She seems all right.

Some of the linking verbs can also function as action verbs For example:

The food tasted good. (linking verb)


They tasted the food. (action verb)

'It is I' OR 'It is me'?

When a personal pronoun follows a linking verb remember to always use the subject case of the pronoun.
Subject cases are :I,we, you, he, she, it, they. Me, us etc. are object cases. (Refer to the section on pronouns).

Correct: It is I at the door.


Incorrect: It is me at the door.
Correct: This is he speaking. (on the phone)
Incorrect: This is me speaking.

Incorrect: It was us who had arrived before they left.


Correct: It was we who had arrived before they left.

Incorrect: It will be us who will decide on this issue.


Correct: It will be we who will decide on this issue.

Let's (Let us) work together vs. Let we work together.


Imperative Mood

The English verb is distinct in its functions to denote either a fact or a command, or a possibility or wish.
These are grammatically termed as Moods. A fact is indicative mood. A command is imperative mood. A
possibility or wish is Subjunctive mood.

Indicative mood: The English verb is distinct in its functions.

Imperative mood:
Complete your homework before you come to the class.
Please leave me alone.
Shut up!

Subjunctive mood:
I wish I were a bird (Subjunctive is explained later)

iQue ideas Page 63


Notice that the Imperative sentences have no subject. The implied subject in all imperative sentences is the
second person (you) either singular or plural. The only exception to the imperative is the first person subject
in sentences like:

Correct: Let us work together.


Correct: Let us decide on this issue.

Incorrect: Let you and I go together.


Incorrect: Let we work together.
Incorrect: Let you and I decide on this issue.
Correct: Let me decide on this issue.
Correct: Let them decide on this issue.

These are imperative sentences with 'you' hidden subject and 'them' and 'me' as the objects of the verb 'let'.
See also the use of pronouns with let under pronouns.

If I were a millionaire AND If I was a millionaire.

The Subjunctive Mood.


A sentence in the subjunctive does one of the following:

 it expresses a wish
 it begins with if and expresses a condition contrary to fact
 begins with as if and as though and describes a speculation
 begins with that and expresses a demand or suggestion.

I wish I were a millionaire.


If I were a millionaire ...
He acted as if he were innocent. .
She treats him as though he were a machine.
It essential that she be present.

Correct: If I were a millionaire, I would buy a house at the beach.

With if clauses in the subjunctive use 'would' or 'could' in the main clause.

These are some of the fixed expressions in the subjunctive.

Long live the King!


God bless you!
Heaven forbid!
Be that as it may, he still wants to get into an IIM.
Come what may, I will clear the CAT.
We are all in the same boat, as it were.

Conditional Clauses

When you construct sentences with 'if remember to follow one of these tense sequences of the verbs. This
sequence is rigid and must not be broken. Error spotting sentences usually have one of these errors.

iQue ideas Page 64


If Clause Main Clause
If you heat ice, it melts. (present + present)
If it rains, we will play football. (present +future)
If it rained, we would play football (past + would)

If it had rained, we would have played football.


(past perfect + would have)
Remember any other combination is wrong.

Normally if clauses will not contain will or would. However, in an exceptional sentence where If is used to
express willingness or request the clause can contain will. But the meaning is different from the above. For
example:

Correct: If you will drop in tomorrow, I will give you the salary.

Active and Passive voice.

In a sentence using active voice, the subject of the sentence performs the action expressed in the verb. In a
sentence using passive voice, the subject is acted upon; the subject receives the action expressed by the verb.

One of the players called an illegal time-out. Active Voice


An Illegal time-out was called by one of the players. Passive Voice.
The panel approved the fee reduction. Active Voice

The fee reduction was approved by the panel. Passive Voice.

 At times a grammatically correct passive sentence can be utterly awkward if it is not constructed for the
above reasons.

For example:

Correct: She opened the door at the sight of her child.


Awkward: The door was opened by her at the sight of her child.

 Another point to remember is not to mix active and passive voice in the same sentence.

Incorrect: She spoke to the Principal and her leave application was approved.
Correct: She spoke to the Principal and got her leave application approved.

Incorrect: The panel approved the fee reduction and the academic calendar was revised.
Correct: The panel approved the fee reduction and revised the academic calendar.

However, the passive voice is justified when the agent (do-er of the verb) is either obvious, or unimportant,
or just unknown. Another reason to use the passive voice is to avoid mentioning the agent at all, or to
highlight the action instead of the agent. If these reasons are not present, construct the sentences in the active
voice in all your writing.
Hence you will find that most of scientific writing is in the passive.

Examples:

iQue ideas Page 65


The North Star can be observed directly overhead at the Earth's North Pole.
Bombs were placed at generally crowded locations.
The blast was averted.
The ad was designed to appeal to the young.

 Still another point to bear in mind is that only transitive verbs (verbs that require an object) must be
transformed into passive constructions.

But this does not mean that all transitive verbs can be transformed. He loves her. though transitive, cannot be
transformed into She is loved by him. The same way there are certain verbs that forbid the passive. For
example: She has a new boyfriend, or The bottle contains medicine, cannot be transformed into A new
boyfriend is had by her or Medicine is contained by the bottle. These are verbs that cannot be transformed
into the passive. A few other such verbs are: resemble, look, like, equal, agree, with, mean, contain, hold,
comprise, lack, suit, fit, become etc.

Some of you are confused about the verb forms in the passive in different tenses. The table below lists the
different tense forms, so that you don't inadvertently mark any of these as errors in themselves.

Auxiliary
Tense Singular Subject Plural Subject Past Participle
E.g. He E.g. They
Present is are shown.
Present Perfect has been have been shown.
Past was were shown.
Past Perfect had been had been shown.
Future will be will be shown.
Future Perfect will have been will have been shown.
Present Progressive is being are being shown.
Past Progressive was being were being shown.

Auxiliary Verbs
Usage Notes on Auxiliary Verbs

Auxiliary Verbs: will, shall, may, might, can, could, must, ought to, should, would, used to, need.

Function: help express shades of time, mood, necessity, advice, ability, expectation, permission, possibility,
etc when they combine with main verbs.

Example: I will have been teaching for ten years.


Auxiliary verb: will have been
Main verb: teaching

Shall, and Will.


Shall and will are used to indicate the future.

When you use shall with the first person I and we, it indicates the simple future tense. When you use will
with the first person I and we, it indicates future tense that expresses determination, promise, etc. (or
coloured future).

Shall we meet tomorrow? — is a simple query.

iQue ideas Page 66


Will we meet tomorrow? — expresses the speaker's anxiety.
We shall complete this assignment by tomorrow. — as a matter of fact..
We will complete this assignment by tomorrow — shows determination and commitment.

It is just the reverse for the second and third persons.

Using shall in second and third persons (you, he, she, it, they, etc.) would indicate some kind of stipulation,
as in –
You shall not steal.
The contraction 'll, is used for both shall and will. You'll not steal.

Legal documents and other such formal texts use shall abundantly.

The students shall pay the fees not later than the 30th.

Shall and Should


The use of shall is rather infrequent, but it is used with I and we for offers and suggestions:

Shall I open the window?


Shall we get out of here?

Should is the past tense of shall and also used to mean 'ought to' as in:

You should carry an umbrella.

Shall when used a simple future and changed into the past tense (as in reported speech) is not should. Use
would instead.

For example:
The students said, "We shall complete this assignment by tomorrow."
The students said that they would complete the assignment by the next day.

Should is used:
to give advice : You should study regularly
in hypothetical situations : Should you need my help, just give me a call.
to give tentative opinions : I should think that he keeps his promises.

Have, Has and Had


Forms of the verb to have are used to create tenses known as the present perfect and the past perfect.

He has left. (Present perfect)


They have left. (Present perfect)
He had left before they arrived. (Past perfect)

The perfect tenses are used when an action took place in the past, but has an effect either on the present
(present perfect) or on another moment in the past afterwards. (past perfect) Refer to tenses for this.

iQue ideas Page 67


The students at times have a problem with has had, have had, and had had constructions. The verb has and
have are used as an auxiliary as explained above to create the perfect tense forms, and/or as a main verb.
Have as main verb has several meanings. Refer to a dictionary for the different meanings.

When has or have is used as main verb and its tense is changed into the perfect, the verb will occur twice —
the first is the auxiliary and the second is the main verb.

For example:
I have a cold; He has a cold. — Simple present.
I had a cold; He had a cold — Simple past.
I have had a cold; He has had a cold — Present perfect
I had had a cold; He had had a cold. — Past prefect.

When has/have is combined with an infinitive to form an auxiliary whose meaning is similar to 'must', we get
common expressions like:

He has to work hard.


I have to have a seat in the IIMS.
He has to have been the first student (from this coaching Institute) to get into an IIM.

All these are correct sentences.

Can and Could

The modal auxiliary can is used in the following ways:

I can attempt enough number of questions but cannot clear the cut —off. (to express ability)
Can I come in? (to seek permission)

(Note that can is less formal than may. Also, some authorities will object to the use of can in this context.
Hence, it is better to avoid this use in strictly formal contexts. See 'can and may' below)

Tata Motors can definitely make better cars than what they are making now. (to express a possibility)

The modal auxiliary could is used in the following ways:


I could attempt enough number of questions but couldn't clear the cut —off (past tense of can)
Could I borrow your bike? (To seek future permission)
We could always go to the movies. (to express a possibility )
She could pass, if she studied regularly. (to express ability subject to conditions)
Could you help me with this Reading Comprehension questions? (polite requests)

Can and May


The modal auxiliary may is traditionally used to seek permission (especially in formal contexts)

May, I come in?


May I speak to Ms. Sanjaya, please?

iQue ideas Page 68


The auxiliary verb can be used to seek permission in informal situations. Primary school students are
sometimes at the receiving end of sarcasm from teachers. "Can 1 come in teacher?" "I don't know if you can,
but you may."

The point then is, may is the best for seeking permission and it is advisable to follow the traditional rule.

May and Might

May and might are interchangeable when we want to express possibility.

I may take up a job after graduation.


I might take up a job after graduation.
Might is the past tense of may.

The student might do nothing without the teacher's permission.

Used in seeking permission might implies greater degree of politeness and uncertainty.

Might I leave the class early today?

'May have been' and 'might have been' communicate two different possibilities.

He may have been hurt. (It is not known whether he is hurt or not — but it is a possibility.)

He might have been hurt. (It is known that he is not hurt — but the possibility of his getting hurt existed in
the past. He is now safe)

Incorrect: Without the effort of the police the murder may have remained unsolved.
Correct: Without the effort of the police the murder might have remained unsolved.

Will and Would


Refer to Shall and Will also.

Will does the following:

If it rains we will play football. (express willingness)


I will inform you tomorrow. (express intention)
The class will begin as soon as the teacher arrives. (prediction)

Would does the following:

Would you help me out? (more polite than will)


He would tell her all kinds of lie. ( characteristic activity)
He would be dead if she didn't love him. (hypothetical)
The phone is ringing. That would be Parag. (probability)

Gerunds
A verb + ing (swimming) is called a gerund when it functions like a noun.

iQue ideas Page 69


Swimming (gerund) like any other noun, is the name of an exercise/sport. A gerund can do all that a noun
does.
Dieting may help you reduce. (subject of the sentence)

They appreciated my painting. (Object of praise)


His favourite activity is sleeping. (Complement of is)
He was arrested for stealing. (Object of the preposition for)
Notice that in all the above sentences you can substitute the gerunds with nouns.
E.g. He was arrested for theft.
Incorrect: My mom is worried about me travelling alone to Delhi.
It is not possible to substitute a noun for traveling. Hence, we need to precede a gerund (noun) with the
possessive pronoun.
Correct: My mom is worried about nay travelling alone to Delhi.
Study these sentences:
Telling her the truth is useless.
We recommend sending your applications early.
He converted the call by scoring well in GD and PI.

Instead of a pronoun if there is a plural, collective, or abstract noun before the gerund, do not use the
possessive.

All my friends object to students smoking on the campus.


The team playing a defensive game was the idea of the coach.
The story demonstrated love overcoming all obstacles.

When an indefinite pronoun comes before a gerund, using the possessive is likely to create an error. In the
exam choose an option that rephrases such sentences in a better way.

Incorrect: She was shocked by someone's making that remark.


Correct: She was shocked that someone would make that remark.

When a noun phrase precedes the gerund, do not use the possessive.

Incorrect: I was deeply touched by the five-year-old child's offering me help.


Correct: I was deeply touched by the five-year-old child offering me help.

Also notice that when the emphasis falls not on the gerund, but on the pronoun the object pronoun can
precede a gerund.

My friends object to my smoking.


My friends saw me smoking. I saw them watching me.

Infinitives
to + verb is called an infinitive. E.g. to seek
Although an infinitive is easy to locate because of the to + verb form, deciding what function it has in a
sentence can sometimes be confusing. The following sentences illustrate the functions of an infinitive. .
To wait seemed foolish. (subject)
Everyone wanted to go. (direct object)
His ambition is to fly. (subject complement)

iQue ideas Page 70


He lacked the strength to resist. (adjective)
We must study to learn. (adverb)

Split infinitives:
Split infinitives are when additional words are included between to and the 'verb' in an infinitive (e.g. to
quickly go). This is not accepted by some authorities. Hence, choose an option without the split infinitive if
it is available. However, it may be necessary to split the infinitive in sentences like these:
Correct: He was able to more than double his salary in one year.
Incorrect: I like to on a nice day walkon the beach.
Correct: On a nice day, I like to walk on the beach.

Gerunds and Participles


The use of gerund or participle can change the meaning of sentences
Correct: She was not impressed with their competing (The competing did not impress him. It is used as a
gerund hence their.)
Correct: He was not impressed with them competing. (They did not impress him as they completed.)
Do not immediately spot an error because you see them competing Completing is used as a participle to
convey as they completed.)

Gerunds and Infinitives


Gerund: swimming
Infinitive: to swim
Their functions overlap. Gerunds function as nouns. Infinitives function as nouns too. Deciding which to use
can be confusing in many situations.

Some verbs take only gerunds and some others only infinitives, and still some others both. Study these
sentences and try to understand why the incorrect sentences do not make sense.
Correct: I hope to get into an IIM.
Incorrect: I hope getting into IIM.
Correct: He avoids drinking before driving.
Incorrect: He avoids to drink before driving.

Study these sentences and try to work out the difference in meaning.
Correct: They like to go to the movies.
Correct: They like going to the movies. (meaning is different)

The difference in meaning is that to go indicates a potential event (`potential, hypothetical, or future events')
while going indicates an actual event.

Forget and remember

These two verbs change meaning depending on whether a gerund or infinitive is used as the object.
She forgot to do her homework. (She didn't do her homework.)
She forgot doing her homework. (Se did it, but she didn't remember.)
She remembers to do her homework. (She regularly remembers.)
She remembers doing her homework. (She did it, and remembers now.)

Certain verbs (feel, hear, see, smell, etc.) take an object followed by either a gerund or a simple verb (e.g.
shout). With many of the verbs that follow the object, the use of the gerund indicates continuous action while

iQue ideas Page 71


the use of the simple verb indicates a one-time action. Still, sometimes the simple verb can indicate
continuous action if one-time action wouldn't make sense in the context.

Study these examples. Try to figure out the difference in meaning.


Correct: We heard him quarrelling with the neighbour. (continuous action)
Correct: We heard him quarrel with the neighbour. (one-time action)
Correct: We heard the victim shouting for help. (continuous action)
Correct: We heard the victim shout for help. (one-time action)

Causative Verbs
Certain verbs are immediately followed by an object (noun or pronoun) and the subject and the object have a
cause-effect relationship. Here is a brief list of causative verbs: help, allow, have, let, require, motivate. get,
make. convince, hire, assist, encourage, permit, employ, force, and allow ... there are more. Some of them
require a "to" while others do not. Note the following patterns:
I told him to clean his room.
I insisted that he clean his room.
I made him clean his room.
Notice the use of ...to clean, that he clean, him clean patterns. Since these are rigid patterns any change is
an error.

Incorrect: I made him to clean his room.


Incorrect: I suggested him to clean his room.
Incorrect: I told that he clean his room.

The most common of these errors is the unnecessary use of the word 'to'

Incorrect: I suggested Kunal to write a test everyday.


Correct: I suggested that Kunal write a test everyday.

Some of the verbs that take the "to" form are: require, command, force, order, remind, and urge.

The verb help can be used with or without "to":"help him (to) clean his room."

Some of the verbs which can take the "that" pattern are: ask, require, request, and demand

VERB TENSES

In competitive exams the error related to tenses are to be found in sentences with two or more clauses where
there are an unjustifiable shifts in tenses. It is hence necessary to get an overview of the English tenses
before we can discuss these errors.

English tenses have two aspects: (i) form — by which we understand the name of the tense, e.g. Simple
present, Present continuous etc. (ii) time — by which we mean the time of the action indicated by the form.
The table below illustrates these. The form of the tense is in the top band. On the left are the various time
implied by this form. On the right are example sentences.

iQue ideas Page 72


Simple Present
Now or the present condition I understand what you are saying
General Truths The sun rises in the east
Habitual Action or a fact She smokes. She visits the temple everyday. She
likes music.
Future time The show starts at 6 p.m. tomorrow.
Present Continuous
Now. Activity in progress He is driving and cannot take your call.
Perception She is feeling lonely
Simple Past
Completed Action I took the test yesterday..
Completed condition The house was built 50 years ago.
Past Continuous
Action that took place over a period of time in the It was raining continuously for several days
past
Past action interrupted by another It was raining when they arrived.
Future
With will or won't — activity that will/will not I will get up early tomorrow
happen/exist in the future I won’t get up early tomorrow
With ‘going to…’ Future in relation to conditions in I am going to get something to eat. I am hungry.
the present.
Present Perfect
with stative verbs events that begin in the past and He has lived in Mumbai for many years.
lead up to and continue in the present
Habitual or continued action She has worn glasses all her life.
With 'ever, never, before' an undefined/unspecified Have you ever been to Mumbai before?
time in the past
Present Perfect Continuous
To express duration of an action that began in the It has been raining for several hours and the roads
past, has continued into the present, and may aren't flooded yet.
continue into the future.
Past Perfect
To describe a past event or condition completed When I arrived home, she had already slept.
before another event in the past
In reported speech Bhavin said that he had informed everyone.
Past Perfect Continuous
To express duration of an action that began before a He had been living in Mumbai for many years before
specified moment in the past the riots uprooted him.
Future Perfect
To express action that will be completed by or before By this time next month, we will have written the
a specified time in the future. CAT. She will not have .finished her work until
tomorrow.
Future Perfect Continuous
To express a continuing action that will be By this time next month, we will have been studying
completed by or before a specified time in the future. for CAT for more than a year.

Notes:
Understanding the six basic tenses allows you to recreate much of the reality of time.
These six are:
Simple Present: They walk
Present Perfect: They have walked
Simple Past: They walked
Past Perfect: They had walked

iQue ideas Page 73


Future: They will walk
Future Perfect: They will have walked

Problems in sequencing tenses usually occur with the perfect tenses.

The present perfect consists of a 'has' or 'have' before the third form (participle) of the verb: e.g. has have
walked/cut/spoken.

I taught for ten years. (simple past)


— I am retired now.

I have taught for ten years. (present perfect)


— the fact that I taught for 10 years is well expressed by the simple past. The present perfect then tells us
that the effect of my action in the past continues into the present somehow. Hence the sentence, I have taught
for ten years' is a response to a situation that discusses in the present my experience of the last ten years. In
all probability I am still teaching.

One more example:


It rained last night (simple past).
— expresses the fact that it rained.

It has rained last night (present perfect)


— is a response to its effect in the present. For example, if my question to you is 'why are the roads flooded?'
(no,„ or, 'why are the trains running late?'(now) , you may respond with 'It has rained heavily last night', etc.
in other words, the event in the past finds its 'perfection' in the present.

Consider these sentences for clarity.


He finished his dinner. He can watch TV.
If he has finished his dinner he can watch TV The past perfect tense designates action in the past just as
simple past does, but the action of the past perfect is action completed in the past before another action.
He finished his dinner and later watched TV. (past)
He watched TV when he had finished his dinner. (past perfect)
In sentences expressing condition and result, the past perfect tense is used in the part that states the
condition.
If he had studied harder he would have passed the examination.
Remember that there can only be one 'would have' clause in a sentence.
The future perfect tense designates action that will have been completed at a specified time in the future.
Saturday I will finish my housework. (simple future)
By Saturday noon, I will have finished my housework. (future perfect)

Verb Tense Consistency


Erroneous or unnecessary shifts in tense..
Once you have a fair idea about the form and the time involved in each tense (study the table) it becomes
easy to spot the errors or the unnecessary shifts in tense in the sentences that are in the question paper.
Do not shift from one tense to another if the time frame for each action or state is the same.
Incorrect: The air contains several pollutants that emanated from industrial emissions.
Contains is present tense, referring to a current state; emanated is past, but should be present (emanate)
because the pollutants still emanate from the emissions.
Correct: The air contains several pollutants that emanate from industrial emissions.

iQue ideas Page 74


Incorrect: In the evening, the sky became overcast, winds strengthened, and a low rumble indicates the
parting monsoon.

Be came and strengthened are past tense verbs; indicates is present but should be past (indicated) to maintain
consistency within the time frame.
Correct: About sunset, the sky darkened, winds strengthened, and a low rumble indicated the pre-monsoon
showers. Incorrect: Yesterday we had gone to school by carbut later took the bus home. .
had walked is past perfect and rode is past simple. Both these events need to maintain consistency in tense.
The time frame in this context is of yesterday. Hence, both verbs need to be in the simple past to be
consistent within this time frame.
Correct: Yesterday we went to school by car but later took the bus home.

Change the tense to indicate a change in time frame from one action or state to another.
Correct: Even before the election took place, he had started behaving like a minister.
Correct: The students are studying extra hours because they know that this year's test will be very difficult.

It is not possible to say that the use of a particular tense/form is wrong in a single clause. The errors in tense
have to be found only in relation to the several events mentioned in the sentence. For example, She was
listening attentively, or She has been listening attentively are not different from each other in isolation, but
only in relation to another event that has to be mentioned. In competitive examinations, questions in which
you have to deal with an error in Incorrect: will always have two events at least. In relation to one, the other
may not be consistent in the given time frame.
Incorrect : It was we who arrived before they had left.
Correct : t was we who had arrived before they left.

Sequence of Tenses (Broad guidelines)

Incorrect: I am eager to take the CAT, because I wanted to get into an IIM.
Correct: I am eager to take the CAT, because I want to get into an IIM.
Combine a simple present with simple present to show same time action

Incorrect: I know I had made a mistake.


Correct: I know I made a mistake.

Combine simple present with simple past to show an earlier action.

Incorrect: I believe that I did not get upset.


Correct: I believe that I have not got upset.

Combine simple present with present perfect to show a period of time extending from the past to the present.

Incorrect: The teacher says that she would check our home work.
Correct: The teacher says that she will check our home work

Combine simple present with future tense to show action to come, use the future tense.

Incorrect: I wanted to cry because she has stolen my pencil.


Correct: I wanted to cry because she stole my pencil.

iQue ideas Page 75


Combine simple past with simple past to show another completed past action.

Incorrect: She knew that she has made a mistake.


Correct: She knew that she had made a mistake.

Combine simple past with Past Perfect to show an earlier action.

Incorrect: Copernicus discovered that the earth was round.


Correct: Copernicus discovered that the earth is round.

Combine simple past with Present tense to state a general truth.

Incorrect: She has grown to be wiser after she had got married.
Correct: She has grown to be wiser after she got married.
Incorrect: She had grown to be wiser after she has got married.
Correct: She had grown to be wiser ever since she got married.

Combine Present Perfect or Past Perfect with past tense for any purpose.

Incorrect: I will be happy if they will declare the results today.


Correct: I will be happy if they declare the results today.

Combine Future tense with present tense to show action happening at the same time.

Incorrect: She will surely pass if she had studied harder.


Correct: She will surely pass if she studied harder.

Combine Future tense with past tense to show an earlier action.

Incorrect: The tuition will probably increase next year if enrollments had not increased.
Correct: The tuition will probably increase next year if enrollments have not increased.

Combine future tense with present perfect to show future action earlier than the action of the main clause.

Incorrect: The sun will have set by the time we will reach the sea shore.
Correct: The sun will have set by the time we reach the sea shore.
Correct: The sun will have set by the time we have reached the sea shore.

Combine future perfect tense with present tense or present perfect tense for any purpose.

Phrasal Verbs

When a verb combines with other words for example, a preposition, the resulting phrase sometimes acts like
a new verb. These are called phrasal verbs. They can have unexpected (idiomatic) meanings. The extra word
is called a particle.

Look after means to take care of After is the particle.

iQue ideas Page 76


Phrasal verbs provide test writers with useful resources for writing tests. The difficulty with phrasal verbs is
that they can have different meanings in different contexts. Some of them cannot be separated from their
particle, and others can be. The list of phrasal verbs is endless. The same verb combines with different words
to create an endless list of phrasal verbs.

Here is (a partial) list of phrasal verbs with the verb Go:

Go about, Go across, Go after, Go against, Go ahead, Go ahead with, Go along with, Go around, Go at, Go
away, Go back, Go back on, Go before, Go below, Go by, Go down, Go down on, Go down to, Go down
with, Go for, Go forth, Go forward, Go in, Go in for, Go in with, Go into, Go it alone, Go off Go off with,
Go on, Go on about, Go on to, Go on with, Go out, Go out for, Go out to, Go out with, Go over, Go over to,
Go past, Go round, Go through, Go through with, Go together, Go towards, Go under, Go up, Go up to, Go
with, Go without etc.

I am told that the verbs Go and Get each have more than 50 phrasal verbs. Read a lot of good English and
keep a good dictionary at hand. The Oxford Dictionary of Idioms will be of help.

The tests include usage issues related to phrasal verbs and their correct use, e.g. some particles can be
separated from the verb so that a noun and pronoun can be inserted, and some particles can't be separated
from the verb. In addition, some phrases are intransitive, meaning they cannot take a direct object. These are
briefly illustrated below. It is not possible, within the framework of this book, to list or to examine phrasal
verbs in detail. That must become your homework.

add up (to add) — Separable


Correct: She added up the total in her notebook.
Correct: She added it up in her notebook.
get around (to evade) — Inseparable
Correct: She always gets around the rules.
Incorrect: She always gets the rules around.
catch on (to understand) — Intransitive (cannot take direct object)
Correct: After I explained the maths problem, she began to catch on.
Incorrect: She began to catch on the maths problem. (catch on cannot take a direct object in this meaning.)

Unfortunately, there is usually no indicator whether an idiomatic phrase is separable, inseparable, or


intransitive. In most cases the phrases must simply be memorized.

Review Exercise
Fill in the blanks. Choices are in brackets.

1. She is only five feet tall. If she _________ (was/were/is) a few inches taller, she could be working in
Airlines.
2. I hate dogs. If I a dog ________ (bit/bites/will bite) me I should die of fright.
3. If I __________ (had/ had had / have) more money, I could have bought my sister a better gift.
4. If it _________ (rains/rained/has rained/had rained) I would not have come to college.
5. She does not believe that I love her. If she will (give/gives/gave/had given) me an opportunity to prove
it, I will be able to convince her.

Fill in the blanks. Choices are given in brackets

iQue ideas Page 77


6. I don't have a pen. ________ (Shall/May/Will) I borrow one?
7. I have no money. _________ (May/Could/Shall) you lend me some?
8. You seem to be in trouble. ___________ (Shall/May/Will) I help you?
9. Dad is late. He ____________ (might/has/ought) to be here by now.
10. I heard you witnessed the blast. You ________ (might have been, must have been, shall have been)
terrified.

Fill in the blanks. More than one option may be right.

11. I miss ________ (live/to live/living) in Mumbai.


12. I enjoyed _______ (be/being/to be) in Mumbai again.
13. He promised ___________ (do/to do/doing) it next week.
14. Parag taught me _________ (to solve/solve/solving) the problem.
15. We saw them __________ (do/doing/to do) it.

Fill in the blanks. More than one option may be right.

16. I tried hard ________ (to convince/convincing) her, but she would not listen.
17. He stated _______ (smoking/to smoke) in the first year in college.
18. I don't remember ___________ (to turn/ turning) off the lights.
19. I forgot _________ (to turn/ turning) off the lights.
20. I can go on __________ (to work/working) like this.

Fill in the blanks. Only one option is correct.

21. The movie (can't, couldn't, shouldn't) be good. It is directed by a novice.


22. That (mightn't, can't, mustn't) be true. The director is quite talented.
23. I haven't decided, but I (must/might) watch it.
24. You (must/should) be joking.
25. Do you think you (can/could) buy a ticket for me?

Fill in the blanks.

26. The house _______ (is still being built/is still built/ is still building)
27. Houses _______ (are made/ are being made/ are making/ have been made) of concrete
28. The house ______ (was sold/has been sold/ had been sold) last week.
29. Do you thing the construction __________ (will be competed/will have been completed/ would be
completed) by next month?
30. It _______ (should be done/should have been done/had been done) last week.

Fill in the blanks

31. This is the village where I _____ (have lived/lived/had lived) as a child.
32. I ______ (didn't come/haven't come/ hadn't come) here since last year.
33. I ___________ (arrived/ have arrived/ had arrived) here a few hours ago.
34. I can't find my way around, because the place __________ (had changed/has changed/changed)
35. The last time I came here ________ (was/has been/had been) ages ago.

Fill in the blanks.

iQue ideas Page 78


36. When I ________ (had reached/reached/was reaching) the class, others had already left.
37. At the end of the class, everyone________ (has been/had been/was) completely bored.
38. They told me that they ________ (finished/had finished/either verb will make sense) their work long
time ago.
39. If I _________ (knew/had known/ either verb will make sense) the truth I would not hesitate to tell you.
40. I _______ (didn't think/ hadn't thought/ either verb will make sense) of it until you asked me.
41. In the following paragraph, check the underlined verbs for inconsistency in tense and correct them if
necessary.

Last night I have gone to Eros to see 'The Day After'. It is very thought-provoking, and it is very depressing.
I just wish it start a little earlier than 10 p.m. I find it hard to stay out until midnight and then got up for an
8:00 a.m. class.

Rewrite the following sentences after rectifying the errors in tense.

42. They had come to Mumbai in 1985. By 2010, they will be living in Mumbai for 25 years.
43. We were late for the class because there had been heavy traffic on the roads. By the time we reached the
class the lectured has already begun.
44. By next November, I will complete my graduation and take up a job, but you are still studying.
45. If he had not left the job, he will have completed ten years in this job by now.
46. Bhavin, who usually taught us Maths is absent this week. Hence we are taught by Parag and Amit this
week.

A sentence is given with a blank. In brackets at the end of the sentence you will find a verb and the intended
meaning. You need to fix the blank with a phrasal verb using the given verb.
Example: I ______ money and had to borrow from my friends. (run - intended meaning: exhaust) Answer: I
ran out of money and had to borrow from my friends.

47. The class_______ everything that is explained very fast. (pick - intended meaning: grasp)
48. The class was disappointed when the professor ______ the most difficult problem.(leave - intended
meaning omit)
49. I just cannot _____________ with my cousins.(get - intended meaning: be congenial)
50. He was unconscious for a while. When he ____________ his wallet was gone.(come)
51. Doctors advised him not to, but he ________ drinking. (keep - intended meaning: continue)
52. He must be really foolish to think that he can ________ with a lie like that to the police. (get - intended
meaning: escape)
53. Everyone says that I ________ my father. (take - intended meaning: resemble)
54. I do not know how his family _________ with so little money. (get - intended meaning: survive)
55. In order to avoid accompanying my friends I __________ a story about not being well. (make - intended
meaning: fabricate)
56. The marriage was ________ because of bereavement in the family. (put - intended meaning: postpone)

Answers and Explanations

1. She is only five feet tall. If she were a few inches taller, she could be working in Airlines. (Subjunctive is
required)
2. I hate dogs. if I a dog hit me I should die of fright. (If +past, would/should +verb)

iQue ideas Page 79


3. If I had had more money, I could have bought my sister a better gift. (if +past perfect, would/could have
+verb)
4. If it had rained, I would not have come to college. (if +past perfect, would/could have +verb)
5. She does not believe that I love her. If she will give me an opportunity to prove it, I will be able to
convince her.
6. I don't have a pen. May I borrow one?
7. I have no money. Could you lend me some?
8. You seem to be in trouble. Shall I help you?
9. Dad is late. He ought to be here by now.
10. I heard you witnessed the blast. You must have been terrified.
11. I miss living in Mumbai.
12. I enjoyed being in Mumbai again.
13. He promised to do it next week.
14. Parag taught me to solve the problem.
15. We saw them do it. We saw them doing it.
16. I tried hard to convince her, but she would not listen.
17. He stated smoking in the first year in college.
18. I don't remember turning off the lights.
19. I forgot to turn off the lights.
20. I can go on working like this.
21. The movie can't be good. It is directed by a novice.
22. That can't be true. The director is quite talented.
23. I haven't decided, but I might watch it.
24. You must be joking.
25. Do you think you could buy a ticket for me?
26. The house is still being built.
27. Houses are made of concrete.
28. The house was sold last week.
29. Do you thing the construction will be competed by next month?
30. It should have been done last week.
31. This is the village where lived as a child.
32. I haven't come here since last year.
33. I arrived here a few hours ago.
34. I can't find my way around: the place has changed.
35. The last time I came here was ages ago.
36. When I reached the class, other had already left.
37. At the end of the class, everyone was completely bored.
38. They told me that they finished or had finished their work long time ago. Answer: either verb will make
sense.
39. If I knew the truth I would not hesitate to tell you.
40. I hadn't thought of it until you asked me.
41. Last night I went to Eros to see 'The Day After'. It was very thought provoking, and it was very
depressing. I just wish it had started a little earlier than 10 p.m. I find it hard to stay out until midnight
and then get up for an 8:00 a.m. class.
42. They came to Mumbai in 1985. By 2010, they will have been living in Mumbai for 25 years. (will have
lived does not show the continuing action)
43. We were late for the class because there was heavy traffic on the roads. By the time we reached the class
the lectured had already begun.

iQue ideas Page 80


44. By next November, I will have completed my graduation and taken up a job, but you will still be
studying.
45. If he had not left the job, he would have completed ten years in this job by now.
46. Bhavin, who usually teaches us Maths is absent this week. Hence we are being taught by Parag and
Amit this week.
47. The class picks up everything that is explained very fast.
48. The class was disappointed when the professor left out the most difficult problems.
49. I just cannot get along with my cousins.
50. He was unconscious for a while. When he came to his wallet was gone.
51. Doctors advised him not to, but he kept on drinking.
52. He must be really foolish to think that he can get away with a lie like that to the police.
53. Everyone says that I take after my father.
54. I do not know how his family gets by with so little money.
55. In order to avoid accompanying my friends I made up a story about not being well.
56. The marriage was put off because of bereavement in the family.

iQue ideas Page 81


8
Prepositions
Prepositions are small words (like in, on etc.) that connect or join nouns or pronouns to the rest of the
sentence. They are somewhat without meaning in themselves, but indicate the time (after), place (above), or
manner (of) in which something else is. When we see a preposition we can expect a noun or pronoun
immediately after it, or a couple of words later. This noun that a preposition is so closely related to is called
its object. Simply, a preposition always needs an object.

Function of a preposition

Prepositions in themselves are rather innocuous; they get their function when they combine with other words
— we call these combinations prepositional phrases (for the love of money - for and of are prepositions).
Because of this, there can be an infinite variety of prepositional phrases. The structure of prepositional
phrases, however, is very simple.

For example, the structure of the prepositional phrase, "in the classroom", is 'in + the + classroom, i.e.
preposition + determiner + noun. If wechange it to "in the large classroom", the formula is: preposition +
determiner + adjective + noun. Most prepositional phrases follow this pattern: preposition + determiner + an
adjective or two + pronoun or noun (called the object of the preposition). The whole combination acts as
adjectives and adverbs.

Usage issues related prepositions are extremely complex for the following reasons:
 Prepositions have several meanings.
 They can be used as adverbs and adjectives.
 they go into innumerable idioms.
 they are resolutely paired with many adjectives, nouns, and verbs.
 there are (tens of ?) thousands of phrasal verbs(preposition + another word)

Hence the possibilities of misuse are also vast. Combined with all these is our inability to even identify
prepositions. Students take time even to give an example for a preposition.

(Did you notice the error in the last sentence? It should be "an example of and not "an example for")

Given below is a list of prepositions. You need not learn this list by heart. This is to enable you to recognize
a preposition when you see it next.

ABOUT, ABOVE, ACROSS, AFTER*, AGAINST, AMONG, AROUND, AT, BEFORE*, BEHIND,
BELOW, BENEATH, BESIDE, BESIDES, BETWEEN, BEYOND, BY, DOWN, DURING, EXCEPT,
FOR, FROM, IN, INSIDE, INTO, LIKE, NEAR, OF, OFF, ON, OUT, OUTSIDE, OVER, SINCE,
THROUGH, THROUGHOUT, TO, TOWARD, UNDER, UNTIL*, UP, UPON, WITH, WITHOUT,
ACCORDING TO, BECAUSE OF, BY WAY OF, IN ADDITION TO, IN FRONT OF, IN PLACE OF, IN
REGARD TO, [N SPITE OF, INSTEAD OF, ON ACCOUNT OF, OUT OF ....etc.

* These prepositions can also act as conjunctions

iQue ideas Page 82


In, On, At

The basic function of IN, ON, AT is to indicate either the place/locationor Time of something or some
event.

Whatever notes is given below, though true, cannot be made applicable to the entire language. These are
broad guidelines, which will hold true to a large extent. You are likely to come across exceptions to what is
said here. Nevertheless, these are important guidelines.

In, On, At — As Prepositions of Place or location.

Study the different uses of in, on. at to indicate place or location. These prepositions are used differently to
indicate place or location.

In On At
In indicates a location inside areas On indicates contact with a line or At indicates a point on the map,
We live in Mumbai. surface without dimensions: We stopped
We live on Linking Road. at Mumbai for a while.
Our house is on the beach.
In indicates location inside a On is used for animals used for At is used to indicate that the
three-dimensional space transportation person is using the thing
We are in the classroom The child sat on the horse. mentioned
We are at our desks.
Someone is at the door.
(knocking)
In is used with vehicles that do On is used with vehicles that
not permit much freedom of permit freedom of movement
movement (walking, standing, etc.):
We are in the car/van. We are on the
bus/train/ship/plane

Study the following sentences to see the above relationship.

IN
 In is used to locate the existence of something/an event inside a specific geographic area: in Mumbai, in
India, in Maharashtra, in Europe, in the South, in the Himalayas, in the garden, in the region etc.
 In locates something or some event inside of a well defined three dimensional space, or inside of a space
that is considered to be three dimensional: in the room, in the house, in the cupboard, in the car, in the
pocket, in the desk, in the book, in sentence 5, in the lake etc.
 When in is used with vehicles, these vehicles do not generally allow any freedom of movement within
them: in a car, in a cab, in an elevator, in a taxi.

ON
 On locates something on a surface directly or around its borders considered as a line or very close to the
perceived boundary or line: on the floor, on the table, on the chair, on S.V. Road, on Princess Street, on
the edge, on the side, on the left, on the right, on the river, on the beach, etc....
 With reference to animals that are generally used for transportation the preposition on is used: on a
camel, on a donkey, on an elephant, on a horse.
 When on is used with vehicles it implies that the vehicle allows a person to move around on that vehicle
or has the freedom to stand, walk within its confines: on a ship, on a bike, on a plane, on a train, on a
bus.

iQue ideas Page 83


AT
 The use of at as a preposition for locations would imply that that location is considered not for its
functions or purposes but merely as a geographical entity, as a general place: at the hospital, at the
restaurant, at the school. at the bank, at Mumbai. E.g one cannot work at Mumbai or at a school, but in
Mumbai, and in a school.
 When a person is near something or using that thing: at the desk (working) at the door (ringing the bell),
and at the window etc.

Now if we compare "She is in the hospital' and 'She is at the hospital' we can understand the difference in
meaning. In the hospital would imply a location in the three dimensional space of the hospital. Hence the
meaning of the sentence is that she is admitted for some ailment. But if we say at the hospital it merely refers
to the hospital as a reference point in space hence the meaning is that she is there for some purpose.

You can generate such examples and make the concept clearer in your mind. Think of examples like, station
and arrived in the house. The same way when we say. 'on the field' we are referring to the field as a surface.

Hence we would say that the players are on the filed. But if the identity of these persons are not players but
merely, say, girls, we would say the girls are in the .field (inside a three dimensional space).

How the noun is perceived by the speaker is also very important for making the correct choice of
prepositions. Also, when you are reading good English, notice the use of prepositions.

In, On, At — Prepositions of Time.

In On At
Well defined time periods take in Days On Monday, On 17 Nov. With specific time: at 10 am, at
— in the 20th century, in the early the stroke of midnight.
1990s, in his youth..
Used in phrases like: in the first On time means 'punctual:. With nouns like: at dusk, at dawn,
few hours..., in the — last few at night, at breakfast
minutes...
in time means 'sufficiently before At is used with ages: at the age of
time.' sixty , at eighteen.

More Examples:
AT
 At refers to a specific time on the clock: at 10 o'clock, at ten to four, at midnight
 At is used in several general reference to time in phrases like: at night, at dawn, at dusk, at bedtime, at
suppertime, at dinnertime, at breakfast, at lunch, at supper, at dinner, at that time, at the time, at the
moment, at present, at that minute, at that hour, at the (beginning, start, end) of the 21" century, at once
 To refer to the age of a person the preposition at is used: at the age of forty, at fifteen.

ON
 The preposition on is firmly attached to Days: on Monday, on Sunday morning, on Saturday afternoon,
on Friday night, on the weekend, on January 18, on the 12th of January, on Wednesday, Jan 12. on my
birthday, on their anniversary, on the (morning, afternoon, evening, night) of September 11, on the
following day

IN

iQue ideas Page 84


 Periods of time are usually expressed using the preposition in such as centuries, years, seasons, months,
and parts of the day: in the nineteenth century, in 1986, in the fifties, in the (early, late) 1980s, in the
(morning, afternoon, evening), in her childhood, in his youth, in the prime of her life, in his old age,
 Also: in the first three months of the year, in the second quarter, in the last half, in the last ten seconds of
the game, in the last few moments of his life, in the time remaining, in the next few months,
 Do not use on/in with next and last.
 For example write: Next Friday and last November. It would be incorrect to say on the next Friday or in
the last November. In the next few days, in the last few hours are correct.

No Preposition
Certain references to location or space will not use a preposition in certain contexts. The following are
examples: downstairs, downtown, inside, outside, upstairs, uptown

He went upstairs.
He went home.
He went outside.

Prepositions as part of idioms.

Another important thing to bear in mind about prepositions is their idiomatic use. There is an endless list of
idioms in English that are propositional phrases. Their meaning cannot be explained through grammatical or
logical reasoning — it is a matter of usage.

For example: My mother looks after me. (tolook after means to take care of)

One has to be aware of the idiom and its meaning; there is no way one can guess or approximate its meaning.
There are innumerable idioms in English created using prepositions.

Prepositions in Parallel Form.

In parallel structures (a later chapter is on parallelism), do not repeat the preposition if both the ideas require
the same preposition.

Incorrect: Parallelism means the use of identical syntactic constructions inclauses or inphrases.
Correct: Parallelism means the use of identical syntactic constructions inclauses or phrases.

However, when the phrases need different prepositions, care should be taken to use the different prepositions
required.

Incorrect: The children were at once fascinated and frightened of the lion in the cage.
Correct: The children were at once fascinated by and frightened of the lion in the cage.

Prepositions of Movement: TO

The preposition to is used in order to indicate a movement toward something.

They go to the college together.


They are going to the Principal's office.

iQue ideas Page 85


Toward OR Towards?

Toward and towards are merely variant spellings of the same preposition. There is no difference in meaning
or implication.
This preposition indicates, in the direction of and several other similar meanings.

She was walking toward the door.

The life of his body, and of every fibre of his body, the life that was the very substance of his body and that
was apart from his own personal life, had yearned toward this light and urged his body toward it in the same
way that the cunning chemistry of a plant urges it toward the sun. - White Fang by London, Jack

Above AND Below


The preposition above is used with several meanings and below in most contexts provides its opposite
meaning.

 at a higher or lower height than another thing. (above or below the surface of the earth)
 in a higher or lower position of authority. (above or below the rank of a manager)
 higher or lower in amount or degree (above or below the cut-off score)
 higher or lower in temperatures (above or below the room temperature)
 above all means more than anything else. (freedom above all — below cannot be used)
 mentioned earlier or later in a piece of writing.(None of the above)

Above also contributes to idioms in several ways: above suspicion, above the law, above reproach etc.

And when I lived with them, then did I live above them.
 Thus Spake Zarathustra: A Book For All And None by Nietzsche, Friedrich

Over AND Under


Over is used with several meanings:
 more than : over ten years' experience.
 during: over the (last ten) years.
 while having a meal or a drink : over dinner/a drink
 spanning, crossing, or moving across: bridge over the river
 land, mountains, (esp. while travelling): over the rough terrain
 covering: veil over her face
 over or all over can mean widespread: all over the word
 over and over means again and again, many times, repeatedly

Under in several cases is the opposite of over:

 less than: under 18


 below the surface: under water.
 according to the provisions of: under the Indian penal Code
 a category or heading: under Reserves and Surplus in a balance sheet
 affected by: under the influence of alcohol.

For AND Since - Prepositions of Time.

iQue ideas Page 86


While referring to time the prepositions for and since follow definite patterns. For has to be followed by a
period (a span of time)
For example, 10 minutes, half an hour, one day, 24 hours, 1 week, centuries, ages are all words that denote
periods. For is used before all such words.

Incorrect: I have been calling you since two weeks.


Incorrect: Liberalisation has been going on since two decades.

Correct: I have been calling you for two weeks.


Correct: Liberalisation has been going on for two decades.

Since is used before a particular moment in time — which is specific. 9 o'clock, 1st January, Monday,
morning etc.are all examples of specific moments. Since will denote from this specific moment in the past
till now. That's why we will see that sentences with since are in the perfect tense. For can be used with any
tense.

I have been waiting here since 9 a.m.


She has changed a lot since she got married.
The universe had not changed since the beginning of time.

Note: Do not use any other tense with since than the perfect. But you will definitely see sentences with since,
in them but are not in the perfect tenses like:Since she is not interested, you should not insist. Here since is
used as a conjunction and not a preposition. If used as a preposition any other tense then the perfect tenses is
wrong.

Incorrect: I am waiting here since 9o'clock,


Incorrect: She changed a lot since last year.
Correct: I have been waiting here since 9o'clock,
Correct: She has changed a lot since last year.

Since can be used to refer to a period (like for) when the sentence begins with It is.. . or is a question with is
it… However, the perfect tense would still be right in these sentences.

Correct: It is a month since we met.


Correct: It has been long time since we met.

Correct: How many years is it since she got married?


Correct: Since how many years has she been married?

Between AND Among


Use between when referring to two.

There was an argument between Amit and Sanjaya.

When three or more are involved we have a choice. We can use between or among depending on the
relationship that we ant to communicate.

We use between to denote a one-to-one relationship, regardless of the number of items.

iQue ideas Page 87


The partition happened between East Pakistan, India, and Pakistan.
Globalisation encourages economic cooperation between nations.

With three or more items among is more appropriate if the emphasis is on distribution rather than individual
relationships.

Industrialisation has caused discontent among the farmers.

Among AND Amongst

These are merely spelling variants of the same preposition. Both are correct and mean the same, but among
is more common.

Round AND Around


Round and Around are interchangeable in most contexts, though at times one or the other may not 'sound'
good to you.

The garden is in bloom around the year.


The garden is in bloom round the year.

Both are correct sentences. The same way, 'please show me round the town' is just as good as around the
town.

In American English around is preferred and in British English round is more common.

However, there are a few limiting circumstances. When you approximate, use around and not round. E.g.
Around 40% are against the motion.

Despite AND In spite of


Despite and in spite of are interchangeable in all contexts. Take care to use despite without the of

He played despite the injury.


He played in spite of the injury.

Till AND Until.


Till and until are interchangeable in all contexts. Until is preferred at the beginning of sentences.

She promised to stay till/until Friday.


You have to remain seated till/until the bell rings.
Until the bill rings, you are not allowed to leave the examination hall.

Beside AND Besides


Beside and besides are two different prepositions and should not be confused.

Beside: by the side of - Walk beside me.


irrelevant — What you said is beside the point.

Be beside oneself to lose self control — She was beside herself with grief.
Besides: other than /except — No one besides Parag could find a solution to the problem.

iQue ideas Page 88


In AND into
Generally, we use into for movement, but not always. In is explained earlier in this chapter.

Into is correct only when the verb shows some kind of movement.

The paper flew into the room.


He walked into the canteen.

Sometimes in and into are used interchangeably and mean the same thing:

He threw the paper into the dustbin OR


He threw the paper in the dustbin.

Sometimes in and into mean different things in the same sentence:

He walked into the park. (entered the park from outside)


He walked in the park (was walking inside the park)

In idiomatic expression they are not interchangeable.

He walks in his sleep. (notinto)


Write in alphabetical order (not into)
He dives into the pool (not in)

Ending the sentence with a preposition.

One of the controversial rules about prepositions is that we should not end a sentence with a preposition. For
example, how much did you buy it for? should be improved to For much did you buy it?

However, this rule is more often broken than kept — and is no more a rule. Winston Churchill rather
exasperated with this kind of strictures responded with his famous: That is nonsense up with which I shall
not put," rather than with — That is nonsense that I shall not put up with, which is elegant.

Ignore this rule, if ending with a preposition does not make the sentence awkward.

Phrasal Verbs

Refer to the chapter on Verbs. A phrasal verb is a verb plus a preposition or adverb which creates a meaning
different from the original verb.

For example: My mother looks after me. (look after is a phrasal verb which means to take care of) There are
different types of phrasal verbs:

 Intransitive Phrasal verbs: They do not take objects.

She tried to get up, but could not. (get up cannot take an object and is complete in itself)

 Transitive phrasal verbs: They like transitive verbs are followed by an object.

iQue ideas Page 89


I went through the questions and found them easy. (went through is followed by the questions which is the
object of went through)

 Separable phrasal verbs: In these phrasal verbs the verb and its particle(s) are separable and something
else can be placed between them.

She left out the third question.


She left the third question out. She left it out. (left and out can be separated)

 Inseparable phrasal Verbs:

He takes after his father (take after which means resemble cannot be separated)

Given below are examples of phrasal verbs according to the above grouping, and their common meanings:

Intransitive phrasal verbs: break down - stop functioning; catch on — to become popular; come to regain
consciousness; come round — to change direction also to return to an earlier condition: drop by - visit
without appointment; get by — to survive; pass out lose consciousness, faint; show up – arrive

Transitive and inseparable phrasal verbs: call on - visit; get over - recover from sickness; go over - review;
go through — read/use up or consume; look into - investigate; run across - find by chance; run into - meet;
take after -resemble; wait on- serve

Separable phrasal verbs: bring up- raise children / mention a topic; call off- cancel; do over - repeat a job;
hand in - submit something; hang up - put something on a hook or receiver; hold up - delay ; hold up - rob ;
look over -examine, check ; look up - search in a list; make up - invent; make out - hear, understand; pick out
- choose; pick up - lift something; put away - save or store; put out - extinguish; turn down - lower volume;
turn up - raise the volume; turn off- switch off electricity; turn off- repulse; turn on - switch on the
electricity.

Three word phrasal verbs. These cannot be separated.

catch up with — keep abreast — I was lagging behind; but I decided to catch up them by putting in extra
hours. come up with— to contribute — The student body came up with several suggestions as well as
donations to help the victims of the flood.
look down on — despise — I don't know why our neighbours look down on us.
look in on— visit a person — On my way to college I decided to look in on my friend who was ill.
zone in on— pay attention after not doing so — I zoned in on what they were discussing when I heard them
mention my name.
read up on—research — He has been reading up on America ever since he decided to study there.
keep up at— continue, not quit — Learning any language is difficult, but you have to keep up at it till you
succeed. get along in—progress — I hope you are getting along in the current job.
cut out on— let down, snub — Expect him to cut out on his friends if it comes to money.
break in on — to interrupt a conversation — The panel broke in on our discussion with his comments during
the group discussion.
check up on— examine, investigate — The professor promised to check up on the progress of the student
from time to time.
look forward to— anticipate with pleasure — I am eagerly looking forward to meeting my parents in the
vacation.

iQue ideas Page 90


cut down on — curtail — He decided to cut down on his smoking.
put up with — tolerate — We have to put up with a great deal of nonsense from our neighbours
walk in on— enter somewhere unexpectedly — He walked in on the officer whom he wanted to catch red
handed.
muscle in on — impose oneself upon — We did not like him to be on the team, but he managed to muscle in
on the group at the last minute.
hang back from— delay or avoid doing something — He is just hanging back from telling us the truth.
face up to— accept an unpleasant truth — He could not face up to the fact that he was not needed in the
team.
chip away at— gradually make something less effective — They have been chipping away at his reputation
ever since he joined the firm.
get through with — finish — It looks as if he would never get through with his project.
zero in on — close in on — focus attention on — After the interrogation of the suspects the police have
zeroed in on the culprit.
talk down to— talk in a way to show your superiority — Some teachers talk down to their students.
get by with — sufficient for the purpose — I think we can get by with one car; but two would be better.
drill down through - get to the bottom of something — They drilled down through the evidence to identify
the murderer.

Review Exercise

Review Exercise Fill in the blanks with in, on or at. If no preposition is needed, write 0 in the blank.

1. We plan to go out for dinner ______ the weekend.


2. When the kids are _______ bed, we can listen to music _________ the radio.
3. She watches movies _______ the theatre ________ every Saturday.
4. He used to sell newspapers ________ Linking Road ________ the summer holidays.
5. We stayed back _______ the hospital ___________ last night so that the patient ______ the hospital
did not feel left alone.
6. She is not ________ the hostel. She went _______ home _________ this evening.
7. He met Amitabh ______ a party _______ Sunday night.
8. She came to my house ________ her bike ________ the evening.
9. He promised to call ________ two weeks.
10. He went ________ cycling to Delhi ______ 2007.

Choose the correct option

11. I could not enter ________ the door so I was forced to climb ________ a window.
(1) through / in (2) between / into (3) out of / between (4) out of / into (5) up / along

12. As there was a fallen tree ________ the path, I had to walk _______ it to go to the other side.
(1) out of / in (2) under / around (3) out of / into (4) along / along (5)across / around

13. She ran________ the path and __________ the steps and her house.
(1) from / into / into (4) by / on / into
(2) along / towards / into (5) in / next to / near
(3) along / up / into

14. To get to the Manager's cabin please go _______ those stairs and ______ the corridor to the end.

iQue ideas Page 91


(1) over / into (2) between / into (3) out of / between (4) out of / into (5) up / along

15. My friend had left school ________ the age of 14 and gone to work ________ a restaurant.
(1) under / in (2) about / in (3) under / around (4) on / in (5) at / in

16. The topper is the person standing ________ the Principal, ________ the guard in uniform.
(1) at / into (2) beside / next to (3) under / around (4) in / next to (5) at / in

17. He put the key _______ the lock after taking it ________ her bag.
(1) into / out of (2) into / from (3) in / out of (4) by / out of (5) up to / out of

18. She came _______ me and asked the best way to get _______ the situation.
(1) from / into (2) towards / over (3) along / up (4) by / on (5) up to / out of

19. He walked _______ the park after climbing _______ the wall.
(1) into / over (2) towards / over (3) along / up (4) by / on (5) up to / out of

20. The thief jumped _______ a waiting car after jumping _________ the window.
(1) into / over (2) between / over (3)out of / out of (4) into / out of (5) up to / out of

21. She walked _________ me without paying attention and walked ________ the door.
(1) from / into (2) towards / over (3) along / up (4) past / towards (5) in / next to

22. She was driving ______ 70 or 80 kmph when she crashed ________ the divider.
(1) at / into (2) out of / in (3) under / around (4) in / next to (5) at / in

23. I put the CD ________ the computer after taking it _________ its cover.
(1) Into out of / in (4) out of / into
(2) out of / in (5) in / out of
(3) inside / outside

24. The shop is ________ Crawford market, ___________ Metro Cinema.


(1) at / in (2) near / of (3) under / around (4) in / next to (5) up / along

25. There was discussion _________ it ________the television _________the 8o'clok news.
(1) about / on (2) on / on (3) of / in (4) on / at (5) with / about

Fill in the blanks with the most appropriate preposition.

26. I got a seat ________ the back of the theatre. ( of, at, in)
27. She took her lessons _________ the teacher ( by, from, except)
28. I have received all of the notes _________ this one. ( between, but, among)
29. ________ the risks involved, he decided to invest in the plan. (except, despite, In spite)
30. When it is heated ice changes __________ water. (in, into, until)
31. We have been waiting _________ several days. (for, since, in)
32. We have been waiting __________ 10 o' clock in the morning. (for, since, in)
33. They will be going to Delhi _________ two days.( since, for, on)
34. I have known all of you_________ a year now. ( since, for , besides)
35. Oranges contain riboflavin ________ vitamin C. (beside, besides, among)

iQue ideas Page 92


36. He has not attended lectures _______ Friday. (since, for, beside)
37. He met his friend ________ the corner of the two main roads. (in, on, at)
38. India is independent ________ 1947 ( for, in, since)
39. We waited __________ about an hour. (for, since, in)
40. There are many other ways to solve this problem _________ this one. (beside, besides, in)

Fill in the blank with the most appropriate phrasal verb.

41. The trip was enjoyable because all of us _______ very well. (get through/get on/ get up)
42. I had to seek help from my brother because I had _______ money.( run out of / run out / run off)
43. The match was _______ because of rain. (blacked out / turned off / called off)
44. How did the captain _________ the poor performance of the team. (come to / account for/ jot down)
45. When he _________ telling stories about the war, there's no stopping him. (gets on / gets off / gets to)
46. I celebrated my 50th birthday last year. I am really in years. (get through/ getting on / get up)
47. The movie ended with everything _________ all right. ( turning out / wearing out / climbing up)
48. I was late for the lecture because I was ________ in traffic. ( lifted off / held down / caught up)
49. When I found that nobody would listen to me in the meeting, I finally __________ in protest. (gave up /
walked out / ran over)
50. A lot of people __________ when they heard that a film star was around. ( turned out / set out / crowded
around )
51. The noise they make during certain festivals really _________ ( jumps me / gets to me / does me in)
52. I have to _______ for your regular teacher today because she is absent. ( fill out / fill in / fill up)
53. The couple have decided to _______ because he was not earning enough. ( carry on / split up / call off)
54. I felt completely _________ when my favourite batsman failed to score. ( let up / let down / let in )
55. My plan to buy a new computer _________ after I talked about it with my father.( tried out / came about
/ made out )
56. It's very easy to _________ her for her sister; they both look alike. ( choose out / mix up / pick down)
57. His scores have shown a _________ in the last two tests. (get down / fall off / stand down )
58. The Principal's explanations of the reasons for the fee hike did not ______ . ( take in / add up / open out)
59. "He promised that he will ________ my money tomorrow." (make back / go back / hand back)
60. I went to the library to _________ a book. (look up / look for / look out)

Answer and Explanations

1. We plan to go out for dinner on the weekend.


2. When the kids are in bed, we can listen to music on the radio.
3. She watches movies at the theatre __0__ every Saturday.
4. He used to sell newspapers on Linking Road in the summer holidays.
5. We stayed back at the hospital __0__ last night so that the patient in the hospital did not feel left alone.
6. She is not in the hostel. She went __0__ home __0__ this evening.
7. He met Abhishek at a party on Sunday night.
8. She came to my house on her bike in the evening.
9. He promised to call in two weeks.
10. He went cycling to Delhi in 2007.
11. I could not enter through the door so I was forced to climb in a window. (a) through / in
12. As there was a fallen tree across the path, I had to walk around it to go to the other side. (e) across /
around
13. She ran along the path and up the steps and into her house. (c) along / up / into
14. To get to the Manager's cabin please go up those stairs and along the corridor to the end. (e) up / along

iQue ideas Page 93


15. My friend had left school at the age of 14 and gone to work in a restaurant. (e) at / in
16. The topper is the person standing beside the Principal, next to the guard in uniform. (b) beside / next to
17. She put the key in the lock after taking it out of her bag. (c) in / out of
18. She came up to me and asked the best way to get out of the situation. (e) up to / out of
19. He walked into the park after climbing over the wall. (a) into / over
20. The thief jumped into a waiting car after jumping out of the window. (d) into / out of
21. She walked past me without paying attention and walked towards the door. (d) past / towards
22. She was driving at 70 or 80 kmph when she crashed into the divider. (a) at / into
23. I put the CD in the computer after taking it out of its cover. (e) in / out of
24. The shop is in Crawford market, next to Metro Cinema. (d) in / next to
25. There was discussion about it on the television on the 8o'clok news. (a) about / on / on
26. I got a seat at the back of the theatre. ( of, at, in)
27. She took her lessons from the teacher ( by, from, except)
28. I have received all of the notes but this one. ( between, but, among)
29. Despite the risks involved, he decided to invest in the plan. (except, despite, In spite)
30. When it is heated ice changes into water. (in, into, until)
31. We have been waiting for several days. (for, since, in)
32. We have been waiting since 10 o' clock in the morning. (for, since, in)
33. They will be going to Delhi for two days. ( since, for, on)
34. I have known all of you for a year now. ( since, for , besides)
35. Oranges contain riboflavin besides vitamin C. (beside, besides, among)
36. He has not attended lectures since Friday. (since, for, beside)
37. He met his friend at the corner of the two main roads. (in, on, at)
38. India is independent since 1947 ( for, in, since)
39. We waited for about an hour. (for, since, in)
40. There are many other ways to solve this problem besides this one. ( beside, besides, in)
41. The trip was enjoyable because all of us ______ very well. (get through/get on/ get up)
42. I had to seek help from my brother because I had ______ money.(run out of / run out / run off)
43. The match was _______ because of rain. (blacked out / turned off / called off)
44. How did the captain ________ the poor performance of the team. (come to / account for/ jot down)
45. When he _________ telling stories about the war, there's no stopping him. (gets on / gets off / gets to)
46. I celebrated my 50th birthday last year. I am really in years. (get through/ getting on / get up)
47. The movie ended with everything ________ all right. ( turning out / wearing out / climbing up)
48. I was late for the lecture because I was _____ in traffic. ( lifted off / held down / caught up)
49. When I found that nobody would listen to me in the meeting, I finally _____ in protest. (gave up /
walked out / ran over)
50. A lot of people _____ when they heard that a film star was around. ( turned out / set out / crowded
around )
51. The noise they make during certain festivals really ________ ( jumps me / gets to me / does me in)
52. I have to ________ for your regular teacher today because she is absent. ( fill out / fill in / till up)
53. The couple have decided to _________ because he was not earning enough. ( carry on / split up / call
oft)
54. I felt completely _____ when my favourite batsman failed to score. ( let up / let down / let in )
55. My plan to buy a new computer _______ after I talked about it with my father.( tried out / came about /
made out )
56. It's very easy to ________ her for her sister; they both look alike. ( choose out / mix up / pick down)
57. His scores have shown a ________ in the last two tests. (get down / fall off / stand down )
58. The Principal's explanations of the reasons for the fee hike did not_______ . ( take in / add up / open
out)

iQue ideas Page 94


59. "He promised that he will ________ my money tomorrow." (make back / go back / hand back)
60. I went to the library to ______ a book. (look up / look for / look out)

iQue ideas Page 95


9
Conjunctions
A conjunction is a joiner. It connects parts of a sentence. Conjunctions are categorized as Coordinating
conjunctions, Subordinating conjunctions, and Correlative conjunctions depending on the function they
perform in a sentence. Among the coordinating conjunctions, the most common are and, but, and or.
Coordinating Conjunctions: FOR, AND, NOR, BUT, OR, YET, SO.
Correlative conjunctions: BOTH...AND, NOT ONLY...BUT ALSO, EITHER...OR, NEITHER...NOR,
WHETHER...OR.

Subordinating Conjunctions:

Time Cause/Effect Contrast Condition


AFTER BECAUSE ALTHOUGH IF
BEFORE SINCE THOUGH UNLESS
WHEN NOW THAT EVEN THOUGH ONLY IF
WHILE AS WHEREAS WHETHER OR NOT
SINCE IN ORDER THAT WHILE EVEN IF
UNTIL SO IN CASE (THAT)

Coordinating Conjunctions
Coordinating Conjunctions are: and, but, or, yet, for, nor, so (They can be easily memorized through the
acronym FANBOYS: For-And-Nor-But-Or-Yet-So. Please note that then and now are not conjunctions.
When two sentences are joined using a coordinating conjunction a comma before the and in the second
clause is always right, though the comma is not used at times.

Alpesh wants to take the CAT, but he does not have 50% at his graduation.
Alpesh wants to take the CAT but he hasn't been careful with his academics.

Both the above sentences are correct. A comma is always right when two sentences (clauses) are joined with
a coordinating conjunction.
A comma is used with but when expressing a contrast:

This is a useful law, but difficult to implement.

The Oxford Comma or the Serial Comma.


In the list x, y, and z, the second comma after y (before and) is called the Oxford Comma or the serial
comma. It is advisable to use this comma though you rarely find journalists using it.

Incorrect:Alpesh spent months studying English, Basic Maths and Reading Comprehension
Correct: Alpesh spent months studying English, Basic Maths, and Reading Comprehension.

We will look at this comma, which is called the serial comma or the Oxford comma, under punctuation
again.

X, Y, and Z AND X and Y and Z.

iQue ideas Page 96


Both X Y, and Z and X and Y and Z are correct. When a coordinating conjunction is used to connect all the
elements in a series, a comma is not used:

Engineers and Commerce Graduates and Management Graduates and Arts Graduates comprise the student
community at IIM Ahmedabad.

Though a comma before the and is always right when two sentences are joined, it is not used when two items
are joined.
Gandhiji and Nehru were contemporaries.
Gandhiji is respected for his adherence to nonviolence and truth.
It is hard to say whether DI or Verbal is more time-consuming.

Beginning a Sentence with 'And' or 'But'

We can begin sentences with and orbut. The belief that a coordinating conjunction should not be used to
begin a sentence does not hold water.
Andand but are used at the beginning of sentences as transitional devices. A transitional device is a word that
facilitates the smooth flow of one sentence into another. Such uses of and and but are perfectly legal. Hence,
not all sentences beginning with and or but are wrong. Some of them are good sentences. However, see if
that sentence can be joined to the earlier sentence or if the sentence is better without the and. Make your
decision accordingly.

And
The use of and in all these contexts is correct:

 When one idea is chronologically sequential to another:


Shilpa sent in her application and waited eagerly for a response.

 When one idea is the result of another:


The students heard the professor's footsteps and promptly stopped talking.

 When one idea is in contrast to another (frequently replaced by but in this usage):
Anita is academically brilliant and Sunita is creative in her thinking.

 When there is an element of surprise (sometimes replaced by yet in this usage):


Mumbai is a rich city and suffers from many symptoms of urban blight.

 When one clause is dependent upon another, conditionally:


Keep drinking like this, and you will find yourself in the hospital.

 When it is a kind of 'comment' on the first clause:


Ashish became addicted to alcohol — and that surprised no one who knew him.

But
The conjunction but is used:

 When the second clause is a contrast to the first:


Dhruv works hard, but he is still not able score well in the tests.

iQue ideas Page 97


 In the sense of on the contrary:
Dhruv did not study by himself, but had joined the best tuition possible.

 In the sense of 'with the exception of ' :


Everyone but Dhruv was able to achieve the cut-off score in verbal.

Or
 When only one option is available:
You can study hard for this exam or you can fail.

 To explain the previous part:


NWC is the premier women's college in the country, or so it seems to most NWC alumnae.

 When the first part may not be authentic:


CAT is not a difficult test, or so our mentor tells us.

Nor
The most common use of nor is as part of the paired conjunction neither ... nor. It can be used with other
negative expressions as well..

Her answers are always neither here nor there.


She didn't say anything, nor did the others.

Yet
Yet functions as a coordinating conjunction meaning something like 'nevertheless' or 'but'. The wordet seems
to communicate something distinctively different from but.
Kapil plays basketball well, yet his favourite sport is badminton.

The above sentence would still be correct without the comma after well. Yet is sometimes combined with
other conjunctions, but ,and or still

Not perfect but yet still beautiful.


This usage is correct.

For
As far as possible do not begin a sentence with for. For has serious sequential implications hence, you will
have to look carefully at the sentences beginning with for. In this respect, it is different from the use of
because and since at the beginning of sentences.

So
Be careful of the conjunction so. Sometimes, it can connect two independent clauses along with a comma,
but sometimes it can't. For instance, in this sentence,
Kareena is not the only actor in her family, so are her brother, sister, and her uncle Kapoor.

In the above, the word so means 'as well' or 'in addition'. However, a semicolon between the two independent
clauses would be smoother. But the above sentence is correct.
In the following sentence, where so is acting like 'therefore', the conjunction and the comma are adequate:
Rachana has always liked Dhruv, so it is no surprise that she is vivacious in his company.

iQue ideas Page 98


Students are sometimes confused about the use of so and so that and tend to accept only so that to introduce
a clause that states the reason for an action. Both so and so that are equally acceptable in this situation:
He stayed up late, so/so that he could complete his project and submit it the next day.

So as in intensifier to replace very is also looked down upon by certain grammarians. However, so
communicates a different meaning from that of very by the inclusion of the speaker's sympathetic assessment
of the situation. Hence It was very unfair that the professor scolded you in front of the class, and, It was so
unfair that the professor scolded you in front of the class have different meanings. The second sentence
(though correct) is better reserved for informal contexts and can be rejected in competitive examinations.

Than
Than is not a coordinating conjunction. Than is a conjunction used to make comparisons. The use of than as
a preposition is best avoided in competitive examinations.

Incorrect: He's taller than me.


Correct: He's taller than I [am]
Than as a conjunction is used to join two clauses as in the above correct sentence.

Then
Then is not a coordinating conjunction.
Many students think that then is used in the same way as but or and.
Incorrect: He lectured for two hours, then he turned his attention to the homework.
Correct: He lectured for two hours, and he turned his attention to the homework.
Then is a conjunctive adverb (See under Adjectives and Adverbs However, nevertheless, therefore...
etc.The difference between a conjunction and a conjunctive adverb is that a conjunctive adverb does not
have a fixed place like a conjunction n a sentence. In the above sentence then can be moved around quite a
bit. E.g. then he turned his attention ... he then turned his attention, he turned his attention to the homework
then. If we use and (in place of then) it is quite fixed in that place.
"He lectured for two hours, then he turned his attention to the homework."— is a comma splice. Comma
splice is a faulty sentence construction in which a comma tries to do the work of a conjunction. The comma
needs a coordinating conjunction to make it work, and the word then simply cannot hold two clauses
together without a conjunction.
Correct: He lectured for two hours; then, he turned his attention to the homework.

Correlative Conjunctions
Correlative conjunctions are:
both . . . and; not only . . . but also; not . . . but; either . . . or; neither . . . nor; whether . or; as . . . as .
The correlative conjunctions join sentence parts that have equal importance.
She topped the class not only in academics but also in enthusiasm.
Whether she makes it to the top B-Schools or loses is immaterial because she will anyhow succeed in life.
(See Parallelism)

Subordinating Conjunctions
Common Subordinating Conjunctions are: After, although, as, as if, as long as, as though, because, before,
even if even though, if if only, in order that, now that, once, rather than, since, so that, than, that, though, till,
unless, until, when, whenever, where, whereas, wherever, while...

When there are two ideas constructed with two clauses the subordinating conjunction makes one idea
subordinate to the other. The sentence will then have a main clause and a Subordinate Clause (Dependent

iQue ideas Page 99


Clause) introduced by one of the subordinating conjunctions. Subordinate clause will then depend on the rest
of the sentence for its meaning and significance.
Because he loved her, he refused to die without meeting her.

Examples will be classified under headings, though these are not quite mutually exclusive.
Notice that some of the subordinating conjunctions in the list above — after, before, since,etc — are also
prepositions, but as subordinators, they are used to introduce a subordinate clause.

Like and As
The usage issues relate to these two small words can sometimes tie in you knots. Though the issue is not as
straightforward as treating Like as preposition and Asas conjunction, from the point of view of competitive
exams we will go by that thinking:
Strictly, the word like is a preposition and not a conjunction,
[Correct: My brother collects things like stamps and coins.]
but it should not be used to introduce a clause

Incorrect: My brother can't play cricket like he did before the accident
Incorrect: It looks like cricket is quickly overtaking hockey as India's national sport.
Correct: My brother can't play cricket as he did before the accident.
Correct: It looks as if cricket is quickly overtaking hockey as India's national sport.

In both the correct sentences above as introduces a clause, and in the first sentence 'My brother collects
things like stamps and coins' like introduces a prepositional phrase.
To introduce a clause, its a good idea to use as, as though, or as if, instead. Study these sentences.

As I told you earlier, the lecture has been postponed. (Not Like I told you... )
It looks as if its going to rain this afternoon. (Not It looks like it's going to...)
Sapna kept looking out of the window as though she had someone waiting for her. (Not Sapna ... like she had
someone...)
In formal, academic text, its a good idea to reserve the use of like for situations in which similarities are
being pointed out:
This B-School is like a two-year rigorous imprisonment.

However, when you are listing things that have similarities, such as is probably more suitable:
JBIMS has several highly regarded neighbours, such as the Express Towers, The Oberoi Towers, the NCPA,
and the British Council Library. (Not Like the Express ....)
(You can to refer to Merriam Webster's Collegiate Dictionary if you want to know more about this)

That
When that is used as a subordinating conjunction it has direct relation to the verb in the main clause. In fact,
the omission of that in such sentences does not impair the sentence in any way.

We knew [that] she would make it to an IIM.


She felt [that] we were overestimating her abilities.
Remember, [that] she didn't have any problem in any of the tests that she took.
(Notice the comma in the last sentence. If you use that, the comma will disappear)
As a general rule, if the sentence feels just as good without the that, if no ambiguity results from its
omission, if the sentence is more efficient or elegant without it, then we can safely omit the that.

iQue ideas Page 100


Beginning a Sentence with 'because'
There is nothing wrong with beginning a sentence with because. Beginning a sentence with because will
highlight the cause.
Because terrorism has become such a serious threat to society, it is only natural that government wants
effective laws to combat it.

Conjunctive Adverbs
Words like however, nevertheless, therefore etc., are not true conjunctions. They are called conjunctive
adverbs or adverbial conjunctions. They, however, often function as conjunctions in joining two independent
clauses. Their Importance is as transitional devices. A transitional device makes the movement of ideas
smooth within long sentences or paragraphs. They are very important in your SOPs and essays. They serve
as transitional devices between one main thought and another.
Common Conjunctive Adverbs: ACCORDINGLY, AFTERWARDS, ALSO, CONSEQUENTLY,
HOWEVER, INDEED, LIKEWISE, MOREOVER, NEVERTHELESS, NONETHELESS, OTHERWISE,
SIMILARLY, SO, STILL, THEREFORE.

The correct use of conjunctive adverbs between clauses is shown below:


The book is expensive; however, it's worth it.

Conjunctive adverbs other than so and otherwise require a semicolon before and a comma after them. (seeso)
If these words( conjunctive adverbs) merely interrupt a sentence, they are not conjunctive adverbs and are
only surrounded by commas.

Either book, however, is fine with me.

Review Exercise

Choose the most appropriate conjunction to go into the blank.

1. We will accompany you __________ you don't reveal our identity to anyone. (so that, as long as, while ,
until , as it)
2. We will not accompany you _________ you promise not to reveal our identity to anyone. (so that, as
long as, while , until , as if)
3. We will accompany you ___________ you don't feel lonely on such a long journey. (so that, as long as,
while , until , as if)
4. We are really concerned about your decision to leave ____________ you seem to be rejoicing over it.
(so that, as long as, while , until , as if)
5. You look ___________ you have no regrets having to leave all of us so suddenly. (so that, as long as,
while , until , as if)
6. The room was hot, _____________ I switched on the air conditioner. (and, but, or, so)
7. I tried to understand Joyce's Finnegan 's Wake, __________ it was too complex. (and, but, or, so)
8. To go from Mumbai to Delhi, you can fly, __________ you can take the train. (and, but, or, so)
9. I have bought a new computer, ___________ it works really fast. (and, but, or, so)
10. I went to have a haircut, _________ the saloon was closed. (and, but, or, so)
11. She was here this morning. Maybe she wanted to tell us something, __________ maybe it was just a
courtesy visit. (and, but, or, so)
12. The movie was house full,________ we went to the beach. (and, but, or, so)
13. He wanted to study for the CAT, __________ he quit his job. (and, but, or, so)
14. We searched diligently,___________ found nothing. (and, yet, nor, so)

iQue ideas Page 101


15. He did not return my call, ___________ has he sent a message. (yet, but, nor, so)

Strike out the inappropriate conjunction:

16. Although / Because it was sunny and pleasant, I decided to carry my umbrella.
17. We did not visit him, even though / because he was in the hospital.
18. Whereas / Since my friends are avid tea drinkers, I prefer a hot cup of coffee.
19. All my friends had cleared the CAT in the first attempt, as/while I had to take it three times.
20. She got into a B-school as / even though she had not cleared the cut— off in DI.

Choose the correct sentence/s in terms of grammar and punctuation:

21. A. I really love ice creams, but I catch a cold when I have them.
B. I went to the doctor but the doctor wasn't in.
C. Everyone in the family, but dad is down with flu.
(1) A only (2) B only (3) C only (4) A and B only (5) All of the above

22. A. I decided to take an umbrella, because I thought it would rain.


B. I decided to take an umbrella because I thought it would rain.
C. Because I thought it would rain, I decided to carry an umbrella.
(1) A only (2) B only (3) C only (4) B and C only (5) All of the above

23. A. The students were frustrated as they had never expected the teacher to be so unconcerned.
B. The students were frustrated, as they had never expected the teacher to be so unconcerned.
C. As they had never expected the teacher to be so unconcerned, the students were frustrated.
(1) A only (2) B only (3) C only (4) A and C only (5) All of the above

24. A. Pay the fees on time, or the authorities may not allow you to attend lectures.
B. I cannot find a pen or a pencil.
C. Pay the fees on time or the authorities may not allow you to attend lectures.
(1) A and B only (2) B only (3) C only (4) A and C only (5) All of the above

25. A. Finish your dinner, and have a walk outside.


B. Carry pencils, and erasers to the examination hall.
C. He completed his MBA and he found a good job with an investment bank.
(1) A only (2) B only (3) B and C only (4) A and C only (5) All of the above

Fill in the blanks. Choice are in brackets.

26. He kept walking _________ he found the landmark in the map. (for, until)
27. She does not speak _________ spoken to. (as if, unless)
28. I have known him _________ he was a child. (until, since)
29. ________ you go there you will be able to meet him.
30. __________ you read this book several concepts are going to be unclear. (If, Until)
31. We will go for a picnic,___________ the Principal gives permission. (than, provided)
32. They went away, __________ they were told not to. ( although, despite)
33. Did you hear the phone ringing __________ the night? ( during, while)
34. He speaks _________ he knows everything. (like, as if)
35. She has specialised in finance, ____________ she is from a top institute. (And, besides)

iQue ideas Page 102


36. He searched the web for hours;__________ , he could not get the information he wanted. (but, however)

Rewrite the following sentences in parallel form.

37. He neither drives slowly nor carefully.


38. They are either arriving today or tomorrow.
39. He is not only smart but also is hard working.
40. The beach was both beautiful and it was clean.
41. The street both has several malls and it has several cinemas.

Answers and Explanations

1. We will accompany you as long as you don't reveal our identity to anyone.
2. We will not accompany you until you promise not to reveal our identity to anyone.
3. We will accompany you so that you don't feel lonely on such a long journey.
4. We are really concerned about your decision to leave while you seem to be rejoicing over it.
5. You look as if you have no regrets having to leave all of us so suddenly.
6. The room was hot, so I switched on the air conditioner.
7. I tried to understand Joyce's Finnegan's Wake, but it was too complex.
8. To go from Mumbai to Delhi, you can fly, or you can take the train.
9. I have bought a new computer, and it works really fast.
10. I went to have a haircut, but the saloon was closed.
11. She was here this morning. Maybe she wanted to tell us something, or maybe it was just a courtesy visit.
12. The movie was house full, so we went to the beach.
13. He wanted to study for the CAT, so he quit his job.
14. We searched diligently, yet found nothing.
15. He did not return my call, so has he sent a message.
16. Although it was sunny and pleasant, I decided to carry my umbrella.
17. We did not visit him, even though he was in the hospital.
18. Whereas my friends are avid tea drinkers, I prefer a hot cup of coffee.
19. All my friends had cleared the CAT in the first attempt, while I had to take it three times.
20. She got into a B-school even though she had not cleared the cut— off in DI.
21. (1) A - But is connecting two independent clauses. B — But is connecting two independent clauses, a
comma is required. C — But is used in the sense of 'with the exception of, hence no comma.
22. (4) A - Because is a subordinating conjunction, no comma needed. B — Because is a subordinating
conjunction, no comma needed. C — As the subordinate clause comes first in the sentence, it must be
offset with a comma.
23. (4) A - As is a coordinating conjunction and needs a comma. B — As is a coordinating conjunction and
needs a comma. C — The introductory clause needs to be offset with a comma.
24. (1) A - Or is a coordinating conjunction and is preceded by a comma. B — No need for a comma if 'or' is
separating two elements in a list. C — 'Or' is a coordinating conjunction and is preceeded by a comma.
25. (1) A — Here 'and' is joining two independent clauses and needs a comma. B — 'And' is separating two
items in the same clause and does not need a comma. C — Here 'and' is joining two independent clauses
and needs a comma.
26. He kept walking until he found the landmark in the map.
27. She does not speak unless spoken to.
28. I have known him since he was a child.
29. If you go there you will be able to meet him.
30. Until you read this book, several concepts are going to be unclear.

iQue ideas Page 103


31. We will go for a picnic, provided the Principal gives permission.
32. They went away, although they were told not to.
33. Did you hear the phone ringing during the night?
34. He speaks as if he knows everything.
35. She has specialised in finance, besides she is from a top institute.
36. He searched the web for hours; however. he could not get the information he wanted.
37. He drives neither slowly nor carefully.
38. They are arriving either today or tomorrow.
39. He is not only smart but also hard working.
40. The beach was both beautiful and clean.
41. The street is lined with both malls and cinemas.

iQue ideas Page 104


10
Subject – Verb Agreement
Subjects are nouns/noun phrases/ noun clauses.
The basic rule is simple. A singular subject takes a singular verb, and a plural subject takes a plural verb.
The grammatical subject is either the do-erof the verb or the be-er of the verb.
He runs fast. (thedo-er of runisHe which is the subject. )
He is fast (there is no action in the verb is. He is the be-er of the verb is. Hence He is the subject)

Singular verbs are: is/was/has/takes. Plural verbs are: are/were/have/take. (Any verb + 's' is singular. E.g.
take —plural; takes — singular)

Remember, the subject can be a single word (He in the above example) or a group of words (phrase or
clause)

For example,
His sister is in XLRI. (His sister is the subject)
His sister who is in XLRI is taking the CAT again. (His sister who is in XLRI is the subject)

And in the subject:


 As a general rule, use a plural verb with two or more 'subjects when they are connected by 'and'.
Correct: Manish and Nitesh are in charge of scheduling.
But, not always,
Correct: Bread and jam is good for breakfast.
Correct: Rice and beans, my favourite dish, reminds me of my village Kannur.
Correct: Slow and steady wins the race.

Either ... or in the subject.


 When two subjects are only related by either...or, neither.... nor, not(only) ... but (also), or the verb will
agree with the subject that is near to it.
Correct: Neither the manager nor his assistantis available.
Correct: Either she or I am getting the Best Student's Award.

Note:Am agrees with the subject close to it, I.


Correct: Neither money nor power was important to him.

Nor in the subject


 When a singular subject is connected by or or nor to a plural subject, put the plural subject last and use a
plural verb.

The book or the magazines are on the shelf.


Neither Parag nor the others are available.

Note: Putting the plural subject second is for the purpose of elegance (in your SOPs). The revere is also
grammatically correct.)

iQue ideas Page 105


Parenthetical element between the subject and the verb:

 Sometimes the subject is separated from the verb by words such as along with, as well as, besides, in
addition to. not, etc. Ignore these words (and the phrase) when determining whether to use a singular or
plural verb.

The politician, along with the newsmen, is expected shortly.


The father, accompanied by his sons, is travelling abroad.

Grammatically, these phrases/nouns following as well as, in addition to, along with, etc. are called
'parenthetical element'. Parenthetical elements are introduced using a preposition; what follows the
preposition is not another subject, as may be the case with conjunctions. The verb agrees with the subject.
The parenthetical element — separated from the subject using commas, or hyphens — deemphasizes the
information thus presented.
Incorrect: Amisha, as well as Sony, were present at the Silver Jubilee party.
Aswellas is preposition (used to introduce Sony). The subject is Amisha and hence the verb has to be
singular.
Correct: Amisha, as well as Sony, was present at the Silver Jubilee party.
Correct: The boys, as well as the girls, were pleased by the teacher's remark. (The subject is 'The boys",
which is plural.)

Study the construction of these sentences. Pay attention to the punctuation.


The boys, as well as the girls, were pleased by the teacher's remark.
Amisha (as well as Sony) was present at the Silver Jubilee party.
The father – along with his sons – wastravelling abroad.

Each in the subject.


 The pronouns each, everyone, everybody, anyone, anybody, someone, either, neither, and somebody are
always singular. Do not be misled by what follows 'of'.

Each of the girls sings well.


Every one of the cakes is gone.
Everyone in this batch is expecting a call from the IIMs.
Neither of them is available to speak right now.

Fractions and percentages.


 With words that indicate portions — per cent, fraction, part. majority, some, all, none, remainder, etc. —
you must look at what follows the 'of' to determine whether to use a singular or plural verb. If what
follows of is singular, use a singular verb.

Fifty per cent of the population is against the SEZ. (population follows of and is singular – uncountable
noun- hence the verb is the singular 'is')

But,
Fifty per cent of the villagers are against the SEZ. (The word 'villagers' that follows of is plural – countable
noun-hence the verb is plural ' are')

iQue ideas Page 106


Subject after the verb.
 The words here and there are not subjects because they are not nouns. In sentences beginning with here
or there, the true subject follows (comes after) the verb. Subjects are underlined.

There are five hundred students.


There is a huge crowd.

Money AND time


 Use a singular verb with sums of money or periods of time.

Ten thousand rupees is a high price to pay.


Five years is the maximum sentence for that offence.

Who, that which


 If the pronoun who, !hat, or which appears as the subject in the middle of the sentence, you must decide
whether to follow it with a singular or plural verb. In order to decide, look at the noun immediately
before the who, that, or which. If it is singular, use a singular verb. If it is plural, use a plural verb.

She is the secretary who writes the letters.


The word in front of who is secretary, which is singular. Therefore, use the singular verb writes.
He is one of the men who do the work.
The word in front of who is men, which is plural. Therefore, use the plural verb do. There are several men
who do the work: he is one of them.

Trousers AND pair of trousers


 Words such as glasses (spectacles), pants, pliers, and scissors are regarded as plural (and require plural
verbs) unless they're preceded by the phrase 'pair of' (in which case the word 'pair' becomes the subject).

My glasses were on the bed.


The trousers that she is wearing are expensive.
A pair of cotton trousers is in the closet.

The statistics are confusing.


 Some words end in 's' and appear to be plural but are really singular and require singular verbs.

The news from Delhi is bad.


Politics is the art of lying.
But we have to look at the word 'economics' differently. It can take singular or plural verb depending on the
situation
Economics is one of the subjects at B.Com.
The economics of the trip were not looked into carefully.

Another word in this class is 'statistics'.

More than one ...


 The expression 'more than one' (oddly enough) takes a singular verb:
More than one student has tried this.

Products and sums

iQue ideas Page 107


 Sums and products of mathematical processes are expressed as singular and require singular verbs.
Two and two makes four.
Four times four divided by two is eight.

Two-in-one
 Some sentences can be tricky in that, the subjects in them may appear plural but are actually singular.
As in:
His colleague and friend is going to help him.
(His colleague and friend is clearly the same person)

But,
His colleague and his friend are going to help him.
(His colleague and his friend are different people)

 At times, using a singular or plural verb changes the meaning of the sentence.
Keeping awake late in the night and working on the computer sometimes gives me a headache. (The above
sentence means that the combination of keeping awake late, and working on the computer can cause a
headache.)
Keeping awake late in the night and working on the computer sometimes give me a headache. (With a plural
verb (give), the sentence implies that keeping awake late, and working on the computer, act separately; either
can bring a headache.)

None
 There is one indefinite pronoun, 'none', that can be either singular or plural. Use a plural verb unless
something else in the sentence clearly determines its number.
Correct: None of you claim responsibility for this incident?
Incorrect: None of the students has done their homework.
Correct: None of the students have done their homework.
Incorrect: None of the luggage have reached us.
Correct: None of the luggage has reached us. (`luggage' determines the number.)

And not AND But not


 lf the sentence has a positive and a negative subject (one is plural, the other singular) the verb should
agree with the positive subject.

The students and not the teacher are to be given the credit for this achievement.
The teacher and not the students has to be given the credit for this achievement.

Collective Nouns
 Collective Nouns have different forms: a flock of sheep, a bunch of keys are phrases, while family* and
furniture are single word collective nouns.

Collective Nouns take a singular verb:

A flock of sheep is in the field.


A bunch of keys is on the table.
My family is going on a vacation.
The furniture has to be replaced.

iQue ideas Page 108


 *However, collective nouns which are collective names given to a group, e.g., family, team, crew, jury,
etc are used in two ways. They may refer to the unit without any reference to the members (then they
take a singular verb) or they may be used to refer to the members. In that case they take a plural verb.

The family is rich. (Family is a collective noun regarded as a unit.)


The family are avid readers. (The individuals of the family are regarded separately.)
The team is going to Australia tomorrow. (the unit)
The team are accompanied by their wives. (members of the team)

Collective Idea
 Sometimes, countable nouns are treated as a quantity and they become a collective idea. These will also
take singular verbs:

One thousand students is good enrollment in a month.


Pravin and Omkar for Verbal Ability is great for the students.

Review Exercise

Do as directed in the brackets.

1. Along the highway, men stood in a human chain. (Underline the subject of the sentence)
2. Beyond the hills were the villages known for their beauty. (Underline the subject of the sentence)
3. End of every tunnel is the light. (Underline the subject of the sentence)
4. Neither the student nor the teacher have a clue about this. (Underline the verb. If the verb does not agree
with the subject, correct the verb.)
5. The manager or his assistant is going to confirm the deal. (Underline the verb. If the verb does not agree
with the subject, correct the verb.)
6. Mr. Lott, along with Speaker Newt Gingrich, were among those who signed the letter. (Underline the
verb. If the verb does not agree with the subject, correct the verb.)
7. Each of those prizes is worth competing for. (Underline the verb. If the verb does not agree with the
subject, correct the verb.)
8. The letter, but not the entire file, were misplaced. (Underline verb. If the verb does not agree with the
subject, correct the verb.)
9. Here is all the gifts that you wanted for your birthday. (Underline the verb. If the verb does not agree
with the subject, correct the verb.)
10. The majority of the Lok Sabha is/are Congressmen. (Strike out the inappropriate verb.)
11. The original document, as well as all the subsequent copies we made, was/were lost. (Strike out the
inappropriate verb.)
12. Almost all of the magazine is/are devoted to advertisements. (Strike out the inappropriate verb.)
13. Here is/are Manish and Bhavin. (Strike out the inappropriate verb.)
14. Taxes deducted at source has/have to be paid to the government. (Strike out the inappropriate verb.)
15. The statistics on the sex ratio in the states is/are a cause of concern. (Strike out the inappropriate verb.)

Choose the correct verb in parentheses.

16. A black and yellow cab (is, are) waiting for him.
17. A system of stringent procedures (control, controls) the experiment.
18. One of the high ranking officers (was, were) involved in corruption charges.
19. Half of the food stored in the refrigerator (has, have) gone bad.

iQue ideas Page 109


20. Most of the news on page three of all these newspapers (is, are) about celebrities.
21. Almost two-thirds of the land acquired for the SEZ (is, are) cultivable.
22. Neither of the methods that you have used to solve this problem (make, makes) sense.
23. One hundred and fifty minutes of the CAT (require, requires) intense concentration.
24. Statistics (is, are) optional at final year of the course.
25. The number books in economics that we need to buy (is, are) three.
26. A number of books in economics that we can buy for the course (is, are) available in the college book
store.
27. Each, pen, pencil, and eraser left behind the students (was, were) collected by the attendant.
28. The weight lifter remarked that one hundred kilograms (was, were) easy to lift.
29. Few (is, are) self disciplined enough to study everyday for the CAT.
30. Lehman Brothers (is, are) now taken over by some other company.
31. Be careful because the new scissors (is, are) very sharp.
32. A swarm of bees (is, are) hovering over that tree.
33. My friend, philosopher, and guide (has, have) arrived to help me.
34. Mr and Mrs. Patel (has, have) invited me for dinner.
35. What my son likes the best on TV (is, are) cartoons.

Answers and Explanations

1. Along the highway, men stood in a human chain. (Underline the subject of the sentence)
2. Beyond the hills were the villages known for their beauty. (Underline the subject of the sentence)
3. At the end of every tunnel there is light. (Underline the subject of the sentence)
4. Neither the student nor the teacher have a clue about this. (Underline the verb. Theteacher is singular,
hence the verb should be has)
5. The manager or his assistant is going to confirm the deal. (Underline the verb. The verb is correct.)
6. Mr. Lott, along with Speaker Newt Gingrich, were among those who signed the letter. (Underline the
verb. The verb is incorrect. Mr. Lott is the subject,along with Speaker Newt Gingrich... is a parenthetical
element. The verb should be singular to agree with Mr. Lott — was)
7. Each of those prizes is worth competing for. (Underline the verb. The verb is correct.)
8. The letter, but not the entire file, were misplaced. (Underline verb. The positive subject takes the verb,
which is the letter — hence singular verb was.)
9. Here is all the gifts that you wanted for your birthday. (Underline the verb. All the gifts is the subject,
hence ) plural are should be used.)
10. The majority of the LokSabha are Congressmen. (Strike out the inappropriate verb.)
11. The original document, as well as all the subsequent copies we made, was lost. (Strike out the
inappropriate verb.)
12. Almost all of the magazine is devoted to advertisements. (Strike out the inappropriate verb.)
13. Here are Manish and Bhavin. (Strike out the inappropriate verb.)
14. Taxes deducted at source have to be paid to the government. (Strike out the inappropriate verb.)
15. The statistics on the sex ratio in the states are a cause of concern. (Strike out the inappropriate verb.)
16. A black and yellow cab is waiting for him.
17. A system of stringent procedures controls the experiment.
18. One of the high ranking officers was involved in corruption charges.
19. Half of the food stored in the refrigerator has gone bad.
20. Most of the news on page three of all these newspapers is about celebrities.
21. Almost two-thirds of the land acquired for the SEZ is cultivable.
22. Neither of the methods that you have used to solve this problem makes sense.
23. One hundred and fifty minutes of the CAT requires intense concentration.

iQue ideas Page 110


24. Statistics is optional at final year of the course.
25. The number books in economics that we need to buy is three.
26. A number of books in economics that we can buy for the course are available in the college book store.
27. Each, pen, pencil, and eraser left behind the students was collected by the attendant.
28. The weight lifter remarked that one hundred kilograms was easy to lift.
29. Few are self disciplined enough to study everyday for the CAT.
30. Lehman Brothers is now taken over by some other company.
31. Be careful because the new scissors are very sharp.
32. A swarm of bees is hovering over that tree.
33. My friend, philosopher, and guide has arrived to help me.
34. Mr. and Mrs. Patel have invited me for dinner
35. 35. What my son likes the best on TV is cartoons.

iQue ideas Page 111


11
Modifiers
The principal function of a modifier is to add information to another part of the sentence. In the chapter on
adjectives and adverbs you studied most of the usage issues related to these word classes, and we considered
them as single words - adjectives and adverbs.

But a modifier can be a group of words (phrase or clause) doing just what an adjective or adverb does. In this
chapter we will be looking at adjectival and adverbial function of groups of words (phrases and clauses as
modifiers). We will look at the possibilities of errors arising out of these.

First learn this basic principle. A modifier is immediately either preceded or followed by what it modifies
(modified)

For example:
It's a beautiful car - beautiful (adjective) modifies car (noun) - they are together.
She drives slowly -slowly (adverb) modifies drives (verb) - they are together.
She drives very slowly - very (adverb) modifies slowly (adverb) - they are together.
She is very slow - very (adverb) modifies slow (adjective) - they are together.

This order of the modifier and the modified should not be broken. If broken it creates an error.

First, in the above examples we again have single words, but we can have a group of words as either the
modifier or the modified or both.

For example:
Suddenly, everyone became quiet. Suddenly modifies everyone became quite.
Seeing the teacher enter the class, everyone became quiet. (seeing the teacher enter the class modifies the
subject everyone.
When I was a bachelor. I used to do my own cooking. ( When / was a bachelor modifies the entire main
clause / used to do my own cooking)

The modifier and what it modifies are always placed next to each other.
Next, there is a limited amount of flexibility one can exercise in placing the modifier in a sentence.

For Example:
We answered the questions carefully.
We carefully answered the questions.
Carefully we answered the questions. (carefully moves around without creating an error)

But this flexibility may not always be available - if the modifier goes too far away from the modified it may
create ambiguity and other kinds of errors in a sentence.

Misplaced Modifiers (Words, Phrases, and Clauses)


Incorrect: He could barely answer ten questions in the verbal section.
Correct: He could answer barely ten questions in the verbal section.
Incorrect: He was almost disappointed with every test he took.

iQue ideas Page 112


Correct: He was disappointed with almost every test he took.

Compare the underlined pans. In this respect, we have to always pay special attention to how only, just,
nearly, barely, often etc. (and other adverbs too) are placed in a sentence.

Incorrect: My friend told me that he saw the dead body on his way back home from college.
Incorrect : I happened to hit a young girl with my bag.

Both the above sentences are ambiguous and hence meaningless. `... on his way hack home from college'
does not modify the dead body but he in the first sentence. And in the second sentence young girl with my
hag is ambiguous.
Correct: My friend told me that on his way back home from college, he saw the dead body.
Correct: With my bag, I happened to hit a young girl.

Squinting Modifiers
The error of squinting modifier happens when a modifier seems to modify two things in a sentence because
of its placement. Place it next to the word that the modifier is actually meant of modify and the error
(ambiguity) will be removed.

Incorrect: He admitted to his girlfriend recently that he had lied to her.


Correct: Recently, he admitted to his girlfriend that he had lied to her.
He recently admitted to his girlfriend that he had lied to her.
He admitted to his girlfriend that he had recently lied to her.

In case of squint modifiers, place the modifier in such a way that the sentence communicates the intended
meaning.

Dangling Modifiers
An error of dangling modifier happens when the sentence has a modifier and the word or phrase it is
supposed to modify is either not stated in the sentence or not available for the modifier. Or worse, modifies a
wrong word.

Incorrect: Cruising at an altitude of 10,000 feet or so, the land below looked a strange place.

The agency that is cruising at an altitude of 10,000 feet or so is not stated in the sentence. Hence the phrase
— a modifier — dangles. Worse, it seems to modify what is placed next to it - the land below - which is
ridiculous.
We can correct the error of dangling modifier in two ways — rectify either the modifier or the modified.
Correct: When we were cruising at an altitude of 10,000 feet or so, the land below looked a strange place.
Correct: Cruising at an altitude of 10,000 feet or so, we felt that the land below looked a strange place.

Incorrect: To avoid sports injuries, the right kind of protective gear has to be worn.
Correct: To avoid sports injuries. one must wear the right kind of protective gear.

Incorrect: After finishing his presentation. the students were invited to comment about it.
Correct: After finishing his presentation. he invited the students to comment about it.
Correct: After he finished his presentation, the students were invited to comment about it.

Split Infinitives

iQue ideas Page 113


Inserting something between the to and the verb (as in to go) is called a split infinitive. Generally such
interruptions (to quickly go, rather than to go quickly) are considered an error. However, it is not always an
error.
We may have to break the infinitive to place the modifier next to the verb for clarity and elegance in certain
sentences.

Incorrect: The panel was unable to discuss the case completely owing to lack of evidence.
Correct: The panel was unable to completely discuss the case owing to lack of evidence.

Though one word interruption like the above are acceptable. long and unwarranted interruptions are always
an error.
Incorrect: The workers decided to. before going on a strike to demand better wages, petition the management
once again.
Correct: The workers decided to petition the management once again before going on a strike to demand
better wages.

Review Exercise

Analyse the following sentence for errors in modifiers and write CORRECT or INCORRECT against each.
If incorrect. rectify the error and rewrite.

1. Smiling cheerfully, Bhavin handed his girlfriend a huge packet of food.


2. The doctor was able to quickly and quietly administer the injection to the sleeping child.
3. After entering all the information about the residential addresses, phone numbers, dates of birth,
anniversary dates etc of his customers, the database seemed made up to date.
4. I had almost read the entire novel, when, to my great disappointment, I found that the last few pages
were missing.
5. Before buying a new computer, it is necessary that you carefully consider the configuration and price.
6. He fell in love with a beautiful girl with long hair from a small village in Kashmir.
7. Rising majestically above the horizon, and covered with shimmering white snow, the sight of the
Himalayas filled us with a sense of timelessness.
8. After shamelessly admitting his guilt, and begging the Principal for mercy, he was able to avoid
punishment for his mistake.
9. Even after saving for years together, barely they had enough money to buy a house in Mumbai.
10. To become a good writer, it is essential that one is a good reader too.
11. I noticed that there was a huge amount of surplus food in the Infosys food courts that was being wasted.
12. Working on the computer for long hours gives her a headache.
13. Although we were at the end of our journey, we were so tired that we decided to stop for the day.
14. He kept the photograph of his wife in a silver frame in his bedroom.
15. If it has to get published, the publishers should find your story credible.

Review Exercise

1. Smiling cheerfully, Bhavin handed his girlfriend a huge packet of food. CORRECT.
2. The doctor was able to quickly and quietly administer the injection to the sleeping child. INCORRECT.
(to quickly and quietly administer is a split infinitive) Quickly and quietly, the doctor was able to
administer the injection to the sleeping child.
3. After entering all the information about the residential addresses, phone numbers, dates of birth,
anniversary dates etc of his customers, the database seemed made up to date. INCORRECT — the

iQue ideas Page 114


database did not enter all the information. After he has entered all the information about the residential
addresses, phone numbers, dates of birth, anniversary dates etc. of his customers, the database seemed
made up to date. OR After entering all the information about the residential addresses, phone numbers,
dates of birth, anniversary dates etc. of his customers, lie felt that the database was up to date.
4. I had almost read the entire novel, when, to my great disappointment, I found that the last few pages
were missing. INCORRECT (almost is misplaced) I had read almost the entire novel, when, to my….
5. Before buying a new computer, it is necessary that you carefully consider the configuration and price.
INCORRECT Before buying a new computer, you should carefully consider the configuration and price.
6. He fell in love with a beautiful girl with long hair from a small village in Kashmir. INCORRECT. He
fell in love with a beautiful long-haired girl from a small village in Kashmir.
7. Rising majestically above the horizon, and covered with shimmering white snow, the sight of the
Himalayas filled us with a sense of timelessness. INCORRECT Rising majestically above the horizon,
and covered with shimmering white snow, the Himalayas filled us with a sense of timelessness.
8. After shamelessly admitting his guilt, and begging the Principal for mercy, he was able to avoid
punishment for his mistake. CORRECT.
9. Even after saving for years together, barely they had enough money to buy a house in Mumbai.
INCORRECT (barely is misplaced) Even after saving for years together, they had barely enough money
to buy a house in Mumbai.
10. To become a good writer, it is essential that one is a good reader too. INCORRECT. To become a good
writer, one has to be a good reader too.
11. I noticed that there was a huge amount of surplus food in the Infosys food courts that was being wasted.
INCORRECT. I noticed that there was a huge amount of surplus food that was being wasted in the
Infosys food courts.
12. Working on the computer for long hours gives her a headache. CORRECT.
13. Although we were at the end of our journey, we were so tired that we decided to stop for the day.
CORRECT.
14. He kept the photograph of his wife in a silver frame in his bedroom. INCORRECT. In his bedroom, he
kept a silver-framed photograph of his wife.
15. If it has to get published, the publishers should find your story credible. CORRECT.

iQue ideas Page 115


12
Parallelism
Parallel construction or parallel structure problems are among the most common in error spotting and
sentence correction questions.

When a sentence has several related ideas in it, the pattern of words used to express these ideas must have
the same grammatical forms (i.e., nouns, verbs, infinitives, gerunds, adjectives, etc.).The consistency in the
structure is required to show that these ideas have the same level of importance in the sentence.

Parallelism creates clarity, elegance, and symmetry in sentences. The following is a classic example of its
beauty: I came; I saw; I conquered - Julius Caesar.

Understand the basic structure of parallelism through these sentences:


He likes hiking, swimming, and cycling.(consistently—ing form)
He likes to hike, to swim, and to ride a bicycle.(consistently infinitive, i.eto +verb)
He likes to hike, swim, and ride a bicycle. (to is used either once or with each verb)

Incorrect: He likes hiking, swimming, and to ride a bicycle.

One way of checking whether the sentence is parallel is to quickly identify the main idea in the sentence.
Once this is identified any deviation from the parallel form in the related ideas in the sentence will be easy to
spot. In above sentence He likes is the main idea. Hiking, swimming, cycling are the related ideas. They
maintain the same grammatical form in all the correct sentences above. One more example:
It doesn't really matter what kind of clothes you wear or the language you speak.

Main Idea Related Ideas


It doesn't really matter what kind of (conjunction) Or clothes you wear the language you speak.

Faulty parallelism stands out when you analyse a sentence for its main idea and related ideas.

It doesn't really matter what kind of clothes you wear


(conjunction) Or language you speak. ....is parallel.

It doesn't matter what kind of clothes you wear or language you speak is the correct sentence. (kind of
clothes and kind of language, not kind of the language.)

Correlative conjunctions

When correlative conjunctions (either ... or, neither ... nor, both ... and, not ... hut, not only ... but also,
whether ... or, or are used or when words like, first, second etc are used to relate the ideas in sentences, make
sure that the sentence maintains parallelism.

Incorrect: His speech not only was long but was also very boring.
Correct: His speech was not only long but also boring
Incorrect: Either you must take the CAT or appear for CET in order to get into a B-School.
Correct: You must take either the CAT or the CET in order to get into a B-School.

iQue ideas Page 116


Comparisons:
When the sentence is comparing two different things, we must maintain parallelism wherever it is possible.
Incorrect: The requirements for JBIMS are not as strict as I IMs.
Correct: The requirements for AIMS are not as strict as the requirements for IIMs.

Prepositions
You can pay the hostel fees to the caretaker or to the warden.
When the elements that are to be parallel use the same preposition it is not necessary to use it twice. Hence,
though the above sentence is right, it would suffice to say:
You can pay the hostel fees to the caretaker or the warden.
I can meet you in the morning or evening.
The weather in Mumbai is the same in summer and winter.

However, when those elements require different prepositions, omitting one of them will be an error.
Incorrect: The students expressed their support and appreciation for the new plans.
Correct: The students expressed their support for and appreciation of the new plans.
Correct: The children were both attentive to and upset about the new teacher.

Quotations
Parallelism is a device that gives poetic beauty to prose. Most memorable passages will abound in parallel
structures. A few are quoted below, and all are from Winston Churchill's speeches. Pay attention to the
highlighted parts and notice the above principles at work.

(i) The day may dawn when fair play, love for one's fellow-men, respect for justice and freedom, will
enable tormented generations to march forth serene and triumphant from the hideous epoch in which
we have to dwell. Meanwhile, never flinch, never weary, never despair.
(ii) "We shall not flag or fail. We shall go on to the end. We shall fight in France, we shall fight on the
seas and oceans, we shall fight with growing confidence and growing strength in the air. We shall
defend our island, whatever the cost may be. We shall fight on the beaches, we shall fight on the
landing-grounds, we shall fight in the fields and in the streets, we shall fight in the hills. We shall
never surrender!"
(iii) "We have not journeyed across the centuries, across the oceans, across the mountains, across the
prairies, because we are made of sugar candy."
(iv) "We have surmounted all the perils and endured all the agonies of the past. We shall provide against
and thus prevail over the dangers and problems of the future, withhold no sacrifice, grudge no toil,
seek no sordid gain, fear no foe. All will be well. We have, I believe, within us the life-strength and
guiding light by which the tormented world around us may find the harbour of safety, after a storm-
beaten voyage."

Review Exercise

Rectify the faulty parallelism and rewrite.

1. Her husband promised her to cook the meal, wash the dishes and to clean the house.
2. The programme was entertaining, had various events, and even full of information.
3. The reasons for his refusal are: first the house was too small; second, that it was too far from his
workplace; third, there weren't enough number of bed rooms.
4. For breakfast she has every day a sandwich, glass of milk, and orange.
5. He was absent on Monday, Tuesday, Wednesday and on Friday.

iQue ideas Page 117


6. The students were exhausted and bored by the 150 minute tests.
7. He loves eating sandwiches but not to make them.
8. She likes chocolates better than having coffee.
9. Sanjaya's sense of humour is better than Amit.
10. The earliest settlers confronted by plants and animals that were unfamiliar to them, either borrowed their
local names or had to invent names of their own.
11. One way to reduce the time required for accurate computing is using parallel processing.
12. Men and women want three things in life: health, wealth. and to be loved.
13. Mahatma Gandhi was a great leader, a true philanthropist, and was truly patriotic.
14. You should not be so worried or discouraged by the inconsistency in scores.
15. The teacher told the students that they should solve one reading comprehension passage every day, that
they should learn ten new words ever day, and to read at least for one hour every day.

Answers and Explanations

1. Her husband promised her to cook the meal, to wash the dishes and to clean the house. OR Her husband
promised her to cook the meal. wash the dishes and clean the house.
2. The programme was entertaining, varied, and even informative.
3. The reasons for his refusal are: first, that the house was too small; second, that it was too far from his
workplace; third, that it did not have enough number of bed rooms.
4. For breakfast she has every day a sandwich, a glass of milk, and an orange.
5. He was absent on Monday, Tuesday, Wednesday and Friday. OR He was absent on Monday, on
Tuesday, on Wednesday and on Friday.
6. The students were exhausted from and bored by the 150 minute tests.
7. He loves eating sandwiches but not making them.
8. She likes chocolates better than coffee.
9. Sanjaya's sense of humour is better than Amit's.
10. The earliest settlers confronted by plants and animals that were unfamiliar to them, either borrowed their
local names or invented names of their own.
11. One way to reduce the time required for accurate computing is to use parallel processing.
12. Men and women want three things in life: health, wealth, and love.
13. Mahatma Gandhi was a great leader, a true philanthropist, and a true patriot.
14. You should not be so worried about or discouraged by the inconsistency in scores.
15. The teacher told the students that every day they should solve one reading comprehension passage, learn
ten new words. and read at least for one hour.

iQue ideas Page 118


13
Redundancy and Wordiness
Redundancy and wordiness are common errors tested in competitive examinations. Both these arrors are
widespread in our normal use of the language.

Redundancy is the repetition of an idea when it is unwarranted. Repetition is also a figure of speech used for
effective expression when it is deliberate and meant to create an impression. But causal and inadvertent
repetition of an idea is termed as redundancy. One of the web sites has this headline: Prominent Institutes
accepting CAT Test scores. CAT stands for Common Admission Test. So, the headline reads: Prominent
Institutes accepting Common Admission Test Test scores. This is redundancy. Other subtle and mindless
redundancies include phrases like: hurry up fast, repeat again, cooperate together, free gift, tiny little child
etc. These are redundant phrases. The correct sentence in the CAT will not have redundancy, especially in
questions in which you are asked to pick out the best sentence from several options.

Sometimes grammatical stylists use the term semantic pleonasm to describe redundancy.

Wordiness is not much different from redundancy. It is the use of more words than are necessary to convey
an idea. Expressions like, brief period of time (brief) at the earliest possible date (at the earliest), arrived at
the conclusion (concluded), absolutely indispensable (indispensable) are wordy. The concise expression is
indicated in brackets. Wordiness is a sign of stupidity. Concision is a sign of intelligence.

We must also note that the language does allow some space for redundancy as well. An old adage, more
precise, a fuller figure etc. are accommodated in the language, though strictly considered they are redundant.

Incorrect: He donated Rs.1000 of his own money for the cause.


Correct: He donated Rs. 1000 for the cause.
Incorrect: The MP rose up to say that in her opinion she thought that the Women's Reservation Bill should be
passed on immediately.
Correct: The MP rose to say that the Women's Reservation Bill should be passed immediately.

Given below is a list of redundant/wordy phrases. You can easily revise them to make them precise.

Cat EXAM, ATM machine, VAT Tax, HIV virus., DVD disc, PIN number, triangular in shape, auto-
biography of my life, the future to come, that final conclusion, ascend up, essential necessity, good success,
nonreading illiterates. empty rhetoric, equally as, inside of, mental telepathy, rarely ever/seldom ever, reason
is because, reason why, continue on, current status, doctorate degree, end product. end result, few in number,
final conclusion jbreign imports, join together, may possibly, mutual understanding, new breakthrough, over-
exaggerate, period of time. rate of speed, repeat again, safe haven, sum total. tall in height, temporary loan,
true fact, 12 noon, unite together. unite together etc.

Types of redundancy:
 Unnecessary modifiers: immediate vicinity, personal opinion, free gift, past memories etc.
 Unnecessary pairs, in which either of the paired words is good enough: each and even': .first and
foremost. Mil and complete, any and all, various and sundry, so on and so forthetc.
 Unnecessary broadening of a definition: blue in colour, small in size, attractive in appearance etc

iQue ideas Page 119


 Unnecessary explanations: until such time, at this point in time, during the course of in the event that.
owing to the fact that, in the final analysis etc.

A few usage issues explained.

All right AND Alright.


All right (two words) is the correct idiomatic expression. Alright is non standard use.
Incorrect: I am alright
Correct: I am all right.

As good or better than


Incorrect: Your guess is as good or better than mine.
Correct: Your guess is as good as mine, or better.

As to whether
Whether is sufficient.
Incorrect: The workers will vote on the question as to whether they should strike.
Correct: The workers will vote on the question whether they should strike. OR
Correct: The workers will vote on whether they should strike.

Character/ Nature
Wordy in most contexts in which a defining word is explained.
Incorrect: All his offers were of a benevolent nature/character.
Correct: His offers were benevolent.

Compare
Avoid compare and contrast. It is wordy. Compare is good enough.
compare to is used to force a comparison between two normally dissimilar objects.
Life is compared to a dream.

compare with is used to put side by side (compare) two objects that are normally similar.
The performance of the BJP is compared with that of the Congress.

Consider
Consider is not generally followed by as or to be. Consider as has a different meaning.
Incorrect: I consider him as my friend. I consider him to be my friend.
Correct: I consider him my friend.
Correct: The professor considered Vajpayee first as a poet and second as an administrator
Here, consider as has a different meaning — 'examined' or 'discussed.'

Due to
One finds (in the press, etc.) instances of more incorrect use of due to than correct, incorrectly substituted for
through, because of or owing to.
Incorrect: He was absent due to illness.
Correct: His absence was due to illness.
Correct: This success is due to Parag's brilliant ideas.
To check if the use of due to is correct: Substitute due to with attributable to in the sentence. If the sentence
makes sense with attributable to the use of due to is correct. Otherwise rephrase.

iQue ideas Page 120


Try and come
This expression has no meaning and is incorrect. Always use try to and not try and.
Try to come, Try to do etc.

Less/ Lesser/fewer
Less should not be misused for fewer. Less refers to quantity, fewer to number.
Certain (rather rare) countable nouns can use fewer and less with a change in meaning. For example,
someone else has less troubles that I have means the extent of the troubles is not great. But if, someone else
has .fewer troubles than I have , it means that my number of troubles is more than his.
It is, however, correct to say, 'The students who requested an additional doubt session were less than ten,
.where the word tens treated something like a collective noun, and less is thought of as meaning a less
quantity or amount.

These are the correct comparative forms:


Few — fewer — fewest
Little — less/lesser – least

Nature
Redundant if used to further explain a quality ( see character) 'generous by nature', reserved by nature' etc.,
are incorrect. But 'poems about nature' lover of nature' are correct expressions because in these contexts the
word refers to rural/natural scenery.

One hundred one


Always use one hundred and one Or One hundred and fifty. Use the and - one hundred one or one hundred
fifty is incorrect.

Should
(Also see the chapter on Verbs.)
A conditional statement in the first person requires should, not would.
Incorrect: I would not have cracked the CAT without Parag's help.
Correct: I should not have cracked the CAT without Parag's help.

People AND Peoples

When the word people refers to body of persons sharing a common religion, culture, language etc. it can be
distinguished from another body of persons sharing a different religion, culture or language. These two
groups then become peoples.
"The State of India's indigenous and Tribal Peoples 2008 report covers indigenous peoples and armed
conflicts, violations of the rights of indigenous tribal peoples with particular focus on women and children,
violations by the armed opposition groups. violations of indigenous peoples' right to land, displacement of
indigenous/ tribal peoples. repression under forest laws…….”

People AND Public


People is a political term and public is a civil term meaning the community as a whole. People show support
to their political leaders and the cricket team enjoys public support.

Persons AND people


The word person has two plurals persons or people. Teh word people can be used in all contexts as the plural
of person.

iQue ideas Page 121


When the term is specific to the numbers involved the plural persons can be used instead of people. There
were twenty-five people in the class. If five persons (people) leave. how many persons are left?
Several people were interviewed Several persons were interviewed are correct sentences.

Review Exercise

Make the.frillowing words' sentences concise. Rewrite them.

1. As a matter of fact. this year we have put more students in the IIMs than we did last year.
2. As far as I am concerned. I do not see any reason for you to take more tests.
3. The farmers in the interior of Maharashtra are in trouble at the present time.
4. The industrial giant had to abandon its project because of the fact that the local farmers objected to it.
5. Due to the fact that there is great deal of speculation in the commodities market. oil prices are always on
the increase.
6. We could quiet clearly make out that for all intent and purposes his remarks were meant to insult her.
7. The current decrease in home loan rates of interest. in a manner of speaking. is just ploy to get more
customers. so that they can increase the rates soon after.
8. His speeches inevitably have a tendency to create a controversy.
9. What I mean to say is that. all things considered, Singh is a god leader.
10. Government has acquired quite a bit of agricultural land for the purpose of starting industries.

Make the following sentences concise. Rewrite the sentences.

11. At this point in time. I understand the reason as to why students duo not like the new teacher.
12. Parag. who is employed as a consultant at SPJIMR, suggests conduct of monthly outbound training
programme.
13. Basically. in view of the fact that he is totally exhausted by his work schedule, there was an
apprehension on the part of teachers that CAT may possibly not figure in his future plans.
14. It is to be hoped that the administration will discover some means to find an absolutely proper and
decent solution to this problem of eave teasing.
15. There is a desire on the part of many of us, students to go on a picninc for the purpose of getting away
from the stress of our studies.

Eliminate wordiness and rewrite

16. He felt that his neighbour's daughter who lived next door was beautiful to look at.
17. He was really sorry due to the fact that he was late.
18. Although the students were several in numbers, each and every one was worried.
19. The teacher provided an explanation to the problem.
20. During the time he was in a village in South India, it was his intention to study in Mumbai.

Answers and Explanations

1. This year we have put more students in the IIMs than we did last year.
2. I do not see any reason for you to take more tests.
3. The farmers in the interior of Maharashtra are in trouble now.
4. The industrial giant had to abandon its project because the local farmers objected to it.
5. Because there is a great deal of speculation in the commodities market, oil prices are always on the
increase.

iQue ideas Page 122


6. We could quiet clearly make out that for his remarks were meant to insult her.
7. The current decrease in home loan rates of interest is just ploy to get more customers, so that they can
increase the rates soon after.
8. His speeches inevitably create a controversy.
9. Singh is a good leader.
10. Government has acquired quite a bit of agricultural land for starting industries.
11. I can't understand why the students do not like the new teacher.
12. Parag, who is a consultant at SPIMR, suggests monthly outbound training programme.
13. Because he is exhausted after his work, the teachers felt that CAT may not figure in his plans.
14. We hope that the administration will find a solution to eave teasing.
15. We want to go on a picnic to get away from studies.
16. He felt that his neighbour's daughter was beautiful.
17. He was sorry because he was late.
18. Although there were many students, everyone was worried.
19. The teacher explained the problem.
20. When he was in South India, he wanted to study in Mumbai.

iQue ideas Page 123


14
Punctuation
It is necessary to be aware of certain principles of punctuation in competitive examinations. The principles
tested in the examination are elementary, hence a detailed study of punctuation is not required; however, it
may help you write better. The basic principles of punctuation are explained here, from a purely examination
point of view: the finer details are left out. You need to pay attention basically to the rules related to the
comma, apostrophe, and semicolon. A rare question in the exam may include issues related to the dash or
colon as well.

Commas

Three important rules for comma use:

I. Use a comma before a coordinating conjunction if it joins two independent clauses.


II. Use a comma with introductory words, phrases, or clauses in a sentence.
III. Use commas to set off elements that interrupt or add information in a sentence.

I. Use a comma before a coordinating conjunction if it joins two independent clauses. (Coordinating
conjunctions are FANBOYS: For-And-Nor-But-Or-Yet-So.)

When two independent clauses are joined using a coordinating conjunction, use a comma before the
conjunction. Independent clauses are parts of sentences that can stand alone as a complete sentences.

For example: She gave me a smile, and then she walked straight past me and kissed him.

Make sure they are independent clauses and not some other construction in which commas are not required.
She gave me a smile. Then she walked straight past me and kissed him. These are independent clauses. In the
case of the second and, i.e., and kissed him, is not an independent clause. Hence a comma will be an error
before and kissed him.

Incorrect: She gave me a smile, and then she walked straight past me, and kissed him.
Incorrect: She gave me a smile and then she walked straight past me and kissed him.
Correct: She walked straight past me and kissed him.

II. Put a comma after introductory words, phrases, or clauses in a sentence.

Yes, Definitely, Sir, well etc. are introductory words.

Sir, we are completely confused.


Definitely, we will work harder.
Ofcourse, we are determined to clear the CAT.
Wherever they occur in a sentence they need to be set off using a comma, how many ever such words there
may be.
Ofcourse, Sir, we are determined to clear the CAT.
Definitely, we are determined, Sir, to work harder.
(Generally, these words are best placed at the beginning of the sentence rather than elsewhere.)

iQue ideas Page 124


Introductory phrases are:
 Prepositional phrases like, after working for several hours (beginning with preposition and ending with a
noun)
 Infinitive and participial phrases (modifiers) like, to become a good teacher.... Trying to make for the
lost time…., etc.

Prepositional phrases:
Always put a comma after a prepositional phrase if it is more than 4 or 5 words. Ver) short ones will not
require comma. Prepositional phrases are phrases that begin with a preposition and end with a noun.

Correct: Before going home he said goodbye to me. (no comma after home - a very short phrase)
Correct: On his birthday he got many gifts (no comma after birthday - a very short phrase)
Incorrect: Before going home without .finishing his lectures .for the day he said goodbye to me.
Correct: Before going home without finishing his lectures for the day. he said goodbye to me.

Before going home without finishing his lectures .for the day is a combination of three prepositional phrases.
( Before going home without. finishing his lectures for the day — no commas are required between them)

Infinitive and participial phrases:


Set off modifiers (infinitive ad participial phrases) with a comma.
Correct: To apply to this institute, you must have at least 95 percentile.
Correct: Seeing her in tears. he felt extremely sorry.

Incorrect: To apply to this institute. will be a waste of money. (The infinitive phrase is not a modifier in this
sentence but the subject of the sentence: hence there should not be a comma.)

Clauses:
When starting a sentence with a dependent clause. use a comma after it. Conversely. do not use a comma
when the sentence starts with the main clause followed by a dependent clause.( The subordinating
conjunctions are as good as a comma)
Correct: If you are not sure about this. let me know now..
Correct: Let me know now if you are not sure about this.
Incorrect: Let me know nosy. if you are not sure about this.

III. Use commas to set off elements that interrupt or add information in a sentence.

Conjunctive adverbs are always set off with a comma. ( also, besides, furthermore, however, indeed, instead.
moreover, nevertheless, otherwise, therefore, thus etc.. are some conjunctive adverbs)

Correct: She saw me: nevertheless she did not acknowledge me.
Correct: She saw me: she did not acknowledge me. Nevertheless

If something or someone is sufficiently identified. the description following it is considered nonessential and
should be surrounded by commas. (Description introduced using who or which)
Amit. who is the director of Star, is quitting. ( Amit is named so the description is not essential )
The boy who comes from Vasai Road got calls from all the six IIMs.
We cannot know which boy is being referred to without the information who comes from Vasai, Road,
therefore. no commas are used.

iQue ideas Page 125


Parenthetical elements are always set off using commas.
Rachana. as well as Nidhi, has got above 99 percentile.
Rachana has got above 99 percentile. As well as Nidhi.
As well as Nidhi, Rachana has got above 99 percentile.

here are several other rules for using the comma. We w ill look at them in brief. But comma use is anyhow
complex; a modest awareness is good enough. Remember Oscar Wide’s comment: “I have spent most of the
day putting in a comma and the rest of the day taking it out”.

Commas in a Series

Use commas to separate three or more words, phrases, or clauses in a series. The last item in the series will
begin with a conjunction.

Journalism: He purchased black, blue and red pens.


Correct: He purchased black, blue, and red pens.

The second comma — before and - is called the Oxford Comma or the Serial comma. Leaving this out can
create confusion in certain constructions.

Incorrect: The room was filled with angry parents, shouting students and teachers. (Were the teachers also
shouting?)

Correct: The room was filled with angry parents, shouting students, and teachers.
Correct: The Will required that her money be equally divided among her husband, daughter, son, and
nephew.

Omitting the comma after son would change the meaning of the sentence and indicate that the son and
nephew would have to share one third of the money. It is safer to use the Oxford Comma.
The same rule applies to phrases and clauses:
Correct: The room was filled with parents who were angry, students who were shouting, and teachers who
were trying to maintain calm.

Commas with Adjectives:

When two adjectives qualify a noun, e.g. intelligent, talented lady, use a comma to separate them only when
it is possible to insert and in between the adjectives (intelligent and talented). If you cannot insert and do not
use the comma.
Correct: She is an intelligent, hardworking woman. (intelligent and hardworking)
Correct: She is an intelligent young lady. (not intelligent and young)

Use a comma when an ly adjective is used with other adjectives.


Correct: She is a friendly, young lady.
To test if an ly word is an adjective, see if it can be used alone with the noun. If it can, (friendly lady) use the
comma.

Incorrect:This essay has many awkwardly, constructed sentences.

Awkwardly is not an adjective because it cannot be used alone with sentences.

iQue ideas Page 126


Correct: This essay has many awkwardly constructed sentences.

Notice the use of commas (or their absence) in date:


16th July 2008
June 16, 2008
Wednesday, June 16, 2008
16 June 2008

In sentences use a comma to separate the day of the month from the year and after the year.
Correct: I met my wife on September 25, 1988, in Ruparel College.
Correct: I met my wife on September 25, 1988 in Ruparel College.

If any part of the date is omitted, leave out the comma.


Correct: We met in September 1988 in Ruparel College.

Use a comma to separate the city from the state and after the state.
Correct: I lived in Mumbai, Maharashtra, for 20 years.

Correct: I lived in Mumbai, Maharashtra for 20 years.

Use commas to surround degrees or titles used with names.


Sreenivas, B. Tech., spoke to Shilpa, MBA.

Use a comma to introduce or interrupt direct speech.


"Why," I asked, "do you always remain silent in these meetings?"
He said, "I do not know."

Semicolon
Use the semicolon in place of a period (full stop) to separate two sentences where the conjunction has been
left out. Incorrect: Call me tomorrow, I will explain everything then. (Use semicolon)
Correct: Call me tomorrow; I will explain everything then.
Correct: I have paid my dues; therefore, I expect all the privileges listed in the Student Rule Book.

Do not use a semicolon in front of words such as therefore and however if they do not connect two complete
sentences or are used as interrupters. Use commas instead.
Correct: I would, therefore, like a response.
Correct: I would be happy, however, to speak in the Seminar.

Use the semicolon to avoid confusion where commas already exist.


This conference has people that have come from Mumbai, Maharashtra; Bangalore, Karnataka; and Chennai,
TN.

Apostrophe

Use the apostrophe with contractions. The apostrophe is placed at the spot where letters have been removed.
Examples:
Don't (do not)
Hasn't (has not)

iQue ideas Page 127


When two words are truncated, the apostrophe is in the second part : shall not - shan't
Apostrophe is used to show the possessive of nouns by placing an apostrophe ( with 's') after a singular noun.
the boy's room — the room belongs to one boy

To show possession of plural noun make the noun plural and put immediately use the apostrophe.
boys' room — the room belongs to several boys. Boys' Common Room.
Birla's car(car belongs to one Birla)
Birlas' car(car belongs the Birla family)

When the plural of the noun does not end in's' use the apostrophe and 's'
Women's , Children's

With singular proper nouns ending in 's' one has the option to, a mere apostrophe or apostrophe +s is used.
Suhas' book
Suhas's book. Both are correct.

Use the apostrophe and s after the second name if two people possess the same item.
Anil and Sunita's home is under construction. (joint ownership of one home)
Anil's and Sunil's job contracts will be renewed next year. (separate ownership)
Anil and Sunita's applications for loans have been approved by the banks. (joint ownership of several
applications)

Apostrophe used with pronouns will indicate that they are contractions. None of the pronouns show the
possessive using the apostrophe.

No apostrophe in yours, hers, theirs, mine, his, its

It's nice (It is nice)


It’s been nice (It has been nice)
Its greatness (the greatness belongs to it)

Avoid apostrophes to show the plurals of numbers and contractions.


Correct: She consulted three MBAs.
Correct: I had lived here in the 70s (or the 1970s).

All the following are correct plurals: He has got three A 's in his mark sheet. Students came in groups of 3 's
and 4 's.
The child was asked to mind his p's and q's. There are several PhDs in this college. There are several Ph.D.'s
in this college. ( 's if the abbreviation is written with periods.)

Colon

Use the colon after a complete sentence to introduce a list of items when introductory words such as namely,
for example, or that is do not appear.
You require the following in your office: tables, chairs, and computers.
I want the following items: a table, chair, and computer.
I want a servant who can do the following: 1) cook, 2) clean the house, and 3) answer telephone calls.
I want a servant who can 1) cook, 2) clean the house, and 3) answer telephone calls.

iQue ideas Page 128


No colon precedes the numbers in the last example because I want a servant who can is not a complete
sentence. Use a capital letter after the colon only if what follows is a quotation. He said : "Let's go."

Use the colon to introduce a list in tabular form whether it is preceded by a complete sentence or not.
I want a servant who can:
a) cook
b) clean the house
c) answer telephone calls.

I want a servant who can do the following:


a) cook
b) clean the house
c) answer telephones calls.

Use the colon to introduce a direct quotation.


It is stated in the Constitution: "Any Indian national above 18 is entitled to vote".

Use colon to introduce an explanation.


There is only one thing you can do now: run away.
Always skip two spaces after a colon while typing.

Capitalization

Capitalize the first word of a sentence. Capitalize a proper noun.


Capitalize the first word of a quoted sentence.
He said, "I love you."

It is often requested that you capitalize a person's title when it precedes the name.
Chairperson Banga OR chairperson Banga

Do not capitalize when the title is acting as a description following the name.
Mr. Banga, the chairperson of the company, will address us at noon.

Capitalize when the person's title follows the name on the address or signature line.
Sincerely,Mr. Banga, Chairperson

Capitalize the titles of high-ranking government officials.


The President will address the Lok Sabha. All MPs are expected to attend.
Prime Minister Manmohan Singh will discuss the matter.

Capitalize directions only when they refer to specific regions.


We have had Presidents from the South.
The southwest monsoon starts in June.

Do not capitalize names of seasons.


I will visit you in summer.

Capitalize titles of publications except for a, an, the, but, as, if; and, or, nor, and other little words when used
internally. Capitalize short verb forms like Is, Are, and Be.

iQue ideas Page 129


‘A Portrait of the Artist as a Young Man' (James Joyce)

Capitalize a sentence after colon if two or more sentences follow.

Period
Use a period after a sentence. Use a period after an indirect question.
He asked where his suitcase was.

If the last word in the sentence ends in a period, do not follow it up with another period.
I know that M.D. She is my sister-in-law.
You please help me in cooking, shopping etc. I will look after the kids.

Dashes
The dash is longer than a hyphen. A hyphen used only between words, and not phrases or clauses, E.g., self-
consciousness (all words with self prefix should be hyphenated), do-it-yourself guides etc.
Dashes are slightly longer and there are two types: the en-dash (width of the letter N), and the slightly longer
en-dash (width of the letter M). We find the en-dash generally between numbers, 5-6 hours, 2008-2009, and
other chronological ideas, Mon-Fri.

The longer em-dash is used to set off parenthetical elements when those elements contain commas or other
punctuation marks, and more commas would create confusion.
Incorrect: Many of our students, Kavita, Lerroy, Sreejit, and Anthony, are in IIMA.
Correct: Many of our students — Kavita, Lerroy, Sreejit, and Rohit — are in IIMA.

The dash is correctly used if there is an interruption in thought or tone. Generally its absence is what you
have to look for in competitive examinations.

Review Exercise

Choose the sentence with the correct comma placement.


1. (1) I took Gone with the Wind the novel by Margaret Mitchell, to read on the train.
(2) I took Gone with the Wind, the novel by Margaret Mitchell to read on the train.
(3) I took Gone with the Wind, the novel by Margaret Mitchell, to read on the train.
(4) I took, Gone with the Wind, the novel by Margaret Mitchell to read on the train.
(5) I took Gone with the Wind the novel by Margaret Mitchell to read, on the train.

2. (1) He is, we are all certain trying to misguide, you and her.
(2) He is, we are all certain, trying to misguide, you and her.
(3) He is we are all certain, trying to misguide, you and her.
(4) He is, we are all certain, trying to misguide you and her.
(5) He is we are all certain, trying to misguide you and her.

3. (1) She has for breakfast, some fruit, a glass of milk, and some bread, and jam.
(2) She has for breakfast some fruit, a glass of milk, and some bread and jam.
(3) She has for breakfast, some fruit, a glass of milk and some bread, and jam.
(4) She has for breakfast some fruit, a glass of milk, and, some bread and jam.
(5) She has for breakfast some fruit a glass of milk, and, some bread and jam.

4. (1) They are planning to leave Mumbai, India, on November 18 this year.

iQue ideas Page 130


(2) They are planning to leave Mumbai, India, on November 18, this year.
(3) They are planning to leave, Mumbai, India, on November 18, this year.
(4) They are planning, to leave Mumbai, India, on November 18, this year.
(5) They are planning to, leave Mumbai India, on November 18 this year.

5. (1) You can clear the CAT, if you study for 3 hours every day.
(2) You can clear the CAT, if you study for 3 hours, every day.
(3) You can clear the CAT if you study for 3 hours every day.
(4) You can clear the CAT if you study, for 3 hours every day.
(5) You can clear the CAT, if you study for 3 hours, every day.

In which of the following can the '/' be correctly replaced by a semicolon?

6. 1. She loves ice creams/vanilla is her favorite flavor.


2. She loves ice creams/ in summer or in winter.
3. She loves ice creams/chocolate, vanilla, butterscotch — everything!
4. Ice creams are great/ the flavors are too many and they all taste good.
(1) 1 and 4 only (4) 3 and 4 only
(2) 2 and 3 only (5) All of the above.
(3) 1 and 2 only

7. 1. Grammar is not very easy/ nor is it very difficult.


2. Grammar is important/communication skills are crucial in management.
3. Grammar is very important/ top management requires good writing skills.
4. We have to study grammar/ though grammar is boring.
(1) 1 and 4 only (4) 3 and 4 only
(2) 2 and 3 only (5) All of the above
(3) 1 and 2 only

8. 1. I am taking the CAT this year/which is quite famous for its surprises.
2. I am taking the CAT this year/no time to waste!
3. I am talking the CAT this year/XAT as well.
4. I am taking the CAT this year/ it is quite famous for its surprises.
(1) 1 only (2) 2 only (3) 1 and 2 only (4) 3 and 4 only (5) 4 only

9. 1. Pooja is a clever student/ well mannered and hardworking.


2. Pooja is a nice girl/ she does not hurt anyone.
3. Pooja is quite an introvert/however, she excels in group discussion.
4. Pooja is unattached/ 1 hope I can win her heart.
(1) 1 only (2) 2 only (3) 1 and 3 only (4) 2, 3 and 4 only (5) 4 only

10. 1. CAT is becoming increasingly popular/IIM aspirants have tripled in the last three years.
2. CAT is becoming increasingly popular/and a difficult one too.
3. CAT is becoming increasingly popular/many students are studying harder than ever.
4. CAT is becoming increasingly popular/even among the non-engineers.
(1) 1 only (2) 2 only (3) 1 and 3 only (4) 2, 3 and 4 only (5) 4 only

Select the correctly punctuated sentence.

iQue ideas Page 131


11. (1) India is incredible; the long beaches, the snowcapped mountains, and the deserts are enchanting.
(2) India is incredible: the long beaches, the snowcapped mountains, and the deserts are enchanting.
(3) India is incredible, the long beaches. the snowcapped mountains, and the deserts are enchanting.
(4) India is incredible: the long beaches, the snowcapped mountains and the deserts are enchanting.

12. (1) Last summer we visited the following places in Malaysia; Sepang Fl circuit, St.Paul's Hill
(A'Famosa). and Mt. Kinabalu.
(2) Last summer we visited the following places in Malaysia: Sepang Fl circuit, St.Paul's Hill
(A'Famosa); and Mt. Kinabalu.
(3) Last summer we visited the following places in Malaysia: Sepang Fl circuit, St.Paul's Hill
(A'Famosa), and Mt. Kinabalu.
(4) Last summer we visited the following places in Malaysia: Sepang; F 1 circuit; St.Paul's hill
(A'Famosa); and Mt. Kinabalu.

13. (1) We enjoyed the sightseeing, food, travel and company of friends.
(2) We enjoyed: the sightseeing, food, travel and company of friends.
(3) We enjoyed the sightseeing food travel and company of friends.
(4) We enjoyed the sightseeing, food, travel and company of friend's.

14. (1) Amit's sister's wedding reception is going to be held in that vast green turf club.
(2) Amit's sisters wedding reception is going to be held in that vast, green turf club.
(3) Amit's sister's wedding reception is going to be held, in that vast green turf club.
(4) Amit's sister's wedding reception is going to be held in that vast, green turf club.

15. (1) I can't find my dads' invitation card, it's probably lost.
(2) I can't find my dad's invitation card; its probably lost.
(3) I can't find my dad's invitation card; it's probably lost.
(4) I cant find my dad's invitation card; it's probably lost.

16. (1) Sreejit's marriage is on Sunday; so his and his brother's friends have already arrived.
(2) Sreejit's marriage is on Sunday, so his and his brother's friends have already arrived.
(3) Sreejit's marriage is on Sunday, so his and his brothers friends have already arrived.
(4) Sreejit's marriage is on Sunday: so his and his brother's friends have already arrived.

17. (1) Suhas's sister-in-law a smart young lady came in his brother-in-law's car.
(2) Suhas's sister-in-law, a smart young lady, came in his brother's-in-law car.
(3) Suhas's sister-in-law, a smart, young lady, came in his brother-in-law's car.
(4) Suhas's sister-in-law, a smart young lady, came in his brother-in-law's car.

18. (1) After dropping her at the wedding, Suhas lost his way but reached Srinivas' office on time.
(2) After dropping her at the wedding, Suhas lost his way, but reached Srinivas' office on time.
(3) After dropping her at the wedding Suhas lost his way, but reached Srinivas office on time.
(4) After dropping her at the wedding, Suhas lost his way but reached Srinivas' office, on time.

19. (1) This summer they are going to visit: Paris, Amsterdam, London, and Switzerland's Alps.
(2) This summer they are going to visit Paris, Amsterdam, London, and Switzerland's Alps.
(3) This summer they are going to visit Paris, Amsterdam, London, and Switzerlands Alps.
(4) This summer they are going to visit Paris Amsterdam London and Switzerland's Alps.

iQue ideas Page 132


20. (1) That stupid boy, his younger brother, is this test's topper.
(2) That stupid boy his younger brother is this test's topper.
(3) That stupid boy, his younger brother, is this tests topper.
(4) That stupid boy, his' younger brother, is this test's topper.

Answers and Explanations

1. (3) Since the book is sufficiently identified by its title, the description following it is nonessential and
should be surrounded by commas.
2. (4) Surround interrupting expressions with commas.
3. (2) She has three items for breakfast; bread and jam cannot be separated because it is one dish.
4. (1) Use a comma to separate city from the country and again after the country in the text. If any part of
the date is left out, omit the comma.
5. (3) Do not use a comma when the sentence starts with an independent clause followed by a dependent
clause.
6. (1) 1 and 4 only. These clauses can stand on their own and are closely related.
7. (2) 2 and 3 only. These clauses are closely related and stand on their own. Nor, although are
conjunctions, hence no semicolon.
8. (5) 4 only. The two clauses in 4 are closely related and can stand on their own.
9. (4) 2, 3, and 4 have independent clauses, and they can be joined using a conjunction.
10. (3) 1 and 3 only. Bothe the clauses in these options are closely related and independent clauses, hence
semicolon can be used.
11. (1) Use of the colon is incorrect — option (1) and (4). There are two clauses, hence the comma after the
first clause is incorrect — option (3).
12. (3) In option (1) the semicolon after Malaysia is wrong. Option (2) the semicolon after (A'Famosa) is
wrong. Option (4) semicolons are wrong.
13. (1) The colon in option (2) is wrong. Option (3) is wrong because the list requires commas. 's for friends
in option (4) is wrong. 14.
14. (4) Option (1) is wrong because a comma is required between vast and green (vast and green) Option (2)
is wrong because sister needs an apostrophe. Option (3) is wrong because the comma after held is wrong.
15. (3) Dads' in option (1) is ridiculous. Option (2) has wrong its. Option (4) has cant without an apostrophe.
16. (2) Option (1) is wrong because so is a coordinating conjunction semicolon is wrong; a comma is
required. Option (3) is wrong because brothers needs an apostrophe. Option (4) is wrong because of the
colon.
17. (4) Option (1) is wrong because smart young lady is parenthetical and needs commas. Option (2) is
wrong because of brother's-in-law. Option (3) is wrong because smart young does not need a comma.
18. (1) Option (2) and (3) are wrong because of the comma after way.But is a coordinating conjunction.
Only if an independent clause follows the comma is justified. Option (3) has no apostrophe for Srinivas.
Option (4) is wrong because of the comma after office.
19. (2) Option (1) is wrong because of the colon. Option (3) is wrong because Switzerland needs an
apostrophe. Option (4) is wrong because a list needs commas.
20. (1) Option (2) is wrong because a parenthetical element needs to be demarcated using commas. Option
(3) is wrong because test needs an apostrophe. Option (4) is wrong because his does not need an
apostrophe.

iQue ideas Page 133


15
Questions based on English Grammar and
Usage
I.
In each question, there are five sentences or parts of sentences that form a paragraph. Identify the sentence(s)
or part(s) of sentence(s) that is/are correct in terms of grammar and usage (including spelling, punctuation
and logical consistency). Then, choose the most appropriate option.

A. Charges and countercharges mean nothing


B. to the few million who have lost their home.
C. The nightmare is far from over, for the government
D. is still unable to reach hundreds who are marooned.
E. The death count have just begun.

(1) A only (2) C only (3) A and C (4) A, C and D (5) D only

Answer Option (4) A, C and D are correct. B is incorrect in the singular use of 'million' with few - it should
be few millions. E needs has to link the singular subject the death count. Countercharge as one word is
correct like counterchallenge.
Example 2

A. When I returned to home, I began to read


B. everything I could get my hand on about Israel.
C. That same year Israel's Jewish Agency sent
D. a Shaliach a sort of recruiter to Minneapolis.
E. I became one of his most active devotees.

(1)C and E (2) C only (3) E only (4) B, C and E (5) C, D and E

Answer Option (1) C and E are correct. 1 should be corrected to -returned home". 2 should be corrected to
'get my hands on' - an idiom. 4 should include a comma, or a dash after Shaliach - a sort of... In fragment 3,
That same year like that very year is acceptable.

II. Choose the grammatically correct sentence from among the four options given for each question.

(1) The Excel Company has greatly enhanced it's advertising expense.
(2) The Excel Company has greatly increased its advertising expense.
(3) The Excel Company has greatly enhanced its advertising expense.
(4) The Excel Company has greatly increased it's advertising expense.

Answer (2) It's is an abbreviation for it is or it has, hence eliminated. increase vs. Enhance - Increase implies:
to become progressively greater (as in size, amount, number, or intensity). Enhance implies, rather than size,
to increase or improve in value, quality, desirability, or attractiveness. Hence option 2.

iQue ideas Page 134


III. In each of the following questions a part of the paragraph or sentence has been underlined. From the
choices given to you are required to choose the one, which would best replace the underlined part.

This government has given subsidies to the Navratnas but there is no telling whether the subsequent one will
do.

(1) whether the subsequent government will do so.


(2) if the government to follow will accept the policy.
(3) if the government to follow will adhere to the policy.
(4) no telling whether the subsequent one will do so.

Answer option (4) Option 1, 2 and 3 are eliminated as they do not contain no telling, hence the sentence
cannot continue. Also, 'will do' (original sentence) is incorrect. Hence 4.

HOW TO SOLVE

In question types II and III, compare all the options together to spot the difference in them. For example, in
type II, the first difference is increase vs. enhance. Decide which makes better sense and eliminate the other
options ( 2 and 4) . Compare options 1 and 3. The difference is it 's vs. Its. Make your decision. In type IV
example, by comparing all the 4 options at once, you have to make the decision whether 'no telling' needs to
be removed from the sentence "but there is if ... " as in options 2 and 3, and "but there is whether ..." as in
option 1, do not make sense. Hence Option 4.

However, in type I questions, elimination is possible by working with the options. Look for a fragment ( A,
B. C etc...) that you are sure is correct or incorrect: eliminate options and work forward to your answer.

Though the above methodology can be applied to save time and to avoid confusion, a candidate must be
aware that grammar questions, like vocabulary questions, are 'technical' questions in which the candidate's
awareness of the principles of grammar is tested. Hence it becomes necessary to learn the principles of
grammar while preparing for the test. There are innumerable grammar books available in the market. The
coaching institutes for competitive examinations also provide abundant notes. However, one must make sure
that the common areas of errors (discussed in this book) are not missed.

While solving these questions make sure that you have read the explanation to each question, because these
explanations will make you aware of most of the principles that you are expected to learn. Hence, it is all
right to get the answers wrong, if you consider these questions learning tools.

iQue ideas Page 135


Practice Exercise - I

Choose the best option to replace the underlined part:

1. The number of foreigners going to India for fertility treatments, specially surrogacy. has dramatically
increased in recent years.

(1) especially surrogacy, have dramatically increased in past years.


(2) especially surrogacy, has dramatically increased in recent years.
(3) specially surrogacy, have dramatically increased in recent years.
(4) specially surrogacy, have shown dramatic improvement in recent years.

2. The LHC is a magnificent engineering project, who's many gee-whiz" features have been widely
reported.

(1) whose many "gee-whiz" feature have been widely reported.


(2) who's "gee-whiz" features has been reported widely.
(3) whose many "gee-whiz" features has been widely reported.
(4) who's many "gee-whiz" features have been widely reported.

3. Venezuelan President Hugo Chavez, outspoken as usual, compared the choice of Liu for the Nobel Prize
with Obama.

(I) compared the choice of Liu for Nobel Prize with that of Obama.
(2) compared the choice of Liu with Obama for the Nobel Prize.
(3) compared the choice of Liu for the Nobel Prize to that of Obama.
(4) compared the choice of Liu and Obama to the Nobel Prize.

4. The OECD expects average GDP growth among their mostly rich members to slow from 2.8% in 2010
and 2.3% in 2011 before bouncing back to 2.8% in 2012.

(1) their most rich members to slow from 2.8% in 2010 and 2.3% in 2011,
(2) their mostly rich members to slow from 2.8% in 2010 and 2.3% in 2011,
(3) its most rich members to slow between 2.8% in 2010 to 2.3% in 2011,
(4) its mostly rich members to slow from 2.8% in 2010 to 2.3% in 2011,

5. A sphere cannot be represented on a flat plane without distortion, which will mean that all map
projections will distort in one way or another.

(1) which means all map projections distort in one way or another.
(2) that means all map projections will distort one way or another.
(3) which will mean that all map projections distorts one way or other.
(4) which means all map projections will distort in one way or others.

6. The blame for rising obesity rates have been pinned on man thin s including having more calorific diet,
the spread of processed food, a lack of exercise and modern man's generally more stressful lot.

(1) has been pinned on many things, including a more calorific diet,
(2) have been pinned to many things, including more calorific diets,

iQue ideas Page 136


(3) have been pinned on many things, including a calorie rich diet,
(4) has been pinned on many things: having a more calorific diet,

7. The tendency to eat late, though it has never been tested properly, many nutritionists believe, as a factor
in putting on weight.

(1) many nutritionists believe, to be a factor in putting on weight.


(2) many nutritionists believe factor in putting on weight.
(3) is believed by many nutritionists to be a factor in putting on weight.
(4) is believed by many nutritionists as a factor in putting on weight.

8. (Pay attention to punctuation also)


Of the billion or so Windows machines, some 63% still uses old XP version launched in 2001. in
America. the proportion of XP users are over 80%.

(1) some 63% still use the old XP version launched in 2001, in America the proportion of XP users are
(2) some 63% still uses the old XP version launched in 2001; in America, the proportion of XP users are
(3) some 63% still use the old XP version launched in 2001 in America, the proportion of XP users is
(4) some 63% still use the old XP version launched in 2001; in America, the proportion of XP users is

9. Though subjected to varying influences, the distinctive features of China's artistic activity possess a
unity which is quite as complete as that of our Western art.

(1) possess a unity which is quite as complete as that of our Western art.
(2) possesses a unity which are quite as complete as our Western art.
(3) possesses a unity which is quite complete like our Western art.
(4) possess a unity which is quite complete like our Western art.

10. To know by parts is science, knowing the whole as a whole is philosophy.

(1) Knowledge of parts is science,


(2) Knowing by parts is science,
(3) To know partially is science,
(4) To know by parts is science,

11. In the fission reactions that nuclear power generation rely on today, heavy elements such as uranium
breaks into smaller ones.

(1) relies on today, heavy elements such as uranium break into smaller ones.
(2) rely on today, heavy elements such as uranium break into smaller ones.
(3) relies on today, heavy elements such as uranium breaks into smaller ones.
(4) rely on today, heavy elements such as uranium breaks into smaller ones.

12. The nature of our party system has changed in the past few decades, and the lesson of the past decade
had been that the party that did not hold the White House had every incentive to refuse to co-operate
with the president's party.

(1) was that the party that does not hold the White House had every incentive
(2) is that the party that does not hold the White House has every incentive

iQue ideas Page 137


(3) had been that the party that had not held the White House had every incentive
(4) had been that the party that did not hold the White House had every incentive

13. Looking at the path of German GDP and the fact that total private sector hours didn't fall by that much, it
becomes clear that Germany's unique recession and recovery was about more than just its choice in
labour market policies.

(1) Looking at the path of German GDP and that total private sector hours didn't fall much,
(2) Looking at the path of German GDP and the fact that total private sector hours didn't fall much,
(3) When one looks at the path of German GDP and the fact that total private sector hours didn't fall
much,
(4) When one is looking at the path of German GDP and that total private sector hours didn't fall much,

14. The Beijing Language and Culture University Press, the largest publisher of Chinese-language
textbooks, says South Korea and Japan are their biggest customers.

(1) say South Korea and Japan are their biggest customers.
(2) says South Korea and Japan are it's biggest customers.
(3) says South Korea and Japan are their biggest customers.
(4) says South Korea and Japan are its biggest customers.

15. (Pay attention to punctuation also)


There are two pieces to Mr Keats' argument; that English is open-source as other languages are not; and
that that is why it has spread.

(1) Mr Keats' argument, that English is open-source as other languages are not and that
(2) Mr Keats' argument: that English is open-source as other languages are not, and that that
(3) Mr Keats' argument — that English is open-source as other languages are not — and that
(4) Mr Keats's argument; that English is open-source as other languages are not; and that that

16. (Pay attention to punctuation also)


Partnership, he said, was a "two-way street", adding of Karzai, "We have to listen and learn. But he's got
to listen to us as well".

(1) "two-way street," adding of Karzai: "We have to listen and learn. But he's got to listen to us, as well."
(2) "two-way street"; adding of Karzai: "We have to listen and learn - but he's got to listen to us as well".
(3) "two-way street,"adding of Karzai; "We have to listen and learn; but he's got to listen to us as well."
(4) "two-way street", adding of Karzai, "We have to listen and learn. But he's got to listen to us, as well."

17. There has been heightened concern among India's civil society about the threats for the environment
caused by the pressures of development and industrialisation.

(1) within the civil society of India about the threats to the environment
(2) within India's civil society about the threats to the environment
(3) among India's civil society about the threats to the environment
(4) for India's civil society about the threats to the environment

18. Unable to cope up with the losses 1.5 lakh workers were laid off by the private sector.

iQue ideas Page 138


(1) Unable to cope up with the losses, the private sector lay off 1.5 lakh workers.
(2) Unable to cope with the losses, 1.5 lakh private sector workers were laid off.
(3) Unable to cope with the losses, the private sector laid off 1.5 lakh workers.
(4) Unable to cope with the losses, the private sector had lain off 1.5 lakh workers.

19. Dasara is now celebrated across Karnataka as a State festival with an exquisitely unique blend of the
religious and the secular.

(1) festival with an exquisitely unique blend of religions and secularism.


(2) festival with an exquisite blend of unique religions and secularism.
(3) festival with an unique blend of religions and the secular.
(4) festival with a unique blend of the religious and the secular.

20. If I was you, then I would not do that.

(1) If I were you, I would not do that.


(2) If I were you, then I would not have done that.
(3) If I was you, then I would not do that.
(4) If I had been you. I would not have done that.

In each of the following questions, there are four sentences or parts of sentences that form a paragraph.
Identify, the sentences) or part(s) of sentence(s) that is/are correct in terms of grammar and usage (including
spelling, punctuation and logical consistency). Then, choose the most appropriate option.

21. A. Sometimes, we think we are only the people with problems.


B. Our problems override other considerations: we can't think beyond our own vexations.
C. When we see a small dampness on our wall we scurry to get them repaired and corrected,
D. but we don't think of people whose homes have been washed away by the rage of the rain gods.
(1)A and B (2) B and D (3) C only (4) C and D

22. A. Chen Jianping repeated again on twitter something her fiancé said
B. about anti-Japan protests getting violent. and added, "Charge, angry youth!"
C. For this she was sentenced one year in a labor camp.
D. Twitter is banned in China, but its easy to use blocked sites with simple internet tricks.
(1)A only (2) B only (3) C only (4) B and D

23. A. The Great Pacific Garbage Patch, is gyre of marine litter in the central North Pacific Ocean.
B. The patch extends over a very wide area. with estimates ranging from area
C. the size of the state of Texas to one larger as the continental United States:
D. however, the exact size is unknown.
(1)A andD (2) B and C (3) C only (4) D only

24. A. At media agency Mindshare's Mumbai office


B. sit a team of ninety-three individuals
C. who's only focus is the consumer giant Hindustan Unilever.
D. In fact, this unit occupies an area the size of most mid-sized agency's in this country.
(1) A only (2) A and C (3) A and D (4) B and C
25. A. One of the classroom exercise I conduct with my Persuasive Communication students
B. at IIMA is taken from Kurukshestra war.

iQue ideas Page 139


C. I ask a student to be Kunti, other to be Kama.
D. Kunti has to persuade Karna to leave Duryodhana and join the Pandavas .
(1) A only (2) A and B (3) C and D (4) D only

26. A. The debasement of currencies. particularly US dollar, has given gold.


B. perceived a monetary unit, a leg up with the metal slated to record it's tenth annual gain,
C. the longest since 1920's. With the US Federal Reserve set to pump $600 billion
D. through purchase of debt. more money is expected to chase commodities such as gold.
(1)A and D (2) A and C (3) C and D (4)D only

27. A. Three years have passed since Dr. Ira Kalish , global research head. Deloitte. visited India.
B. But the changes in modern trade are all too apparent to Kalish.
C. There has been well-publicised expansions and scale backs.
D. Some have taken a fall, and others picked pace.
(1)A and B (2) B and C (3)C and D (4) D only

28. A. The world of luxury is innovating in ways which strike an emotional chord with the effluent buyer.
B. And the elite are not complaining.
C. Vishakha Doshi Director Marketing & Communication Entrack distributors for Montblanc India says
that people identify with the core values of such legends.
D. "Each of the limited edition luxury, writing instrument pays tribute to icons and is a collector's pride”
(1) A, C and D (2) B only (3) All are correct (4) All are incorrect

29. A. NRIs does not include a person who has gone out of India on employment, business or vocation,
B. or any other purpose for an uncertain period.
C. Also, a person who has come to stay in India other than employment.
D. business or vocation, or on any other purpose for an uncertain period.
(1)A.C and D (2)B only (3) All are correct (4) All are incorrect

30. A. Companies will have to compete not only on prices to the consumer
B. but also remunerative prices to farmers.
C. This is possible only if they invest on better seeds. fertilisers and farm practices.
D. The farmer has to get higher returns from his land and they need support to do this.
(1)A only (2) B only (3) A and D (4) C and D

31. A. Its one thing to win power. another to wield it.


B. Two disspirited leaders met in Delhi this week.
C. President Obama was chastened by dramatic electoral losses in the US Congress and Prime Minister
Manmohan Singh disheartened by never ending corruption scandals.
D. Both seemed to have forgot the fundamentals of what created their respective democracies.
(1)A and B (2) C and D (3) A and C (4) B and D

32. A. Just like America's founding fathers were obsessed with liberty.
B. so were Indias founders deeply attached to dharma--so much so that they placed
C. the dharma-chakra in the middle of Indian flag.
D. The Congress party still does not realize how much it has diminished by the relentless series of
corruption scandals.
(1) All are correct (2) All are incorrect (3) A and D (4) B and C

iQue ideas Page 140


33. A. Her words had had the desired effect
B. and when he had reached her place, he stopped for longer than he had intended to.
C. Throughout the drive to her home.
D. he had kept thinking about how much he would miss her.
(1) All are correct (2) All are incorrect (3) A and D (4) B and C

34. A. Couldn't make it to Arnab's show tonight. Nor to Rahul's ( Headlines Today).
B. The topic on both were the same - it's the big story about Raakhi Sawant's
C. show timing getting changed to a later slot.
D. But hopefully kids won't be awake to watch this rubbish. So... it's all good.
(1) All are correct (2) All are incorrect (3) A. C and D (4) A, B and C

35. A. Liquid Comics. an entertainment company found by three Indians


B. have announced the digital iPad release of "Untouchable".
C. a graphic novel with a supernatural horror twist
D. exploring various themes of racial prejudice during the British Raj.
(1)A and B (2) C and D (3)A andC (4) B and D

36. A. That the US and the UK are the most popular destination for management studies is not big news.
B. But the finding of a survey conducted by the Graduate Management Admission Council have thrown
up some surprises.
C. Number 4 on the list of the Top 10 preferred destination for B-school aspirants is India.
D. According to experts, India's foray into the elite league has a lot to do with the emergence of -
institutions such as the Indian School of Business. Hyderabad.
(1)A only (2) A and D (3)B and C (4) D only

37. A The fact is that Indian cities, unlike many cities abroad, lack in leadership.
B. The mayor. supposed to be the first citizen of the city,
C. has neither the time or the power to steer the city's affairs.
D. His tenure is a glorious one year in cities like Delhi and Bangalore or a extended 20 months in some
other cities.
(1) A only (2) B only (3) B and C (4) D only

38. A. CPM's diminished clout today is surely thanks the mismatch


B. between adherence to ideological purity and the inability to follow through in practice.
C. Survival in politics is to the nimble-footed.
D. Hence 'revolutionary' communists have been outrun by social democrats.
(1)A only (2) B and D (3) C and D (4) A and D

39. A. Forget the fact that the Chinese economy has grown faster than India.
B. Forget the fact that in terms of infrastructural development be it power plants
C. and highways or hospitals and schools China is way ahead.
D. Perhaps the most fundamental difference between China and India is the vulnerability of each to
terror.
(1) A only (2) B and D (3) C and D (4) D only

40. A. The organization that benefitted from the scam were the Internet Service Providers.
B. Rather than a risky military operation, American officials hope to isolate Mr.Khameini.
C. Shahrukh Khan plus Kareina Kapoor mean a hit film.

iQue ideas Page 141


D. 93.5, the first FM station, launched a few years ago turning teenagers to radio listeners.
(1) All are correct. (2) All are incorrect. (3) A and B (4) C and D

41. A. Mild showers turning gusty increasing to a storm today.


B. Any mention and explanation of the idea is likely to be dismissed.
C. Since becoming a Project Manger in April, I have been working in Bangalore.
D. It was a great concept and execution of the project.
(1) A only (2) B only (3) C only (4) D only

42. A. The cause of the power outage is not known to us.


B. These were the circumstances when I made that the decision to quit.
C. The King rewarded him for a commitment evidenced by years of loyal service.
D. He cut off his long hair, which made him look younger.
(1)A and D (2) A and C (3) B and C (4) B and D

43. A. Diseases when suffered by poor appear


B. to be more deadly than when they afflict better of people.
C. The less affluent, indeed, are more likely to die
D. off diseases than people who are rich.
(1)A only (2) D only (3) B and C (4) C only

44. A. In autumn of 1940, Warsaw's Jewish population.


B. swollen by forced immigration, amounted to nearly 450,000 people,
C. all of those walled into an area covering less than four square kilometres.
D. By early 1942 about 83.000 died from hunger.
(1) D only (2) B only (3) B and D (4) C only

45. (Pay attention to punctuation also)

A. The first human settlements in Myanmar appeared some 11000 years ago.
B. The stone and fossilized-wood tools used by these people
C. have been named Anyathian, from Anyatha (another term for Upper Burma);
D. little else, however, is known of these people.
(1) B. C, and D (2) A. C. and D (3) A, B. and D (4) A, B, and C

46. A. With the expansion of Indian commerce in Southeast Asia


B. between the 1st to the 4th centuries AD, Myanmar's importance increased.
C. Indian merchants and seamen came to Myanmar as traders
D. rather as conquerors or colonists.
(1)A and D (2) A and C (3) B only (4) B and C

47. A. It's some time since anyone accused GM of making a good move.
B. The company surrendered its title as the worlds top-selling carmaker
C. to Toyota this year, in part because GM underestimated drivers appetite
D. for leaner, greener cars — a desire filled spectacularly by Toyotas Prius.
(1)B and C (2) A and D (3) C and D (4) None of the above (are correct)

48. (Pay attention to punctuation also)


A. Dhohi Ghat is the story of 4 characters: Arun. the famous lonely painter (Aamir):

iQue ideas Page 142


B. Munna. the dhohi who wants to be a star (Prateik),
C. Shai, the banker on a sabbatical in India (Monica).
D. and Yasmin, the recently married Muslim girl who records her life in Mumbai for her brother back
home (Kriti).
(1)A only (2) B. C and D (3) A and D (4) D only

49. A. All over the world. children study math and science
B. in the hope for becoming Bill Gates. But the hero-entrepreneur myth can be dangerous,
C. because what healthful economies really need is not more entrepreneurs,
D. but rather enough people who are willing to work for them.
(1) A only (2) B, C and D (3) A and D (4) D only

50. A. Not only the Earth's surface is warming.


B. but the troposphere is heating up too.
C. In review of four decades of data on troposphere temperatures,
D. the scientists found that warming was occurring in troposphere layer.
(1)A only (2) B only (3) A and B (4) D only

In the following questions choose the grammatical& correct sentence from among the four options given for
each question.

51. (1) It looks like it is going to rain this afternoon.


(2) It looks as if it is going to rain this afternoon.
(3) It looks as if it is going to rain today afternoon.
(4) It looks like it is going to rain today afternoon.

52. (1) A belief that he is humble and being really humble is what makes him so likeable.
(2) A belief that he is humble and being really humble are what makes him so likable.
(3) Believing that he is humble and really being humble is what makes him so likeable.
(4) Believing that he is humble and really being humble are what makes him so likable.

53. (1) This book left by who knows whom? has been lying here for days.
(2) This book — left by who knows whom — has been lying here for days.
(3) This book left by — who knows who? — has been lying here for days.
(4) This book left by — who knows who — has been lying here for days.

54. (1) I can understand my sister better than my mother.


(2) I can understand my sister more than my mother.
(3) I can better understand my sister than my mother.
(4) I can understand my sister better than I can understand my mother.

55. (1) She said that Raj had a fall last week and he had been injured.
(2) She said that Raj had fallen the previous week and he had injured.
(3) She said that Raj had had a fall last week and he was injured.
(4) She said that Raj had had a fall the previous week and he had been injured.

56. (1) In speaking with the Minister, he denied that there were any payoffs involved in the deal.
(2) While speaking with the Minister. we were informed that there were no payoffs in the deal.
(3) While speaking with the Minister, the Minister denied that there were any payoffs in the deal.

iQue ideas Page 143


(4) In speaking with the Minister, he denied to the journalists that there were any payoffs in the deal.

57. (1) She intended to get married to him, but then she goes and breaks relations with him.
(2) She intends to get married to him, but then she goes and breaks relations with him.
(3) She intended to get married to him, but then she went and broke relations with him.
(4) She intended to get married to him, but then she will go and break relations with him.

58. (1) Of all the students in the class whom do you think the teacher likes the most?
(2) Of all the students in the class who do you think the teacher likes the most?
(3) Who do you think the teacher likes the best of all students in the class?
(4) In the class, who do you think the teacher likes the most of all the students?

59. (1) In 2009, she was promoted to the position of General Manager, Systems.
(2) She was promoted in 2009 to a General Manager, Systems post.
(3) She was promoted to a General Manager, Systems position in 2009.
(4) She was promoted in 2009 to General Manager, Systems.

60. (1) If I had any idea, I would have warned you immediately.
(2) If I had had any idea, I would have warned you immediately.
(3) If I have any idea, I would warn you immediately.
(4) 1 had had any idea, I would warn you immediately.

61. (1) It is recommended that the college acquire a large enough playground.
(2) It is recommended that the college acquires a large enough playground.
(3) It is recommended that the college must acquire large enough playground.
(4) It is recommended that the college acquires a large enough playing ground.

62. (1) The Beatles was an English rock band, formed in Liverpool in the 1960, and one of the most
commercially successful and critically acclaimed act in the history of popular music.
(2) The Beatles was an English rock band, formed in Liverpool in 1960, and was one of the most
commercially successful and critically acclaimed acts in the history of popular music.
(3) The Beatles were an English rock band, formed in Liverpool in 1960, and was one of the most
commercially successful and critically acclaimed acts in the history of popular music.
(4) The Beatles were an English rock band, formed in Liverpool in 1960, and one of the most
commercially successful and critically acclaimed acts in the history of popular music.

63. (1) More than one student has expressed their support for the campaign.
(2) More than one student have expressed their support for the campaign.
(3) More than one student has expressed his/her support for the campaign.
(4) More than one student have expressed his/her support for the campaign.

64. (1) The majority of the population in our country today is young below 30 year of age.
(2) The majority of the population in our country today are young below 30 year of age.
(3) A majority of population in our country today is young below 30 year of age.
(4) A majority of population in our country today are young below 30 year of age

65. (1) The labour union is expected to issue a statement to the management on Monday to request an
increase in the coffee break's duration.

iQue ideas Page 144


(2) On Monday, the labour union is expected to issue a statement requesting the management to an
increase in the coffee-break's duration.
(3) On Monday, the labour union is expected to request the management for an increase in the duration
of the coffee break.
(4) The labour union is expected to issue a statement on Monday to the management in order to request
to increase the duration of the coffee-break.

66. (1) I had an intuition that our plan was not going to be successful.
(2) I had an intuition that our plan is not going to be successful.
(3) I had an instinct that our plan was not going to be successful.
(4) I had an instinct that our plan is not going to be successful.

67. (1) The museums, the markets, and the restaurants make our city a place to stay and not merely a place
to come to.
(2) Museums, markets, and restaurants make our city a place to stay in and not merely a place to come.
(3) The museums, the markets, and the restaurants make our city a place to stay at and not merely a place
to come to.
(4) The museums, the markets, and the restaurants make our city a place to stay in and not merely a
place to visit.

68. (1) It is through the co-operation of people like yourself that we had been able to achieve this.
(2) It was through the co-operation of people like yourself that we have been able to achieve this.
(3) It is through the co-operation of people like you that we have been able to achieve this.
(4) It was through the co-operation of people like you that we have been able to achieve this.

69. (1) She likes all kind of foods, but Chinese and Thai is what she likes best.
(2) She likes all kinds of food, but Chinese and Thai are foods she likes best.
(3) She likes all kind of foods, but Chinese and Thai are foods she likes the best.
(4) She likes all kinds of food, but Chinese and Thai are what she likes the best.

70. (1) For the purpose of this contract "vendors" means all those to whom services are subcontracted.
(2) For the purpose of this contract "vendors" mean all those to whom services is subcontracted.
(3) For the purpose of this contract "vendors" mean all those to who services are subcontracted.
(4) For the purpose of this contract "vendors" means all those who's services are subcontracted.

71. (1) CPL's personalised instruction ensures that their faculty are there to help you to reach your goals.
(2) CPL's personalised instruction ensures that the members of its faculty are there to help you reach
your goals.
(3) CPL's personalised instructions ensures that each of their faculty is there to help you reach your
goals.
(4) CPL's personalised instructions ensure that their faculty is there to help you reach your goals.

72. (1) They know like us that a few lines of code could wreak as much havoc as a few handfuls of bombs.
(2) They know like us that a few lines of code could wreck as much havoc as a few handsful of bombs.
(3) They know, as do we, that a few lines of code could wreak as much havoc as a few handfuls of
bombs.
(4) They know, as do we, that a few lines of code could wreck as much havoc as a few handsful of
bombs.

iQue ideas Page 145


73. (1) He worked harder and harder, but there was no way he could catch up with his friend, Raj.
(2) He worked, harder and harder, but there was no way he could catch up, with his friend Raj.
(3) He worked harder, and harder, but there was no way he could catch up with his friend, Raj.
(4) He worked harder and harder, but there was no way he could catch up with his friend Raj.

74. (1) Manish, our math teacher, said he would meet us here; however, I doubt whether he will be able to
come, especially if it rains.
(2) Manish, our math teacher, said he would meet us here, however, I doubt whether he will be able to
come, especially if it rains.
(3) Manish our math teacher said, he would meet us here, however, I doubt whether he will be able to
come, especially if it rains.
(4) Manish, our math teacher said, he would meet us here; however, I doubt whether he will be able to
come, especially if it rains.

75. (1) Frost's poem "Stopping By Woods on a Snowy Evening" contains these haunting lines: "The woods
are lovely, dark and deep / But I have promises to keep."
(2) Frost's poem, Stopping By Woods on a Snowy Evening, contains these haunting lines "The woods
are lovely, dark and deep / But I have promises to keep."
(3) Frost's "poem," Stopping By Woods on a Snowy Evening, contains these haunting lines: The woods
are lovely, dark and deep / But I have promises to keep.
(4) "Frost's poem Stopping By Woods on a Snowy Evening," contains these haunting lines: "The woods
are lovely, dark and deep / But I have promises to keep."

iQue ideas Page 146


16
Includes actual CAT questions
Practice Exercise - II

Exercise 1

In each question, there are five sentences or parts of sentences that form a paragraph. Identify the sentence(s)
or part(s) of sentence(s) that is/are correct in terms of grammar and usage (including spelling, punctuation
and logical consistency). Then, choose the most appropriate option.

1. A. In 1849, a poor Bavarian imigrant named Levi Strauss


B. Landed in San Francisco, California,
C. at the invitation of his brother-in-law David Stern
D. owner of dry goods business. E. This dry goods business would later became known as Levi Strauss
& Company.
(1)B only (2) B and C (3) A and B (4) A only (5) A, B and D

2. A. In response to the allegations and condemnation pouring in,


B. Nike implemented comprehensive policy changes in their labour policy.
C. Perhaps sensing the rising tide of labour concerns,
D. from the public would become a prominent media issue,
E. Nike sought to be a industry leader in employee relations.
(1)D and E (2) D only (3) A and E (4) A and D (5) B, C and E

3. A. Charges and countercharges mean nothing


B. to the few million who have lost their home.
C. The nightmare is far from over, for the government
D. is still unable to reach hundreds who are marooned.
E. The death count have just begun.
(1)A only (2) C only (3) A and C (4) A, C and D (5) D only

4. A. I did not know what to make of you.


B. Because you'd lived in India, I associate you more with my parents than with me.
C. And yet you were unlike my cousins in Calcutta, who seem so innocent and obedient when I visited
them.
D. You were not curious about me in the least.
E. Although you did make effort to meet me.
(1)A only (2) A and B (3) A and E (4) D only (5) A and D

5. A. Relations are improving between the United States and Libya.


B. President Bush telephoned the Libyan leader, Col. Muammar el-Qaddafi,
C. on Monday, after Libya has paid about $1.5 billion
D. to State Department to clear up terrorism-related claims E. from bombings and hijackings during the
1980's.
(1)A only (2) A and B (3) A and E (4) C only (5) C and D

iQue ideas Page 147


6. A. I've had a intense year.
B. Due to the book launch and movie release, I've been out there on display,
C. focussing too much on the external world.
D. Its time to look within, get in touch with myself again and create something new.
E. As part of that, I have to stop the blog, interacting with media and other events.
(1) A only (2) A and B (3) A and E (4) C only (5) B and C

7. A. Now setting up Microsoft Project Manager to work with your company's existing
B. IT infrastructure can be a complicated task.
C. Systems will have to replace to make them compatible, space has to be allocated
D. for new hardware and there are considerable cost in implementing a desirable system.
E. A data center has to be setup and your IT department has to complete all this in addition to their
normal duties.
(1) A only (2) B only (3) B and E (4) C only (5) B and D

8. A. We are heading into an age in which jobs are


B. likely to be invented and made obsolete faster and faster.
C. The chances of today's college kids working in the same jobs for the same companies for their whole
careers is about zero.
D. In such a age, the greatest survival skill you can have is the ability to learn how to learn.
E. The best way to learn how to learn is to love to learn, and the best way to love to learn is to have
teachers who inspire.
(1)Aonly (2) A and B (3) A , B and E (4) B and C only (5) C and D

9. A. A New Zealand company called HortResearch is a world renowned fruit science company,
B. and they develop a unique and innovative fruit and food products for sustainability.
C. One of their previous innovative technologies were in developing a bio sensor to measure the
hormone levels in saliva.
D. This can be monitored in real time and also adopted to sports.
E. Right now, it is investigating technologies for measuring biochemical indicators of stress and
performance in real time.
(1) A only (2) A and B (3) A and E (4) E only (5) A, C and E

10. A. Maragadavalli was running her father's household ever since her mother had died when she was 13.
B. It took a long time for her family to settle down after that, and she was 26 by the time she had got
married.
C. With 13 years of running a household under her belt, she married into a family meticulously run by
mother-in-law.
D. She liked the fact that there were servant maids to help with the housework.
E. She had very less to do around the house except cut vegetables and wait for her husband to come back
from work.
(1) A only (2) A and D (3) A and E (4) D only (5) A, C and E

iQue ideas Page 148


Exercise 2

Each question consists of four sentences on a topic. Some sentences are grammatically incorrect or
inappropriate.
Select the option that indicates the grammatically correct and appropriate sentence(s).

1. A. when virtuoso teams begin their work, individuals are in, and group consensus is out.
B. As project progresses, however, the individual stars harness themselves to the product of the group.
C. Sooner or later, the members break through their own egocentrism and become a plurality with
single-minded focus on the goal.
D. In short, they morph into a powerful team with a shared identity.
(1) A and C (2) A and D (3) B and D (4) A,C and D (5) B, C and D

2. A. Large reductions in the ozone layer, which sits about 15-30 km above the Earth, take place each
winter over the polar regions, especially the Antarctic, as low temperatures allow the formation of
stratospheric clouds that assist chemical reactions breaking down ozone.
B. Industrial chemicals containing chlorine and bromine have been blamed for thinning the layer because
they attack the ozone molecules, making them to break apart.
C. Many an offending chemicals have now been banned.
D. It will still take several decades before these substances have disappeared from the atmosphere.
(1) D only (2) B and D (3) A and D (4) A and C (5) A, B and C

3. A. The balance of power will shift to the East as China and India evolve.
B. Rarely the economic ascent of two still relatively poor nations has been watched with such a mixture
of awe, opportunism, and trepidation.
C. Postwar era witnessed economic miracles in Japan and South Korea, but neither was populous enough
to power worldwide growth or change the game in a complete spectrum of industries.
D. China and India, by contrast, possess the weight and dynamism to transform the 21st-century global
economy.
(1)A, B and C (2) A and D (3) C (4) C and D (5) A and C

4. A. People have good reason to care about the welfare of animals.


B. Ever since Enlightenment, their treatment has been seen as a measure of mankind's humanity.
C. It is no coincidence that William Wilberforce and Sir Thomas Foxwell Buxton, two leaders of the
movement to abolish the slave trade, helped found the Royal Society for the Prevention of Cruelty to
Animals in 1820s.
D. An increasing number of people go further: mankind has a duty not to cause pain to animals that have
the capacity to suffer.
(1)A and D (2) B (3) A and C (4) C and D (5) B and D

5. A. Unlike the rest of the world, it's not difficult to find reasons for the importance of foreign policy in the
US electoral arena.
B. The country founded by immigrants still has large first-generation immigrant population who are
directly or indirectly connected with their land of origin.
C. The electoral process has initiated a debate about the role of the US in world affairs and at the same
time the world is trying to understand what change the new president will bring in with his world view.
D. US's aspirations to be a world leader makes it play an important role in world affairs.
(1)A and C (2) B and D (3) A and B (4) C (5) C and D

iQue ideas Page 149


6. A. Mere dependence on military means completely failed in places like Afghanistan and Pakistan.
B. There is no substitute to political intervention, and micro realities of these hotspots needs to be
understood.
C. For instance, the war against terror failed to make gains even after the electoral verdict in favour of
progressive National Awami Party during the recent Pakistan elections.
D. Political problems need political solution and not military solutions.
(1) A , B and C (2) B and C (3) A and C (4) C (5) C and D

7. A. This debate among secularism and pseudo-secularism underlines one common trait.
B. That is vote bank politics.
C. In their rush to gain votes and thereby power, politicians of all colours strive to outwit each another
by conveniently shifting their stand even if that means hurting the religious sentiments of one group or
other.
D. From Shah Bano to Salman Rushdie to Taslima Nasreen, the so-called secularists would always
ensure that the sentiments of minorities are respected even at the risk of making mockery of the word
"secular" that they themselves inserted in the Constitution.
(1)A and C (2) B and D (3) A, B and D (4) C (5) C and D

8. A. These relief camps are crammed as the supplies and space are pushed to limit.
B. An additional 1,500 people are arriving every day, as army and Border Security force have been
pushed into action.
C. These additional people further strain meager resources within the relief camps.
D. The number of people still marooned is unknown.
(1)A and B (2) B and D (3) A and C (4) D (5) A,C and D

9. A. I left at 7 am and it took me 12 hours to reach the district block office of Araria.
B. Araria borders Nepal, and is one of the worst affected district by the current Kosi Floods.
C. The Government has one coordination office only for the relief effort meeting the needs of the two
districts —Araria and Saupal.
D. Approximately 8,000 people from Araria and Saupal are living in 15 relief camps.
(1) D only (2) B and D (3) A and D (4) A and C (5) A, B and C

10. A. What is important is that we need to realise that life is not one thing or the other.
B. Its not black or white.
C. Its a blend — a beautiful blend of so many things, so many moments, there are so many emotions.
D. If we talk about failure, we also need to talk about success.
(1)A and D (2) B and C (3) A and C (4) C and D (5) A,C and D

iQue ideas Page 150


Exercise 3

In the following, there are sentences or parts of sentences labelled A, B , C, D etc.. . Choose for your answer
the fragment that carries an error. Ignore punctuation errors if any.

1. (1) In these blogs, I've laughed, spit out my coffee,


(2) cursed, cried, learned invaluable lessons
(3) about work and motherhood, and fell in love
(4) with hundreds of people I've never met.
(5) No error.

2. (1) Thank you all for conducting a landmark experiment —


(2) sharing what really goes on
(3) inside our heads when it comes
(4) to balancing (and unbalancing) working,
(5) raising kids, and living our lives.

3. (1) And its equally important to spread the news about companies that treat working parents
(2) with fairness and respect, so that we all can try to work at these companies,
(3) and send a message to others treating working parents
(4) justly is a valuable employee recruitment and retention advantage.
(5) No error.

4. (1) As a kid, my father used to take me to a joint


(2) called Roosters that served two eggs,
(3) two pancakes, two strips of bacon, two pieces of toast
(4) and all the butter patties and coffee
(5) one could consume for $2.22.

5. (1) Over the course of more than 500 columns and 100,000 comments,
(2) I've learned a lot about how deeply we all care
(3) about juggling work and family,
(4) but I've never learned what
(5) any of you looks like.

6. (1) I've been in Delhi for four days now


(2) and so far the stay had been uneventful,
(3) barring a stray incident
(4) where I walked into a lamppost and then walked around Connaught Place
(5) with a bloody nose.

7. (1) To be a citizen is to possess the rights


(2) enshrined in our constitution,
(3) and equally, the obligation of duty.
(4) When one citizen upholds an individual right for another,
(5) he enacts his duty to himself.

8. (1) The analyses of data collected shows no evidence


(2) that the vaccine increased the risk

iQue ideas Page 151


(3) of any serious complications,
(4) according to the federal
(5) Centers for Disease Control and Prevention.

9. (1) This study takes its place alongside earlier research showing
(2) that kids who are allowed to serve their own food
(3) take smaller portions than they would typically be served
(4) and ate 25 percentage less, and that kids tend to eat more
(5) when the portions on their plates have been double.

10. (1) A growing body of evidence is supporting the "use-it-or-lose-it" hypothesis of aging well.
(2) It says staying mentally active helps keep the brain spry.
(3) So far, research has focused on reading,
(4) crossword puzzles,
(5) and learning languages.

11. (1) In a sense we are being pushed back


(2) to a 1984-like scenario
(3) when scare-mongering
(4) became the base for vote-gathering.
(5) No error.

12. (1) Sachin is the one who has the maximum number of runs
(2) in ODI and also in Test cricket
(3) and he has scored the highest
(4) numbers of centuries in both
(5) types of the game.

13. (1) Sixty years is too small a period to acquire new civili sational traits
(2) and to mould our DNA. When that happens, we will automatically
(3) realise the importance of the rule of law, the true meaning of freedom and democracy
(4) and then we will behave like citizens who will not allow anyone
(5) to sell their votes, and MPs will not be on sold.

14. (1) The problem with the mainstream Indian Left


(2) is that though they are honest and committed bunch of politicians but are
(3) unwilling to look inward , to find out what is wrong
(4) with Marxism in theory and in practice.
(5) No Error.

15. (1) Mao knew that communism in its classical form


(2) will not gel into agrarian Chinese society .
(3) Hence he redefined Marxism so that it would be
(4) in sync with local needs and
(5) practiced it in a way that suited China.

iQue ideas Page 152


Exercise 4

the following questions, each sentence has been divided into our parts, marked A, B, C and D. Identify that
part of the sentence which needs to be changed for the sentence to be grammatically correct. 1995 CAT.

1. (1) Almost all school teachers insist that


(2) a student's mother
(3) is responsible for the student's conduct
(4) as well as his dress.

2. (1) In the forthcoming elections


(2) every man and woman
(3) must vote for the candidate
(4) of their choice.

3. (1) If one has to decide


(2) about the choice of a career
(3) you should choose that option
(4) which is really beneficial.

4. (1) It is essential that diseases like tuberculosis


(2) are detected and treated
(3) as early as possible in order to
(4) assure a successful cure.

5. (1) The Bombay Police have found


(2) the body of a man
(3) who they believe to be
(4) the prime suspect in the murder case.

In each of the questions, four different ways of presenting an idea are given. Choose the one that conforms
most closely to Standard English Usage (2003 Leaked CAT)

6. (1) The running of large businesses consist of getting somebody to make something that somebody else
sold to somebody else for more than its cost.
(2) The running of large businesses consists of getting somebody to make something that somebody else
will sell to somebody else for more than it costs.
(3) The running of large businesses consists of getting somebody to sell something that somebody else
made for more than it cost.
(4) The running of large businesses consist of getting somebody to make something else that somebody
will sell to somebody else for more than it costs.

7. (1) From the sixteenth century onwards, people started feeling disdainful and self conscious about their
body and its products that led to a heightened focus on emotional and bodily regulations.
(2) The heightened focus on controlling the body and emotions comes from disdain and self-
consciousness about the body and its products, found in the sixteenth century.
(3) From the sixteenth century onwards, a growing disdain for and self-consciousness about the body
and its products took hold, leading to a heightened focus on emotional and bodily regulation.

iQue ideas Page 153


(4) The heightened focus on emotional and bodily regulations started from the sixteenth century
onwards, when people felt disdain and self-consciousness about the body and its products.

8. (1) We are force to fall back on fatalism as an explanation of irrational events.


(2) We are forced to falling back on the fatalism as an explanation of irrational events.
(3) We are forced to fall back on fatalism as explanations of irrational events.
(4) We are forced to fall back to fatalism as an explanation of irrational events.

9. (1) Creativity in any field is regarded not only as valuable for itself but also as a service to the nation.
(2) Creativity tin any field is not regarded only as valuable on its own, but also as a service to the nation.
(3) Creativity, in any field, is not only regarded as valuable, but also as a service to the nation.
(4) Creativity in any field is regarded not only as valuable in itself but also as a service to the nation.

10. (1) If precision of thought had facilitated precision of behaviour, and if reflection had preceded action, it
would be ideal for humans.
(2) It would be ideal for humans if reflection preceded action and if precision of thought facilitated
precision of behaviour.
(3) It would be ideal for humans if precedence of reflection was followed by action and precision of
thought, by precise behaviour.
(4) It would have been ideal for humans, if precise action and behaviour preceded precise reflection.

iQue ideas Page 154


Exercise 5

Each question is a sentence broken into four parts and labeled A, B, C, and D. Select that part which has an
error.

1. (1) You did not wait


(2) for us before you
(3) went to meet him.
(4) Isn't it?

2. (1) The police have prevented


(2) Rajan in leaving the
(3) city without informing them
(4) two days in advance.

3. (1) I would have


(2) given you the
(3) documents yesterday if
(4) you had asked for it.

4. (1) Who have you


(2) invited for the lecture
(3) on astrophysics
(4) this evening?

5. (1) A feasibility survey has now


(2) been completed in India to establish
(3) a network of felicitate contacts
(4) between small and medium enterprises.

6. (1) Privatization generally represents


(2) an ideological response
(3) to the perceived problem
(4) in the public sector.

7. (1) The Indian's government's choice


(2) of the EEC as a partner
(3) stem from the fact
(4) that the community is the most important market for India.

8. (1) A person who earns a


(2) few thousand rupees
(3) and decides to save
(4) many of it must be a miser.

9. (1) Had you been in my


(2) position, you were definitely
(3) shown your displeasure
(4) at the turn of events.

iQue ideas Page 155


10. (1) I definitely disagree
(2) with the position that
(3) requires that money
(4) is a key motivator.

11. (1) This has slowed the progress


(2) of reforms in many countries
(3) because the choice of either of the extreme
(4) positions inevitably invite criticism.

12. (1) Gavaskar was a great batsman who


(2) having played more than 100
(3) test matches, he then decided
(4) to call it a day.

13. (1) When we sold of all our


(2) furniture, crockery and
(3) other household goods.
(4) the room looked bare.

14. (1) In the history of mankind


(2) it has always been
(3) minority which have been
(4) able to change the world.

15. (1) Management education is


(2) becoming highly sought after
(3) by aspiring ambitious students
(4) because of high demand in the job market.

iQue ideas Page 156


Exercise 6

Choose the grammatically correct sentence from among the four options given for each question.

1. (1) The Excel Company has greatly enhanced it's advertising expense.
(2) The Excel Company has greatly increased its advertising expense.
(3) The Excel Company has greatly enhanced its advertising expense.
(4) The Excel Company has greatly increased it's advertising expense.

2. (1) Today we love, what tomorrow we hate; today we seek, what tomorrow we shun, today we desire,
what tomorrow we fear.
(2) Today, we love what tomorrow we hate, today, we seek what tomorrow we shun, today, we desire
what tomorrow we fear.
(3) Today we love what tomorrow we hate, today we seek what tomorrow we shun, today we desire
what tomorrow we fear.
(4) Today we love what tomorrow we hate; today we seek what tomorrow we shun; today we desire
what tomorrow we fear.

3. (1) There's Mr. Som, whom they say is the best singer in the country.
(2) There's Mr. Som, who they say is the best singer in the country.
(3) There is Mr. Som, whom they say is the best singer in the country.
(4) There is Mr. Som who, they say is the best singer in the country.

4. (1) I am not one of those who believe everything they hear.


(2) I am not one of those who believes everything I hear.
(3) I am not one of those who believes everything he hears.
(4) I am not one of those who believes in everything one hears.

5. (1) The Board of Directors will hold its next meeting in July.
(2) The Board of Directors will hold its next meeting in July.
(3) The Board of Directors shall hold the next meeting in July.
(4) The Board of Directors shall hold its next meeting in July.

6. (1) The state of his affairs were such as to cause anxiety to his creditors.
(2) The state of his affairs was such as to cause anxiety to his creditors.
(3) The state of his affairs are such as to cause anxiety to his creditors.
(4) The state of his affairs are such as to cause anxiety for his creditors.

7. (1) Cannot one do what one likes with one's own?


(2) Cannot one do that one likes to do with his own?
(3) Cannot one do that one likes with his own?
(4) Cannot one do what he likes with his own?

8. (1) Each of the students has done well.


(2) Each of the student has done well.
(3) Each of the students have done well.
(4) Each of the student have

9. (1) None of us were comfortable with what was happening.

iQue ideas Page 157


(2) None of us was comfortable with what was happening.
(3) None among us were comfortable with what was happening.
(4) None amongst us were comfortable with what was happening.

10. (1) Neither the king nor his ministers desires war.
(2) Neither king nor his ministers desires war.
(3) Neither the king nor his ministers desire war.
(4) Neither king nor his ministers desire war.

iQue ideas Page 158


Exercise 7

In each of the questions below, there are four statements which express the same idea. Choose the alternative
that is most concise and clear.

1. (1) The history of Modern Industry's performance, which is marginal at best, may be an indication of
solvency problems that will occur in the future
(2) Modern industry's history of marginal performance may indicate solvency problems in the future.
(3) The history of marginal performance of Modern Industry may indicate future solvency problems.
(4) Modern Industry's history of performance, which is marginal at best, may indicate future solvency
problems.

2. (1) On the whole food front one may say that we can enjoy festivals in the consciousness that though
some things are dear, the basic foods are better in quality and still low in price.
(2) We can enjoy festivals knowing that though some things are clear, the basic foods are of better
quality and still low in price.
(3) Although we know that some things are dear, we can still enjoy festivals because the basic foods are
better in quality and low in price.
(4) On the whole food front, although some things are dear, we can still enjoy festivals as the basic foods
are of better quality and quite cheap.

3. (1) The dull are likely to have a limited conceptual grasp.


(2) It is unlikely that the dull would have adequate conceptual grasp.
(3) The dull are not likely to grasp concepts easily.
(4) It is unlikely that the dull can grasp concepts easily.

4. (1) I request you to kindly deliver to me a tin of milk powder.


(2) Could you please send me a tin of milk powder?
(3)May I request you to please send me a tin of milk powder?
(4) Can I have milk powder sent to me please?

5. (1) There is no question of it not being possible to freeze you to death and wake you up as and when you
want.
(2) There is no question of it not being possible to freeze you to death and wake you up whenever you
want.
(3) Undoubtedly, it should be possible to freeze you to death and wake, you up whenever you want.
(4) Undoubtedly, it should be possible to freeze you to death and wake you up when you want.

6. (1) Finally, there will be unexpected, unanticipated implicational consequences of this development
concerning human life.
(2) This development concerning human life will finally have unanticipated consequences.
(3) This development concerning human life will finally have unexpected and unanticipated
implications.
(4) This development concerning human life will finally have unexpected implications.

7. (1) His definition of reality has first to be made coincident with the point of view adopted by the author
whom he is discussing.
(2) His definition of reality has first to be made coincident with the point of view adopted by the author
who he is discussing.

iQue ideas Page 159


(3) His definition has to first agree with the point of view adopted by the author he is discussing.
(4) His definition of reality has first to coincide with the point of view adopted by the author he is
discussing.

8. (1) I should be very much obliged if you could be kind enough to send me the required material.
(2) I should be much obliged if you could send me the required material.
(3) I should be very much obliged if you could kindly send me the required material.
(4) I should be grateful and very much obliged if you could kindly send me the required material

9. (1) I am sorry; a prior engagement prevents me from joining you at dinner on Monday.
(2) I regret to say that I am very sorry that a previous engagement will prevent me from joining you at
dinner on Monday.
(3) I regret to inform you that because of a previous engagement I will be prevented from joining you at
dinner on Monday.
(4) I am sorry to say that I am unable to join you at dinner on Monday because of a prior engagement.

10. (1) Looking back, our inability to confirm speakers gave us the maximum headaches.
(2) Looking at it with hindsight our inability to confirm speakers gave us the maximum headaches.
(3) On hindsight we find that our inability to confirm speakers gave us the maximum headaches.
(4) On hindsight, our inability to confirm speakers gave us the maximum headaches.

iQue ideas Page 160


Exercise 8

From the given statements, choose the one that correctly expresses the idea. .

1. (1) The best part of the programme is the dances.


(2) The best part of the programme are the dances.
(3) The best part of the programme are the dance.
(4) The best parts of the programme is the dances.

2. (1) The professor, as well as the students, was pleased with their results.
(2) The professor, as well as the students, were pleased with their results.
(3) The professor as well as the students were pleased with their results.
(4) The professor as well as the students were pleased with their results.

3. (1) He was unwilling to testify, he was afraid of the defendant.


(2) Because he was afraid of the defendant, he was unwilling to testify.
(3) He was unwilling to testify: he was afraid of the defendant.
(4) Because he was afraid of the defendant he was unwilling to testify.

4. (1) When you have good health, one should feel fortunate.
(2) When you have good health, you should feel fortunate.
(3) When one have good health, you should feel fortunate.
(4) When one has good health, he should feel fortunate.

5. (1) Either you or he have to be here.


(2) Either you or he has to be here.
(3) Neither you nor he have to be here.
(4) Neither you nor they has to be here.

6. (1) Children begin by loving their parents; as they grow older they judge them; sometimes they forgive
them.
(2) Children begin by loving their parents, as they grow older they judge them; sometimes they forgive
them.
(3) Children begin by loving their parents; as they grow older they judge them, sometimes they forgive
them.
(4) Children begin by loving their parents, as they grow older they judge them; sometimes they forgive
them.

7. (1) Gopal and Ramesh have not finished his work.


(2) Gopal and Ramesh has not finished his work.
(3) Neither Gopal nor Ramesh have finished their work.
(4) Neither Gopal nor Ramesh has finished his work.

8. (1) The fact that Raghu was a good student he had many offers for good jobs.
(2) The fact that Raghu was a good student resulted in his having many offers for good jobs.
(3) The fact Raghu was a good student resulted in him having offers for good jobs.
(4) The fact that Raghu was a good student resulted in him having many offers for good jobs.

iQue ideas Page 161


9. (1) The people of this company, have always been aware, of the needs for products of better quality and
lower price.
(2) The people of this company, have always been aware of the need for products of better quality and
lower price.
(3) The people of this company have always been aware of the need for products of better quality and
lower price.
(4) The people of this company, have always been aware of the need for products of better quality, and
lower price.

10. (1) The Dean finally agreed to see me. To talk about my financial problems.
(2) The Dean finally agreed to see me, to talk about my financial problems.
(3) The Dean, finally agreed to see me to talk about my financial problems.
(4) The Dean finally agreed to see me to talk about my financial problems.

11. (1) We invited only the people who he said were his friends.
(2) We invited only the people whom he said were his friends.
(3) We invited only the people whom he said was his friends.
(4) We invited only the person whom he said were his friends.

iQue ideas Page 162


Exercise 9

In each of the following questions a part of the paragraph or sentence has been underlined. From the choices
given to you, you are required to choose the one, which would best replace the underlined part

1. This government has given subsidies to the Navratn as but there is no telling whether the subsequent one
will do.

(1) whether the subsequent government will do so.


(2) if the government to follow will accept the policy.
(3) if the government to follow will adhere to the policy.
(4) no telling whether the subsequent one will do so.

2. Rahul Bajaj has done a great job of taking the company to its present status, but it is time that he let go
of the reins

(1) let go of the reins.


(2) stepped down.
(3) let go off the reins.
(4) delagated responsibility.

3. With the pick-up in the standard of education, expensive private schools have started blooming up in
every corner of the country.

(1) started blooming in every corner of the country.


(2) have started mushrooming all over the country.
(3) mushroomed all over the country.
(4) blossomed all over the country.

4. It is important that whatever else happens, these two factors should not be messed around with.

(1) It is important that


(2) It is a fact that
(3) It should be urgently understood that
(4) It should be understood that

5. It must be noted that under no circumstance should the company go in for diversification.

(1) It must be noticed


(2) It must be noted
(3) It must be pointed out
(4) It should be noticed

6. British Airspace has been focusing on building European links.


(1) concentrating on creating European links.
(2) pursuing ways of building European connectivity.
(3) stressing on building European links.
(4) focusing on forging European links.

iQue ideas Page 163


7. The appetite of banks for funds was lost under the onslaught of the slowdown, corporates refused to
borrow even as bank deposits flourished.

(1) bank deposits flourished.


(2) bank deposits swelled
(3) bank deposits were enhanced.
(4) bank deposits flummoxed

8. The eighth-century revival of Byzantine learning is an inexplicable phenomenon, and its economic and
military precursors have yet to be discovered.

(1) a phenomenon yet to be discovered.


(2) a phenomenon incompletely explained.
(3) an inexplicable phenomenon.
(4) an unidentifiable phenomenon

9. The management can still hire freely but cannot scold freely.

(1) cannot scold at will.


(2) cannot give umbrage
(3) cannot take decisions to scold.
(4) cannot scold willfully.

10. Many people mistake familiar for a vulgar style, and suppose that to write without affectation is to write
at random speed.
(1) is to write at random.
(2) is to write randomly
(3) is to write fast
(4) is to do speed writing

iQue ideas Page 164


Exercise 10

In each of the following sentences, a part of the sentence is underlined. Beneath each sentence, four different
ways of phrasing the underlined part are indicated. Choose the best alternative from among the four.

1. it was us who had left before he arrived.

(1) We who had left before time he had arrived.


(2) us who had went before he arrived.
(3) us who had went before had arrived.
(4) we who had let before he arrived.

2. The MP rose up to say that in her opinion, she thought the Women's Reservation Bill should be passed
on unanimously.

(1)rose to say that she thought the Women's Reservation Bill should be passed
(2)rose up to say that, the Women's Reservation Bill should be passed on
(3)rose to say that, in her opinion, she thought that the Women's Reservation Bill should be passed
(4)rose to say that, in her opinion, the Women's Reservation Bill should be passed on

3. Mr. Pillai, the president of the union and who is also a member of the community group, will be in
charge of the negotiations.

(1) since he is a member of the community group


(2) also being a member of the community group
(3) a member of the community group
(4) in addition, who is a member of the community group

4. Since the advent of cable television, at the beginning of this decade, he entertainment industry took a
giant stride forward in our country.

(1) this decade saw the entertainment industry taking


(2) this decade, the entertainment industry has taken
(3) this decade, the entertainment industry had taken
(4) this decade, the entertainment industry took

5. His mother made great sacrifices to educate him, moving house on three occasions, and severing the
thread on her loom's shuttle whenever Mencius neglected his lessons to make him understand the need to
persevere.

(1) severing the thread on her loom's shuttle whenever Mencius neglected his lessons to make him
understand the need to persevere.
(2) severed the thread on her loom's shuttle whenever Mencius neglected his lessons to make him
understand the need to persevere.
(3) severed the thread on her loom's shuttle whenever Mencius neglected his lessons to make him
understand the need for persevering.
(4) severing the thread on her loom's shuttle whenever Mencius neglected his lessons to make them
understand the need to persevere.

iQue ideas Page 165


6. If you are in a three-month software design project and, in two weeks, you've put together a programme
that solves part of the problem, show it to your boss without delay.

(1) and, you've put together a programme that solves part of the problem in two weeks
(2) and, n two weeks you've put together a programme that solves part of the problem
(3) and, you've put together a programme that has solved part of the problem in two weeks
(4) and, in two weeks you put together a programme that solved only part of the problem

7. Many of these environmentalist proclaim to save nothing less than the planet itself.

(1) to save nothing lesser than


(2) that they are saving nothing lesser than
(3) to save nothing less than
(4) that they save nothing less than

8. Bacon believes that the medical profession should be permitted to ease and quicken death where the end
would otherwise only delay for a few days and at the cost of great pain.

(1) be delayed for a few days


(2) be delayed for a few days and
(3) be otherwise only delayed for a few days and
(4) otherwise only delay for a few days and

9. The fact is that in a country where gender is perhaps one of the more restrictive forces in political life,
Hillary's and Palin's candidatures have made way for women to fudge on an equal footing, or at least
partially,

(1) Hillary's and Palin's candidature have made way for women to judge on an equal footing, or at least
partially.
(2) Hillary's and Palin's candidature has made way for women to be judged on an equal footing, or at
least partially.
(3) Hillary and Palin's candidature has made way for women to be judged on an equal footing, or at least
partially.
(4) Hillary and Palin's candidature have made way for women to be judged on an equal footing, or at
least partially.

10. The ideological warfare between secular, pseudo-secular and Hindutva groups and the government
machinery in border states give a free passage to the Bangladeshis to quietly slip into this country.

(1) the government machinery in the border states give a free passage to the Bangladeshis to quietly slip
into this country.
(2) the government machineries in the border states give a free passage to the Bangladeshis to quietly
slip into this country.
(3) the government machinery in border states gives a free passage to the Bangladeshis to quietly slip in
this country.
(4) the government machinery in the border states gives a free passage to the Bangladeshis to quietly slip
into this country.

iQue ideas Page 166


Exercise 11

In each of the questions below, four different ways of writing a sentence are indicated Choose teh best way
of writing the sentence.

1. (1) The main problem with the notion of price discrimination is that it is not always a bad thing, but that
it is the monopolist who has the power to decide who is charged what price.
(2) The main problem with the notion of price discrimination is not that it is always a bad thing, it is the
monopolist who has the power to decide who is charged what price.
(3) The main problem with the notion of price discrimination is not that it is always a bad thing, but that
it is the monopolist who has the power to decide who is charged what price.
(4) The main problem with the notion of price discrimination is not it is always a bad thing, but that it is
the monopolist who has the power to decide who is charged what price.

2. (1) A symbiotic relationship develops among the contractors, bureaucracy and the politicians, and by a
large number of devices costs are artificially escalated and black money is generated by underhand
deals.
(2) A symbiotic relationship develops among contractors, bureaucracy and politicians, and with a large
number of devices costs are artificially escalated and black money is generated through underhand deals.
(3) A symbiotic relationship develops among contractors, bureaucracy and the politicians, and by a large
number of devices costs are artificially escalated and black money is generated on underhand deals.
(4) A symbiotic relationship develops among the contractors, bureaucracy and politicians, and by large
number of devices costs are artificially escalated and black money is generated by underhand deals.

3. (1) The distinctive feature of tariffs and export subsidies is that they create difference of prices at which
goods are traded on the world market and their price within a local market.
(2) The distinctive feature of tariffs and export subsidies is that they create a difference of prices at
which goods are traded with the world market and their prices within a local market.
(3) The distinctive feature of tariffs and export subsidies is that they create a difference between prices at
which goods are traded on the world market and their prices within a local market.
(4) The distinctive feature of tariffs and export subsidies is that they create a difference across prices at
which goods are traded with the world market and their prices within a local market.

4. (1) Any action of government to reduce the systemic risk inherent in financial markets will also reduce
the risks that private operators perceive and thereby encourage excessive hedging.
(2) Any action by government to reduce the systemic risk inherent in financial markets will also reduce
the risks that private operators perceive and thereby encourage excessive gambling.
(3) Any action by government to reduce the systemic risk inherent in financial markets will also reduce
the risks that private operators perceive and thereby encourages excessive gambling.
(4) Any action of government to reduce the systemic risk inherent in financial markets will also reduce
the risks that private operators perceive and thereby encourages excessive gambling.

In the following, identify the incorrect sentence or sentences.

5. A. It was a tough situation and Manasi was taking pains to make it better.
B. Slowly her efforts gave fruit and things started improving.
C. Everyone complemented her for her good work
D. She was very happy and thanked everyone.
(1)A (2)D (3)B and C (4) A and D

iQue ideas Page 167


6. A. Harish told Raj to plead guilty.
B. Raj pleaded guilty of stealing money from the shop.
C. The court found Raj guilty of all the crimes he was charged with.
D. He was sentenced for three years in jail.

(1)A and C (2)B and D (3)A, C and D (4)B, C and D

7. A. Last Sunday, Archana had nothing to do.


B. After waking up, she lay on the bed thinking of what to do.
C. At 11 o'clock she took shower and got ready.
D. She spent most of the day shopping.

(1)B and C (2)C (3)A and B (4)B, C and D

Each statement below has a part missing. Choose the best option from the four options given below the
statement to make up the missing part.

8. Many people suggest ________ and still others would like to convince people not to buy pirated
cassettes.

(1) to bring down audiocassette prices to reduce the incidence of music piracy, others advocate strong
legal action against the offenders
(2) bringing down audiocassette prices to reduce the incidents of music piracy, others are advocating
strong legal action against offenders
(3) bringing down audiocassette prices to reduce the incidence of music piracy, others advocate strong
legal action against the offenders
(4) audiocassette prices to be brought down to reduce incidents of music piracy, others advocate that
strong legal action must be taken against offenders

9. The ancient Egyptians believed _________ so that when these objects were magically reanimated
through the correct rituals, they would be able to function effectively.

(1) that it was essential that things they portrayed must have every relevant feature shown as clearly as
possible
(2) it was essential for things they portray to have had every relevant feature shown as clearly as possible
(3) it was essential that the things they portrayed had every relevant feature shown as clearly as possible
(4) that when they portrayed things, it should have every relevant feature shown as clearly as possible

10. Archaeologists believe that the pieces of red-ware pottery excavated recently near Bhavnagar and
___________ shed light on a hitherto dark 600-year period in the Harappan history of Gujarat.

(1) estimated with a reasonable certainty as being about 3400 years old
(2) are estimated with a reasonably certain to be about 3400 years old
(3) estimated at about 3400 years old with reasonable certainty
(4) estimated with reasonable certainty to be about 3400 years old.

iQue ideas Page 168


Explanatory Answers
Chapter 15
Practice Exercise-I

Answers and Explanations

1. (2) Specially vs. especially (Specially means exclusively for, especially means — for a particular
purpose); 'especially suit the context better because 'infertility treatments' are mentioned as the class of
treatments the tourists go for. Options 3 and 4 are eliminated. 'The number of foreigners...' is the subject
of the sentence. "the number" is always singular, hence 'has' is the correct verb.

2. (1) Who's vs. whose — whose is the possessive of who. 'many features' is plural, hence the verb has to
be the plural 'have.'

3. (3) Choice is compared, hence that of (choice of) is essential. Otherwise choice and Obama are directly
compared making it an error. To identify either the similarities or the differences between two things,
use "compare to." To identify both the similarities and the differences (at the same time), use "compare
with." (`Compared with' places the things compared on equal level.) India is often compared with China.
India is often compared to the US. In comparing with something, one finds or discusses both things that
are alike and things that are different.

4. (4) The OECD is singular (that is why the verb is 'expects' rather than expect; the pronoun has to be
consistent, hence its is necessary- their is incorrect. 'mostly rich members' is correct, 'their most rich
members' is vague. `from' needs to be followed by 'to', not 'and.' Option 4 corrects these errors.

5. (1) The shift to future tense is unnecessary from the present 'cannot be represented' as it is a fact. It is an
error to change which to that, as which introduces a descriptive clause. 'one way or another' is the correct
idiom.

6. (4) Since the subject is 'the blame' — the verb has to be singular "has'. 'pinned on' is correct idiom. 'A
more calorific diet' is parallel with 'the spread of ..' and a lack of ...' Hence option 4 is faulty parallelism.

7. (3) There are there are two clauses in this sentence: The tendency to eat late is believed (hr nutritionists)
to he a factor in putting on weight and (though)it has never been tested properly. The first clause can also
be correctly constructed as "The tendency to eat late, many nutritionists believe, is a factor in putting on
weight.- But, this is not among the options. "believed .... as a factor" is incorrect.

8. (4) Portions (percent, fractions etc..) the verb will agree with the noun what follows the of (50 of the
people come from 50% of the money comes from ... etc) In this sentence the noun is machines; hence,
the verb 'use' is correct. When there is already a comma in any of the punctuation that replaces a
conjunction has to be a semicolon. In the above sentence, there is no conjunction. Option 3 makes the
sentence incorrect because the sentence is not saying "launched in 2001 in America." "The
proportion..."is the subject in the next clause; hence, the verb has to be singular 'is'.

9. (1) There is no error in the original sentence nor does it need any improvement. 'features' needs a plural
verb 'possess'. To establish comparison 'that of is necessary.

iQue ideas Page 169


10. (2) Faulty parallelism in all the other options.

11. (1) Pure subject-verb agreement question. The subject 'nuclear power generation' requires the singular
verb 'relies' and 'heavy elements' requires the plural verb 'break'.

12. (2) Tense inconsistency. The lesson of the past decade has to be either -is' or .was' , hence had been is i
eliminated. The option with was (1) continues the sentence into the present tense and is illogical. The
consistent present tense in option 2 makes the whole idea a fact. ( simple present tense is used to state
facts)

13. (3) Modifier error if 'it' is retained as the subject 'it 'cannot he 'looking.' Options I and 2 are, hence,
eliminated. Option 4 unnecessarily uses the continuous tense. and has a parallelism error. When one
looks at the path and the tact are parallel. Hence option 3 is the only correct choice.

14. (4) The verb 'says' and the pronoun 'its' are consistent with the subject "The Beijing Language and
Culture University Press".

15. (2) A colon correctly introduces the list of ideas. Semicolon in its place is incorrect. Dashes (option 3)
are incorrect because the idea (between the dashes) is not parenthetical. The second 'that' is necessary to
maintain parallelism. The possessive of Keats can be either Keats' or Keats Is.

16. (1) Commas and periods generally go inside the quotation marks (there are situations in which they are
outside). A semicolon has no place in this sentence. The comma after us. helps to remove the ambiguity
that may arise "us as well" or "listen as well." Adding of Kauai is better followed by a colon than a
comma. A comma would suffice if it was "adding to Karzai".

17. (2) 'among' vs. 'within', the latter is a better preposition in the context. 'threats to the environment' is
idiomatically correct. There is no need to change 'India's civil society'.

18. (3) 'cope up' is incorrect idiom: cope' as verb is sufficient. The first part of the sentence is a modifying
phrase subject modifier): the correct subject is 'private sector'. 'laid off is the correct past tense of 'lay off.

19. (4) 'exquisite' and 'unique' together becomes redundant. 'The religious and the secular' is parallel (both -
adjectives ).

20. (1) The subjunctive mood ( If I were. ) - "in grammar. subjunctive mood designates the mood of a verb
used to express condition. hypothesis, contingency, possibility. etc., rather than to state an actual tact:
distinguished from imperative, indicative." Also, in modern English "If I was you ..." is completely
incorrect. If .... then as a conjunction is best avoided. though it is not always incorrect: 'it- would suffice
in most situations.

21. (2) B and D. In A 'only' is misplaced: "we are the only people..." In C the pronoun 'them' is inconsistent (
in number) with its antecedent 'a small dampness': 'it' is the correct pronoun.

22. (2) B only. In A 'repeated again' is redundant. 'repeated' is correct. In C 'sentenced one year..' is
incorrect: 'sentenced to one year... will make it correct In D its is used instead of it's.

iQue ideas Page 170


23. (4) D only. In A 'gyre' needs a determiner before that 'a gyre' would be correct. In B 'ranging from an
area...' would be correct. 'from and- is incorrect. In 'as large as' or 'larger than' would correct the part.
"larger as" is incorrect.

24. (1) A only . In B. The verb 'sit' for the subject 'team' is incorrect. It should be 'sits'. In C, who's has to he
replaced with 'whose'. In D agency 's is incorrect. it should be agencies.

25. (4) D only. A should he corrected to 'one of the ... exercises'. B has to include the definite article the
Kurukshetra war" C should be corrected "another to be D has no error.

26. (4) D only A must include the definite article before US dollar - the US dollar. B has two errors:
'perceived as a monetary unit" and it's vs. its. C has two errors: since 1920's is incorrect, it should be
corrected to since the 1920s - the is necessary and no apostrophe for the plural of years.

27. (1) A and B are correct. The verb in C must be "there have been…” In D the correct idiom would he
"picked up pace”.

28. (2) B only. Sentence A uses effluent (flowing out) instead of affluent (wealthy). B is correct - the
collective noun elite is considered as individuals, hence the plural verb is correct. C is not punctuated at
all - it requires commas "Vishakha Doshi, Director, Marketing & Communication, Entrack, distributors
for Monthlane India says this way: that people identify with the core values of such legends." D is
incorrect in "each of the instrument." It should be "each of the ... instruments”.

29. (4) All are incorrect. A is incorrect in NRIs does not: it should be NRIs do not... B is incorrect as the
preposition is missing " on employment etc... is correct, but on cannot be applied to 'or on any other
purpose" hence a new preposition needs to be inserted e.g. for any other purpose". C also misses the
preposition - other than for/on employment will be correct. D is incorrect "on any other purpose"- "for
any other purpose" would be correct.

30. (1) A only. In B the preposition on is missing after but also' giving rise to faulty parallelism. C has the
incorrect preposition "invest on' rather than 'invest in'. In D, the farmer is singular hence the pronoun
they is inconsistent with it.

31. (2) C and D. There are no errors in C and D. The past participle of forget is both forgot and forgotten.
Hence D is correct. A is wrong because of its - we require it's. B has a spelling error 'dispirited' is the
correct spelling -single 's'.

32. (2) All are incorrect. A is incorrect in Just like - it should be lust as America's...". B is incorrect as
"India's" needs an apostrophe. C is incorrect as it should read "in the middle of the Indian Flag". D is
incorrect as it should read: ".... how much it is diminished by..."

33. (1) All are correct.

34. (3) A, C and D are correct. In B, the verb should be 'were' as the subject is 'the topic'.

35. (2) C and D . A is erroneous in 'found by' instead of 'founded by'. B has the incorrect plural verb 'have'
for the singular subject company.

iQue ideas Page 171


36. (4) D only. In A and C the singular destination is incorrect in the context. Plural destinations is
necessary; similarly in B,.finding as a noun and have as its verb are inconsistent.

37. (2) B only. A is incorrect in "lack in leadership'- the intended meaning is "Indian cities ... lack
leadership. C is incorrect as neither is followed by or instead of nor. D is incorrect because it should be
"an extended ...".

38. (2) B and D. A is incorrect as the correct idiom is "thanks to the mismatch..." C is incorrect because "of'
is the appropriate preposition in "survival ... of the nimble footed.-

39. (4) D only. A should be "faster than India's or India's economy."B and C have punctuation errors, "be it
power plants .... schools in China" is parenthetical a dash or a comma should be placed at the beginning
of the parenthetical phrase and at the end.

40. (2) All are incorrect. In A 'the organization' is the subject; hence the verb should be 'was'. In B 'rather
than a risky phrase cannot modify the subject American officials. The sentence can be corrected by
writing, "Rather than risk a military operation. ".. One plus one equals two: hence SRK plus KK means a
hit..." Sentence D can be corrected by changing 'turning' to 'turned'.

41. (2) B only. B is correct. Am' makes the subject singular and the verb is correctly singular is. A, C and D
are awkward and incorrect. Suggested improvements: Mild showers will become gusty and increase to
storm today. Since I became a Project Engineer in.... The project was well conceived and nicely
executed.

42. (2) A and C are correct. In C evidence is used as a verb and is correct. B is incorrect as circumstances is
not clearly temporal hence 'when' is not appropriate. 'circumstances in which...-would correct the
sentence In D whatever 'which' refers to is not clear, hence the sentence is awkward can he improved to
lie looked younger when he cut off his long hair.

43. (4) C only. A is incorrect because an adjective if used as a noun must have the definite article - the poor.
B is incorrect better off 'people needs ff in off In D 'die off' is incorrect — die of diseases will be correct.

44. (2) B only. A requires the definite article before autumn. C requires 'all of them' and not all of those. In
D the verb needs to be in the past perfect because of "by early 1942...." "had died from hunger."

45. (2) B, C, and D. A is incorrect 11,000 and not 11000. The other punctuation marks are correct.

46. (2) A and C are correct. B is incorrect because 'between' is to be followed by 'and' and not to - between
the I" and the 4th centuries or from the 1" to the 4'1' century. D is incorrect because it should be 'rather
than as conquerors..."

47. (4) None of the above. A has a tense error. With 'since' one needs to use the perfect tense. Hence A
should It's been some time since In B world's needs an apostrophe. In C drivers needs an apostrophe
after drivers'. . In D, Toyota's needs an apostrophe.

48. (3) A and D. Both the fragments need a semicolon at the end of them instead of a comma. The correct
way to punctuate a long list with commas within it ( Arun, the famous...) is as it is in fragment A.

iQue ideas Page 172


49. (3) A and D. B is incorrect in hope for becoming instead of hope of becoming. Healthy is confused with
healthful in fragment C. The context refers to healthy economies.

50. (2) B only. A is incorrect as it should read, "Not only is the Earth's atmosphere warming...." When a
senrtence begins with a negative adverb the verb order gets inverted, e.g., Never have I seen such a
beautiful.... etc. In C and D articles (determiners) are missing before review and troposphere ( a and the
respectively).

51. (2) As is a conjunction and like is a preposition; always use as, as if, as though etc. to introduce clauses.
Today afternoon is incorrect. This afternoon, this morning, tonight etc.

52. (3) "Believing that he is humble and really being humble" are not considered separately — it is a
compounded singular subject like "slow and steady wins the race"; hence, the verb is is correct. Between
options A and C — A is faulty parallelism.

53. (2) left by who knows whom' is parenthetical, hence B punctuates t correctly. Left by whom (him) is
better than `left by who (he)', though some authorities may accept "who (left it?). "who knows whom" is
more of an exclamation than an interrogation, hence the question mark may be omitted. Option B makes
the best choice.

54. (4) Sentences A and B are ambiguous. Does it mean what is stated in D or that 1 can understand my
sister better than my mother understands my sister? Sentence D is unambiguous, hence the answer.

55. (4) This is the reported speech of the sentence: Raj had a fall last week and he was injured. In the other
sentences either the tense is incorrect or the adverb (last week must become previous week in reported
speech)

56. (2) While speaking with the minister modifies the subject of the sentence. In none of the other sentence
do we have any meaningful subject that can be modified by this phrase. Option C is ambiguous. Option
D is also ambiguous.

57. (3) Option C is consistent in the use of the simple past. Shifting to tenses especially as in option A is
common in casual speech; it is not acceptable, however, in writing.

58. (1) Who versus whom, whom fits the sentence because it is the object o likes "likes whom", and not likes
who?

59. (4) Option D eliminates the redundancy of the words position and post.

60. (2) In if constructions go this way: If I had had I would have; If I had I would; If I have ... I will; If I
have ... I am)

61. (1) The verb recommended makes the subjunctive appropriate in the above sentence, hence the verb is
used in its base form "acquire" — recommended makes "must acquire" redundant.

62. (4) Though the rock band Beatles is a single entity, we think of teams as groups of individuals and use a
plural verb. Hence Beatles were is correct. Option C uses 'was' later in the sentence.

iQue ideas Page 173


63. (3) "More than one ..." though plural in sense is always singular in construction — the verb has is
correct. With a singular concept the plural pronoun 'their' is incorrect. Hence C.

64. (1) The word majority is singular when what follows it is thought of as unit. It is plural when the
members making that unit of thought of. In the above sentence 'young below thirty years of age' means
the individuals are thought of and not the unit. Hence the plural verb are is appropriate. This is also
known as 'notional agreement' in grammar.

65. (3) C is concise and clear and does with wordiness and awkward phrasing like coffee-break's duration
etc. Among the choices C is the most elegant sentence.

66. (1) Though A and C are both right (they correct the tense error of the present tense) intuition scores over
instinct in the context. For the difference refer to a dictionary. Animals have only instincts; human
beings have both —instincts and intuition.

67. (4) The verb 'stay' used alone is awkward - as if the city needs to be fixed in one place. 'stay in' means to
reside, hence correct. `to come to' is ambiguous, 'visit' clear.

68. (3) 'yourself is a reflexive pronoun and has nothing to reflect to in this sentence, hence, 'you" would be
correct. There is tense inconsistency in the other options.

69. (4) "the best" is necessary to correctly express the comparison. 'kinds of food" is correct.

70. (1) `vendors' is used a term, hence the singular verb means is correct. 'to whom' is correct and not who
and who's.

71. (2) Since it is not very clear that the collective noun 'faculty' in the context is the unit or members, it is
not possible to clearly decide the verb should be singular or plural-, option B removes the uncertainty.
CM, is a company the pronoun has to be it rather than they. "help you reach .'our goal" (without the to) is
correct.

72. (3) We need the conjunction CM in the sentence and not the preposition 'like.' Wreak is the correct word
in the context and not wreck. Wreak means to cause the i infliction of. For the word handful both
'handful' and 'handfuls' are standard plural forms — hence that part is not decisive.

73. (4) Only D is correctly' punctuated. Since the name Raj is essential to identify his friend there cannot be
a comma after friend.

74. (1) When 'however' comes between two sentences use semicolon. In other options commas are
misplaced.

75. (1) Answer Option A. Only option A is correctly punctuated.

iQue ideas Page 174


Chapter 16
Practice Exercise-II

Answers and Explanations

Exercise 1

1. (1) B only. The word immigrant is spelt incorrectly in A. B has no errors. C is incorrect because David
Stern is parenthetical hence should be surrounded by commas. D is incorrect because dry goods business
needs a determiner — 'owner of a dry goods business'. E is incorrect in tense would later become and not
would later became.

2. (4) A and D. B is incorrect because the plural pronoun 'their' is used for Nike (company) C is a incorrect
because of the comma at the end separating it from the remaining part of the phrase 'labour concerns
from the public'. E has the incorrect article a before industry — an industry is correct.

3. (4) A, C and D are correct. B is incorrect in the singular use of 'million' with few — it should be few
millions. E needs has to link the singular subject the death count. Countercharge as on word is correct
like counterchallenge.

4. (5) A and D are correct. B has a tense error it should be I associated... C also has a tense error, it should
be who seemed so innocent... E is incorrect, although is a conjunction', it is here used as an adverb — it
is better replaced with however.

5. (2) A and B. Fragment C has a tense error 'has paid 'should be 'had paid' because of 'telephoned' in the
earlier fragment. D should have 'the' before State Department. E has an inappropriate apostrophe in the
plural of 1980s. A and B are correct. Hence option 2.

6. (5) B and C. A is incorrect — it should be an intense year. In D it's is needed instead of its. E has a
parallelism error. "to stop the blog, media interactions and other events" would be correct with all the
items in the series as nouns.

7. (2) B only. A is incorrect because there should be a comma after introductory adverbs like now. ( Now
the trouble began is correct — now is not an introductory' word. B is correct. C is incorrect — systems
will have to replace should read as to be replaced. D is incorrect. It should there are considerable costs. E
is incorrect IT department is singular (the verb has is correct) but the pronoun 'their' is incorrect. The
pronoun should be its.

8. (3) A. B and E are correct. 'head into' is idiomatically correct. (Motorola is heading into an abyss) C is
incorrect because 'chances' needs a plural verb — the sentence has is. Un such a age is incorrect — it
should an age.

9. (3) A and E only. B is incorrect — the pronoun "they" is inconsistent with HortResearch which is
singular, also, "a unique and innovative fruit and food products" should be corrected to "unique and
innovative fruit and food products". C is incorrect "one of their" is inconsistent with HortResearch, and
one of previous innovative technologies is singular, hence the verb should be was and not were. D is
incorrect because adopt is confused with adapt. A and E are correct.

iQue ideas Page 175


10. (4) D only. A has a tense error — when since is used as conjunction to show the time the perfect tense is
a must. The sentence should read "Maragadavalli had been running her father's household ever since ....
". Also there is ambiguity in " ... mother had died when she was 13." B also has a tense error. "By the
time she got married" rather than "she had got married"- the unnecessary shift in tense makes the
sentence meaningless. E is incorrect. The intensifier 'very' is used incorrectly with the comparative less,
it should read very little rather than very less.

Exercise 2

1. (2) [A and D are correct]. B and C are incorrect. B is incorrect - 'As project progresses' should be
corrected to “As the project progresses..” The (definite or indefinite) article is required because project is
a countable noun — countable nouns require a determiner. C is incorrect in the phrase 'a plurality with
single-minded focus ' should be corrected to “a plurality with a single-minded focus…”. The noun
'focus' is a countable noun and needs a determiner (definite/indefinite article) ‘a focus’ is correct. (Refer
to Determiners in this book)

2. (3) [A and D are correct]. B and C are incorrect. B is incorrect because 'making them to break apart' is
incorrect idiom. The verb make (like help) does is not followed by the infinitive form of the verb but the
verb itself; We do not say I made him to speak the truth, but we say I made him speak the truth. A should
read "making them break apart", C is incorrect in 'many an offending chemicals'. The correct sentences
will be 'many offending chemicals (have)' or 'many an offending chemical (has)'.

3. (2) [A and D are correct]. B and C are incorrect. When sentences begin with negative ,adverbs (refer to
No sooner... in this book) the word order is reversed. Hence "Rarely has the economic ascent... "will be
correct. C is incorrect. 'Post war era 'is a countable noun and needs a determiner. It has to be corrected to
'The post war era'.

4. (1)[A and D are correct]. B and C are incorrect. B should read 'Ever since the Enlightenment...' (the
Enlightenment was a philosophical movement of the 18th century, hence is a specific/unique noun)
Option C is incorrect 1820s refer to a specific period hence the definite articles is required before it. "in
the 1820s "would be correct.

5. (4) C is correct [A, B and D are incorrect] A is incorrect as it should read unlike in the rest of the world,
because the comparison is between in the rest of the world and in the US electoral arena — comparison
error. B is incorrect because 'population' is referred to as 'who are'. People who are and population which
is will be correct. D is incorrect because US's aspirations is plural, hence the verb should be make and
not makes.

6. (3) A and C are correct [ B and D are incorrect] micro realities in B requires the plural verb need and not
needs. D is incorrect because Political problems need a political solution. Solution a countable noun
requires an article, or we could say need political solutions.

7. (2) B and D. [B and D are correct A and C are incorrect] secularism and pseudo secularism are just two
hence this debates between, not among. In C, the wrong idiom each another' should be corrected to 'one
another'.

8. (4) A is wrong because the pushed to limit is erroneous it should be pushed to the limit. B is wrong
because army and Border Security Force both need determiners before them - the army and the Border

iQue ideas Page 176


Security Force. C is incorrect because meagre resources refers to an already identified source hence the
definite article is required 'the meagre resources'. The number takes a singular verb, hence D is correct.

9. (3) A and D. [B and C are incorrect] B is incorrect as it should read 'one of the districts' as 'one of is
always followed by a plural noun. In C the modifier only is misplaced.. the sentence should read `... has
only one coordination office....', also 'meeting the needs can be improved to 'to meet the needs'.

10. (1) A and D. B and C are incorrect] Both B and d contain the incorrect its without the apostrophe. As
both are contraction of it is, the apostrophe should be used. B has faulty parallelism. •... of so many
things. so many moments. so many emotion' would be correct.

Exercise 3

1. (3) I've laughed, cried .... And fallen in love and not fell in love.. - tense error in C. I've spit or I've -
participle is spit and spat, both are correct. .

2. (5) 'Raising kids' is one of the commonest errors found in informal speech and writing. We 'rear kids' not
raise them. Raising in the sense of bringing up is used only for animals. We raise pigs and we raise
horses and rear children.

3. (1) It 's should be used instead of its in the first part of the sentence.

4. (1) "As a kid..." is a modifier — it seems to modify 'my father' which is erroneous. We should correct it
to 'when I was a kid.....'or "As a kid I used to accompany my father to...."

5. (5) 'Any of you look like' and not 'any of you looks like' though 'nay of you' is singular, it is still you
which takes a plural verb in English. 'any of them' is 'he, she or it hence takes singular.

6. (4) D should read as 'in which I walked into ... 'or 'when I walked into ....'

7. (4) D the preposition for should be corrected to of— 'right of another% rather than 'right for another'
Answer Option A. Subject — verb agreement problem in option A.

8. (1) The analyses (of data) is plural, hence the verb in fragment A should be show and not shows. We
may also correct the sentence by retaining the verb shows and changing analyses to analysis (singular)
— the error in either case is in fragment A.

9. (4) 25 percentage less is erroneous — 25 percent less is correct usage. Use the word "percent" with
numbers; use "percentage" without using a number. For example: 50 percent children, and a considerable
percentage of children.

10. (4) Parallelism error. Reading, learning languages are in ing form, but crossword puzzles ( a noun) is not
parallel. It should be changed to doing crossword puzzles. We can also make the sentence parallel by
changing it to : focused on books, crossword puzzles, and languages. Then the error will be in two
fragments. Hence it is not possible to work the question that way.

11. (4) D carries the incorrect idiom 'base for vote-gathering'. It should be corrected to basis for vote-
gathering'.

iQue ideas Page 177


12. (4) 'numbers of centuries' is incorrect. Though the writer is referring to both types of the game, Sachin in
both has the highest number and not the highest numbers.

13. (5) We will not allow anyone to sell their votes has a problem in pronoun agreement, anyone is singular
hence he or she should be used rather than the plural their. Sixty years as period is singular, hence the
singular verb is correct. There is no error of parallelism. There is no shift in tense.

14. (2) Fragment B has serious errors. First is that the pronoun they has no proper antecedent. The
antecedent seems to be the Indian Left — which is to be treated a singular. The next error is the
collective will begin with an article, hence they are an honest and committed bunch of will be correct.
The other fragments have no problems.

15. (2) Will not gel into is non standard and has no meaning. Gel or jell as a verb means to solidify or set.
Gel jell with is generally used colloquially. In formal text it is an error. The second error is in the tense
'will not' should be changed to 'would not'.

Exercise 4

1. (3) for the student's conduct...'should be replaced by 'is conduct.' Pronoun has to be used to avoid
redundancy.

2. (4) 'Every man and woman' is singular hence the pronoun that refers to every should be singular - 'his or
her' rather than 'their'.

3. (3) 'one' is gender neutral and singular, hence it cannot be replaced with an other pronoun (you). The
correct sentence should read "one should choose that option" in C.

4. (4) Confusion between assure and ensure. D should read ensure a successful career. Ensure means make
sure that '-something happens - in other words it is followed by a noun clause that… Assure on the other
hand means convince someone.

5. (3) The man is the object of the verb found, hence 'who' should be replaced with 'whom'. Refer to who
and whom' in this book. Found him + they believe him to be.. etc. hence whom.

6. (2) The running of large businesses is singular hence consists of 'more than it costs' is correct.

7. (3) Option C expresses the temporal and cause-effect relationship between the ideas clearly.

8. (1) 'to falling back' eliminates B. 'Explanations' eliminates C. 'fall back to' eliminates D. A is the best
option.

9. (4) parallelism with not only but also is maintained in option D.. All other options have faulty
parallelism.

10. (2) Option B expresses the different ideas in the sequence they are required. Option B maintains the
tense sequence with if clauses.

iQue ideas Page 178


Exercise 5

1. (4) The question tag should repeat the auxiliary verb in the main clause. D should hence be 'did you?'

2. (2) The correct idiomatic usage is 'prevented someone from leaving'. The noun police always takes a
plural verb. Hence the part A is correct. Remember, a singular verb the with the noun 'police' is wrong.

3. (4) The 'documents' is a plural noun. The pronoun in option D should hence be them. 4.

4. (1) "Whom have you invited...." will be correct (refer to who vs. whom in this book.

5. (3) Fragment C is meaningless. First, felicitate is confused with facilitate. Second, network to facilitate is
expressed as network of .... Fragment C should read, a network to facilitate contacts.

6. (3) A plural noun should be for a generic reference to the problems in the public sector. 'the perceived
problem makes the problem specific. The correct phrase in the context is 'to the perceived problems'.

7. (3) 'The Indian Government's choice' is the subject of the sentence and is a singular noun . The verb stem
should agree with it and should be singular 'stems'.

8. (4) a few thousand rupees is an amount. Hence one can save most of the amount and many of the rupees.
Hence fragment D should be corrected to 'most of it

9. (2) Refer to the guidelines for framing sentences with if. Had you been is equivalent of if you had been
— the past perfect in the if clause has to be followed up with conditional perfect (would have shown) in
the main clause. Hence. instead of 'were' one has to use would have in fragment B.

10. (3) 'Awkward phrasing' in fragment C. because of requires. A position requires prior experience is
idiomatically correct. In this context position has a different meaning. Requires in the context has to be
replaced with assumes to make the sentence idiomatically correct.

11. (4) Subject verb agreement error. Since the subject is the singular noun "choice", the verb should be
'invites'.

12. (3) 'he then' is unnecessary and erroneous grammatically. 'who' is the subject in the clause decided to oil/
it a day. 'who, he then' is incorrect. 'he then should be deleted from the fragment C.

13. (1) 'Of should be replaced with 'oil'. Sold off is the correct idiom.

14. (3) Here 'minority' refers to a specific part of the population; hence the definite article 'the' should be
used.

15. (3) Aspiring students creates an ambiguity; are they aspiring to be students? The first adjective is
(aspiring) is out of place. Even if it can be retained one will have to write aspiring and ambitious. Refer
to adjectives in this book.

Exercise 6

iQue ideas Page 179


1. (2) "Enhanced' means to increase or improve the quality or value or desirability of something. Increase
implies to become greater in size, amount, number or intensity. The correct usage here would be
'increased'. The possessive form 'its' and not the contracted 'it's is required.

2. (2) The sentence is divided into three different clauses hence each should be separated by a semicolon.

3. (2) The clause, 'they say is the best singer in the country'describes the subject Mr. Som, hence we need to
use 'who' Choice D is wrong due to the wrong placement of the comma after 'who'.

4. (1) The relative pronoun who refers to 'those' which is plural hence a plural verb believe has to be used,
and the pronoun used should refer to 'those', hence 'they hear'.

5. (1) 'Its' and not 'it's' is the possessive form for `it'.'It's' means 'it is'. Option C is ambiguous in that it raises
the question 'which meeting?'

6. (2) The 'state of affairs' is a singular noun hence should be followed by a singular verb 'was'.

7. (1) The pronoun 'one' (gender neutral singular) cannot be changed to he/she, and should remain 'one'
throughout the sentence.

8. (1) 'Each or like 'one of will always be followed by a plural noun (students). Each as the subject will
always take a singular verb.

9. (1) 'None' in the absence of anything to prevent the plural verb will take a plural verb. None of the other
options are entirely wrong, and can be accommodated. Since we are specifically asked to choose
between correct options, the more often used construction in A becomes the answer.

10. (3) Because we are talking about a particular king we should use 'the king'. Moreover, the verb should be
consistent with the noun after 'nor'. The noun ministers (plural) takes the verb because it is near to the
verb, hence desire.

Exercise 7

1. (3) C precise and to the point. Others are vague, confusing or too wordy.

2. (2) All other options are either confusing or unnecessarily wordy. 'on the whole food front' is
unnecessary. "of better quality' is a better expression than 'better in quality'.

3. (3) Option C is the most concise and clear.

4. (2) Option B is concise, polite, and clear in communication.

5. (3) Between C and D 'Whenever' communicates the freedom to wake up better.

6. (2) 'Unexpected' and 'unanticipated' are synonyms, so using both in a sentence is redundant.
'consequence' is a better word to suggest the effect of something on something else.

7. (3) The appropriate idiomatic use is 'definition should agree with'.

iQue ideas Page 180


8. (2) Option B is sufficiently polite, clear and concise. 'very' is redundant in all the other options.

9. (1) Option A is sufficiently polite, clear and concise.

10. (1) Option A is clear and concise. "Looking back' is the correct idiomatic usage.

Exercise 8

1. (1) The subject here is 'the best part', which is singular and should therefore be followed by a singular
verb is'. The plural complement/object (dances) of the verb has no impact on the verb.

2. (1) 'as well as' is a preposition and introduces a prepositional phrase; hence, the phrase should be set off
by commas, and the verb agrees with the main subject, which in this case is 'the professor'.

3. (2) A is wrong with the comma trying to do the work of a conjunction. C is wrong because semicolon is
wrongly used between clauses. D is wrong because when the subordinate conjunction begins the
sentence, the main clause is set off using a comma.

4. (2) The pronoun should remain consistent throughout the sentence. When you have ... you should...

5. (2) When 'either' and 'neither' are followed by 'or' and 'nor' respectively, the number of the verb
(singular/plural) verb depends on the noun following 'or' and 'nor'.

6. (1) These are three separate sentences. The only punctuation possible is either a period after each
sentence or a semicolon after each sentence.

7. When 'neither' is followed by 'nor', the verb depends on the noun following 'nor'. In this case it is
singular; hence the verb should also be singular. The pronoun will also agree with the noun that follows
'nor'.

8. (2) Before a gerund (having many offers) a possessive pronoun is required. Option A is wrong without a
conjunction.

9. (3) The sentence does not require any commas. Hence the other options get eliminated.

10. (3) Incorrect punctuation makes all the other options wrong.

11. (1) 'who' is the subject of the verb 'were'.

Exercise 9

1. (4) Since there is a shift in tense from present perfect to future in the two clauses the verbs have to be
complete, "will do" in the second clause is not sufficient to indicate what it will do. Hence "will do so' is
required.

2. (1) 'to let go of the reins' is the correct idiom. Hence no change required.

3. (3) Option C used the correct idiom — mushroom which means to spread/proliferate.

iQue ideas Page 181


4. (4) Parallel construction: It should be... these should not be. Option C brings in an unnecessary urgency.

5. (2) "company go in" in the sentence justifies the 'must' in the earlier part. The best expression is in the
original sentence. Hence no change required.

6. (4) `To forge' implies to create a lasting relationship based on hard work.

7. (2) Since the context requires a physical expansion in size swelled is more appropriate than flourish
(would imply increase in numbers).

8. (3) The original is the best phrasing.

9. (1) 'At will' implies as and when one likes which is the intended meaning of the sentences. Freely would
imply without any reason.

10. (1) 'speed- has no significance in the sentence — hence the original needs to be corrected. 'at random'
best expresses the idea.

Exercise 10

1. (4) The verbs is, was, are, were, will be always followed by a subject pronoun. The tense sequence is
maintained in D.

2. (1) Rose up is redundant, in her opinion, she thought, is redundant, passed on is incorrect idiom in the
context.

3. (3) Two elements in parallel to describe Mr. Pillai. Other options are wordy.

4. (2) Since takes the perfect tense. This decade requires present perfect.

5. (1) Parallelism error in other options. Moving, severing will be parallel. Option D has a pronoun error in
'them'.

6. (2) The other options distort the meaning of the sentence. In two weeks is not parenthetical hence the
original is incorrect for the commas.

7. (4) 'Proclaim that they save...' is better expression than the available options.

8. (3) 'should be permitted....' and would be delayed ....'are parallel.

9. (3) There is no necessity to treat Hilary's and Palin's candidature separately because in the context their
candidature is treated as a trend. That makes the verb singular. `to be judged' communicates the intended
meaning.

10. (4) The ideological warfare is singular, hence the verb should be gives. Border states is a countable
specific noun hence the definite article is required. Both these errors in the original are rectified in option
D.

iQue ideas Page 182


Exercise 11

1. (3) Parallelism — Not that it is ... but that it is...

2. (2) 'generate money through deals' will be correct rather than by and on. The generic reference to
contractors etc. does not require an article. 3.

3. (3) 'prices on the world market' and the 'prices within a local market' the preposition between is called
for.

4. (2) Reduce ... reduce... encourage — parallelism. Action by someone is different in meaning from action
of someone. Action by government is appropriate in this context.

5. (3) The correct idiom to use in sentence 2 will be 'bear fruit. In sentence 3 complement is confused with
compliment.

6. (2) 2 and 4. In both sentences the preposition 'of should be replaced with to correct the error in idiom.
Plead guilty to and sentenced to are correct. .

7. (2) should be corrected to lay in bed thinking what to do'. 3 should read took 'a shower' rather than took
shower.

8. (3) Incidence vs. incidents — incidence (occurrence); incidents (events). Suggest bringing down is more
appropriate than suggest to bring down.

9. (3) A is wrong because must have after essential is redundant. B is wrong in the tense used 'they portray'
after believed and it was... D is wrong in the pronoun it referring to things.

10. (4) 'a reasonable certainty' eliminates options A and B. In C, with reasonable certainty does not modify
old but estimated.

iQue ideas Page 183

You might also like